SlideShare a Scribd company logo
1 of 83
ĐẠI HỌC QUỐC GIA HÀ NỘI
TRƯỜNG ĐẠI HỌC KHOA HỌC TỰ NHIÊN
————————–
VŨ THỊ HIỀN
SÁU PHƯƠNG PHÁP GIẢI CÁC
BÀI TOÁN PHỔ THÔNG
LUẬN VĂN THẠC SĨ KHOA HỌC
Hà Nội - 2015
ĐẠI HỌC QUỐC GIA HÀ NỘI
TRƯỜNG ĐẠI HỌC KHOA HỌC TỰ NHIÊN
————————–
VŨ THỊ HIỀN
SÁU PHƯƠNG PHÁP GIẢI CÁC
BÀI TOÁN PHỔ THÔNG
Chuyên ngành: Phương pháp toán sơ cấp
Mã số : 60460113
LUẬN VĂN THẠC SĨ KHOA HỌC
Người hướng dẫn khoa học: GS.TS. Đặng Huy Ruận
Hà Nội - 2015
Mục lục
Mở đầu 1
1 Phương pháp quy nạp 2
1.1 Nguyên lý quy nạp . . . . . . . . . . . . . . . . . . . . . . . . . . . 2
1.2 Phương pháp chứng minh bằng quy nạp . . . . . . . . . . . . . . . 2
1.2.1 Cơ sở quy nạp . . . . . . . . . . . . . . . . . . . . . . . . . . 3
1.2.2 Quy nạp . . . . . . . . . . . . . . . . . . . . . . . . . . . . . 3
1.2.3 Vận dụng phương pháp quy nạp để giải một số bài toán . 4
2 Phương pháp chứng minh phản chứng 17
2.1 Cơ sở lý thuyết . . . . . . . . . . . . . . . . . . . . . . . . . . . . . 17
2.2 Nội dung của phương pháp phản chứng . . . . . . . . . . . . . . . 18
2.3 Trình bày lời giải của phương pháp phản chứng . . . . . . . . . . . 19
2.4 Một số ví dụ minh họa . . . . . . . . . . . . . . . . . . . . . . . . . 19
3 Phương pháp suy luận trực tiếp 28
3.1 Vài nét về phương pháp suy luận trực tiếp . . . . . . . . . . . . . . 28
3.2 Các ví dụ về vận dụng phương pháp suy luận trực tiếp . . . . . . 29
4 Phương pháp đồ thị 35
4.1 Một số khái niệm và kết quả cơ bản của lí thuyết đồ thị . . . . . . 35
4.2 Phương pháp đồ thị . . . . . . . . . . . . . . . . . . . . . . . . . . . 36
4.2.1 Xây dựng đồ thị mô tả các quan hệ . . . . . . . . . . . . . 37
4.2.2 Dựa vào các kết quả của lý thuyết đồ thị hoặc lý luận trực
tiếp suy ra đáp án của bài toán D . . . . . . . . . . . . . . 37
4.3 Một số ví dụ . . . . . . . . . . . . . . . . . . . . . . . . . . . . . . . 37
5 Phương pháp bảng 53
5.1 Giới thiệu về phương pháp bảng . . . . . . . . . . . . . . . . . . . . 53
i
MỤC LỤC
5.2 Một số ví dụ minh họa . . . . . . . . . . . . . . . . . . . . . . . . . 53
6 Phương pháp sơ đồ 67
6.1 Các bước thực hiện phương pháp sơ đồ . . . . . . . . . . . . . . . 67
6.1.1 Thiết lập sơ đồ . . . . . . . . . . . . . . . . . . . . . . . . . 67
6.1.2 Dựa vào cấu trúc của sơ đồ mô tả quan hệ và điều kiện
đã cho trong bài toán mà suy ra đáp án . . . . . . . . . . . 67
6.2 Một số ví dụ . . . . . . . . . . . . . . . . . . . . . . . . . . . . . . . 67
Kết luận . . . . . . . . . . . . . . . . . . . . . . . . . . . . . . . . . . . . 77
Tài liệu tham khảo . . . . . . . . . . . . . . . . . . . . . . . . . . . . . 79
ii
Mở đầu
Toán phổ thông chẳng những nhiều về số lượng, còn phong phú về chủng
loại.
Mỗi chủng loại đòi hỏi một phương pháp giải thích hợp. Bởi vậy có nhiều
phương pháp giải toán phổ thông.
Với khối lượng có hạn, luận văn chỉ xin phép trình bày sáu trong những
phương pháp thường dùng nhất.
Luận văn gồm phần mở đầu và sáu chương:
Chương I trình bày về phương pháp quy nạp,
Chương II trình bày về phương pháp phản chứng,
Chương III trình bày về phương pháp suy luận trực tiếp,
Chương IV trình bày về phương pháp đồ thị,
Chương V trình bày về phương pháp bảng,
Chương V I trình bày về phương pháp sơ đồ.
Mỗi phương pháp đều có phần tóm tắt cơ sở lý thuyết và phần vận dụng
phương pháp để giải bài tập.
Luận văn được hoàn thành dưới sự hướng dẫn tận tình của thầy giáo GS. TS
Đặng Huy Ruận. Em xin bày tỏ lòng kính trọng và biết ơn sâu sắc đến Thầy.
Em xin trân trọng cảm ơn ban lãnh đạo khoa Toán - Cơ - Tin học, khoa Sau
Đại học, Trường Đại học Khoa học tự nhiên, Đại học Quốc Gia Hà Nội, các
Thầy, Cô giáo đã trang bị kiến thức, tạo điều kiện cho chúng em trong thời gian
học tập tại đây. Tôi cũng xin gửi lời cảm ơn đến các Đồng nghiệp tại trường
Phổ Thông Hồng Đức - Hà Nội, những người đã động viên giúp đỡ tôi rất nhiều
trong quá trình hoàn thành luận văn này.
Luận văn khó tránh khỏi hạn chế và sơ xuất. Rất mong được sự chỉ bảo của
Quý thầy cô và Quý bạn đọc để luận văn được hoàn thiện hơn.
Tôi xin chân thành cảm ơn!
1
Chương 1
Phương pháp quy nạp
Phương pháp quy nạp có vai trò vô cùng quan trọng trong toán học, khoa
học và cuộc sống. Đối với nhiều bài toán trong chương trình toán phổ thông là
những bài toán logic, tức những bài toán không mẫu mực phương pháp quy nạp
cho ta nhiều cách giải hữu hiệu.
Suy diễn là quá trình từ "tính chất" của tập thể suy ra tính chất của cá thể,
nên luôn luôn đúng, còn quá trình ngược lại, tức quá trình quy nạp: đi từ "tính
chất" của một số các thể suy ra "tính chất" của tập thể thì không phải lúc nào
cũng đúng, mà quá trình này chỉ đúng khi nó thỏa mãn một số điều kiện nào
đó, tức thỏa mãn nguyên lý quy nạp.
1.1 Nguyên lý quy nạp
Nếu khẳng định S(n) thỏa mãn hai điều kiện sau:
a) Đúng với n = k0 (số tự nhiên nhỏ nhất mà S(n) xác định).
b) Từ tính đúng đắn của S(n) đến n = t (hoặc đối với mọi giá trị của n (k0 ≤
n ≤ t)) (t ≥ k0), ta cần chứng minh tính đúng đắn của S(n) đối với n = t + 1, thì
khiØS(n) đúng với mọi n ≥ k0.
1.2 Phương pháp chứng minh bằng quy nạp
Giả sử khẳng định S(n) xác định với mọi n ≥ t0. Để chứng minh S(n) đúng
∀n ≥ t0 bằng quy nạp ta cần thực hiện theo hai bước sau:
2
Chương 1. Phương pháp quy nạp
1.2.1 Cơ sở quy nạp
Thực hiện bước này tức là ta thử xem sự đúng đắn của S(n) với n = t0 nghĩa
là xét S(t0) có đúng hay không?
1.2.2 Quy nạp
Giả sử khẳng định S(n) đã đúng đến n = t (hoặc đối với mọi n (t0 ≤ n ≤ t))
(t ≥ t0). Trên cơ sở giả thiết này ta chứng minh tính đúng đắn của S(n) đối với
n = t + 1, tức S(t + 1) đúng.
Nếu cả ba bước trên thỏa mãn, thì theo nguyên lý quy nạp S(n) đúng với
∀n ≥ t0.
Chú ý: Trong quá trình quy nạp nếu không thực hiện đầy đủ cả ba bước: Cơ
sở quy nạp, giả thiết quy nạp và chứng minh quy nạp, thì có thể dẫn đến kết
quả sai lầm, chẳng hạn:
- Do bỏ bước cơ sở quy nạp, ta đưa ra kết luận không đúng: Mọi số tự nhiên
đều bằng nhau! Bằng cách quy nạp như sau: Giả sử các số tự nhiên không vượt
quá k + 1 đã bằng nhau. Khi đó ta có
k = k + 1
Thêm vào mỗi vế của đẳng thức trên một đơn vị ta có
k + 1 = k + 1 + 1 = k + 2
Cứ như vậy suy ra mọi số tự nhiên không nhỏ hơn k đều bằng nhau. Kết hợp
với giả thiết quy nạp: Mọi số tự nhiên không vượt quá k đều bằng nhau, đi đến
kết luận sai lầm: Tất cả các số tự nhiên đều bằng nhau!
- Do bỏ qua khâu quy nạp nên nhà toán học Pháp P.Fermat (1601-1665) đã
cho rằng các số dạng 22n
+ 1 đều là số nguyên tố.
P.Fermat xét 5 số đầu tiên:
Với n = 0 cho 220
+ 1 = 21
+ 1 = 3 là số nguyên tố.
n = 1 cho 221
+ 1 = 22
+ 1 = 5 là số nguyên tố.
n = 2 cho 222
+ 1 = 24
+ 1 = 17 là số nguyên tố.
n = 3 cho 223
+ 1 = 28
+ 1 = 257 là số nguyên tố.
n = 4 cho 224
+ 1 = 216
+ 1 = 65537 là số nguyên tố.
3
Chương 1. Phương pháp quy nạp
Nhưng vào thế kỷ 18 Euler đã phát hiện với n = 5 khẳng định trên không
đúng, bởi vì:
225
+ 1 = 4294967297 = 641 × 6700417
là hợp số.
1.2.3 Vận dụng phương pháp quy nạp để giải một số bài toán
Phương pháp quy nạp được sử dụng trong tính toán, trong chứng minh và
trong suy luận dưới nhiều dạng khác nhau, nhưng trong phần này chỉ trình bày
việc vận dụng phương pháp quy nạp để giải các bài toán logic, tức các bài toán
"không mẫu mực".
Ví dụ 1.2.1. Chứng minh rằng: Nếu trong túi có một số tiền nguyên (nghìn)
không ít hơn 8000đ, thì luôn luôn có thể mua vé sổ số loại 5000đ và 3000đ.
Lời giải: Ta sẽ giải quyết bài toán này bằng phương pháp quy nạp.
1) Cơ sở quy nạp. Nếu trong túi có số tiền ít nhất, tức 8000đ, thì ta mua một
vé sổ số loại 5000đ và một vé sổ số loại 3000đ. Khi đó
1 × 5000đ + 1 × 3000đ = 8000đ
và ta đã tiêu được hết số tiền có trong túi.
2) Quy nạp. Giả sử với k(k ≥ 8000) nghìn đồng ta đã tiêu hết bằng cách mua
các vé sổ số loại 5000đ và 3000đ. Nếu có thêm 1000đ nữa ta cũng có thể mua
được bằng cách sau đây:
a) Nếu trong các vé sổ số đã mua có ít nhất ba vé loại 3000đ, thì ta trả lại
ba vé loại 3000đ, đưa thêm 1000đ và lấy về hai vé loại 5000đ. Khi đó
3 × 3000đ + 1000đ = 2 × 5000đ.
b) Nếu trong các vé sổ số đã mua có không quá hai vé loại 3000đ, thì phải
có ít nhất một vé loại 5000đ. Bởi vì trong túi không ít hơn 8000đ, mà đã tiêu
hết. Khi đó đem trả lại một vé loại 5000đ, đưa thêm 1000đ và lấy về hai vé loại
3000đ, ta có
1 × 5000đ + 1000đ = 2 × 3000đ
Như vậy trong mọi trường hợp từ kết quả tiêu k nghìn đầu tiên đã suy ra
được cách tiêu nghìn thứ k + 1, nên bài toán đã được giải quyết xong.
4
Chương 1. Phương pháp quy nạp
Ví dụ 1.2.2. Em An cầm một tờ giấy và lấy kéo cắt thành 7 mảnh. Sau đó nhặt
một trong những mảnh giấy đã cắt và lại cắt thành 7 mảnh. Và em An cứ tiếp
tục cắt giấy như vậy. Sau một hồi em An thu tất cả các mẩu giấy đã cắt ra và
đếm được 122 mảnh. Liệu em An đếm đúng hay sai?
Lời giải:
1) Mỗi lần cắt mảnh giấy thành 7 mảnh, tức là đã tạo ra thêm 6 mảnh giấy,
nên công thức tính số mảnh giấy sau n bước thực hiện một mảnh giấy thành 7
mảnh có dạng: S(n) = 6n + 1.
2) Tính đúng đắn của công thức S(n) được khẳng định bằng quy nạp theo n.
10) Cơ sở quy nạp. Với n = 1, em An cắt mảnh giấy có trong tay thành 7
mảnh, nên có
S(1) = 6.1 + 1 = 6 + 1 = 7
20) Quy nạp. Giả sử sau k bước em An đã nhận được số mảnh giấy là
S(k) = 6k + 1
Sang bước k + 1 em An lấy một trong những mảnh giấy nhận được trong k
bước trước và cắt thành 7 mảnh, tức em An đã lấy đi một trong S(k) mảnh và
thay vào đó 7 mảnh được cắt ra nên
S(k + 1) = S(k) − 1 + 7 = 6k + 1 − 1 + 7 = 6k + 7
= 6k + 6 + 1 = 6(k + 1) + 1
Vậy số mảnh giấy em An nhận được sau n bước cắt giấy là S(n).
3) Do S(n) = 6n + 1 ≡ 1 (mod 6), nhưng 122 = 6.20 + 2 ≡ 2 (mod 6), nên em
An đếm không đúng.
Ví dụ 1.2.3. (Chứng minh tính chất bằng quy nạp).
Cho x + 1
x, x = 0 là một số nguyên. Chứng minh rằng với mọi số nguyên dương
n, số
T(n, x) = xn
+
1
xn
cũng là số nguyên.
Lời giải: Khẳng định được chứng minh bằng quy nạp.
1) Cơ sở quy nạp. Với n = 1 có T(1, x) = x + 1
x là số nguyên, theo giả thiết.
2) Quy nạp. Giả sử khẳng định đúng với mọi số nguyên k(n ≥ k ≥ 1) nghĩa
là
T(k, x) = xk
+
1
xk
5
Chương 1. Phương pháp quy nạp
là số nguyên.
Với n = k + 1 số
T(k + 1, x) = xk+1
+
1
xk+1
=
= x +
1
x
xk
+
1
xk
− xk−1
+
1
xk−1
theo giả thiết quy nạp, các số x + 1
x, xk−1 + 1
xk−1 và xk + 1
xk đều nguyên, nên
T(k + 1, x) là số nguyên và khẳng định đúng với mọi số nguyên dương n.
Ví dụ 1.2.4. Chứng minh rằng mọi số nguyên lớn hơn 1 đều có thể viết dưới
dạng tích của các thừa số nguyên tố.
Lời giải: Ta chứng minh khẳng định bằng quy nạp theo n.
1. Cơ sở quy nạp. Với n = 2, ta có 2 = 2,
Với n = 3, ta có 3 = 3, n = 4, ta có 4 = 2 × 2
Vậy khẳng định đúng với n = 2, 3, 4.
2. Quy nạp. Giả sử với mọi số nguyên n đều phân tích được thành tích của
các thừa số nguyên tố. Ta chứng minh n+ 1 cũng phân tích được thành tích của
các thừa số nguyên tố.
Thật vậy
• Nếu n + 1 là số nguyên tố thì nó bằng tích của chính n + 1.
• Nếu n + 1 là hợp số thì n + 1 = a.b với 2 ≤ a, b < n.
Theo giả thiết quy nạp, thì a, b đều phân tích được thành tích các thừa số
nguyên tố.
Suy ra, n + 1 cũng phân tích được thành tích các thừa số nguyên tố.
Theo nguyên lý quy nạp, mọi số nguyên n > 1 đều phân tích được thành tích
các thừa số nguyên tố.
Ví dụ 1.2.5. (Chứng minh tính chia hết bằng quy nạp). Chứng minh rằng với
mọi số nguyên dương n số 23n
+1 chia hết cho 3n+1 23n
+ 1
...3n+1 và số 23n
+1
không chia hết cho 3n+2 23n
+ 1
...3n+2 .
Lời giải: Bài toán được giải quyết bằng quy nạp. Kí hiệu 23n
+ 1 = An.
1) Cơ sở quy nạp.
Với n = 1 ta có A1 = 231
+ 1 = 23 + 1 = 8 + 1 = 9, nên A1
...32 và A1
...33.
Với n = 2 ta có A2 = 232
+ 1 = 513, nên A2
...33 và A2
...34.
6
Chương 1. Phương pháp quy nạp
2) Quy nạp. Giả sử khẳng định đã đúng với n = k ≥ 2, nghĩa là Ak
...3k+1 và
Ak
...3k+2.
Vì Ak
...3k+1, nên
∃M ∈ N (Ak = M.3k+1
và M
...3) (1.1)
Xét n = k + 1
Ak+1 = 23k+1
+ 1 = 23k
.3
+ 1 = 23k 3
+ 1 = 23k
+ 1 23k 2
− 23k
+ 1
= 3k+1
.M. 23k
+ 1
2
− 3.23k
= 3k+1
.M. 3k+1
.M
2
− 3.23k
= 3k+1
.M. 32k+2
.M2
− 3.23k
= 3k+2
.M. 32k+1
.M2
− 23k
10) Khi đó Ak+1
...3k+2, nên với mọi số nguyên dương n đều có 23n
+ 1
...3n+1.
20) Ak+1
...3k+3.
a) Vì M
...3 (theo (1.1)), nên
3k+2
.M
...3k+3
(1.2)
b) Do k ≥ 2, nên ∃t ∈ N (k = t + 2) và 32k+1.M2 = 3k+3+t.M2.
Bởi vậy
32k+1
.M2...3k+3
(1.3)
Giả sử 23k ...3k+3. Khi đó 23k ...9, nhưng 23k
= 23 k
= 8k = ±1 (mod 9), nên
23k ...3k+3
(1.4)
Từ các quan hệ (1.3) và (1.4) ta suy ra
32k+1
.M2
− 23k ...3k+3
(1.5)
Từ (1.2) và (1.5) suy ra: Ak+1
...3k+3, nên với mọi số nguyên dương n số An không
chia hết cho 3n+2.
Ví dụ 1.2.6. (Chứng minh bất đẳng thức bằng quy nạp)
Cho n(n ≥ 1) số dương x1, x2, · · · , xn thỏa mãn điều kiện
x1.x2. · · · xn−1.xn = 1
Chứng minh rằng
x1 + x2 + · · · + xn−1 + xn ≥ n
và dấu đẳng thức xảy ra khi và chỉ khi x1 = x2 = · · · = xn
7
Chương 1. Phương pháp quy nạp
Lời giải: Bài toán được giải quyết bằng quy nạp.
1) Cơ sở quy nạp. Với n = 1 ta chỉ có số x1. Vì x1 = 1, nên x1 thỏa mãn bất
đẳng thức x1 ≥ 1.
2) Quy nạp. Giả sử khẳng định đã đúng với k số dương tùy ý có tích bằng 1.
Xét k + 1 số dương tùy ý x1, x2, · · · , xk, xk+1 với x1.x2. · · ·xk.xk+1 = 1. Có hai khả
năng đặt ra:
a) Nếu x1 = x2 = · · · = xk = xk+1, thì xi = 1 (1 ≤ i ≤ k + 1). Khi đó
x1 + x2 + · · · + xk + xk+1 = k + 1
và khẳng định được chứng minh.
b) Nếu k + 1 số được xét x1, x2, · · · , xk, xk+1 không đồng thời bằng nhau, thì do
tích của chúng bằng 1 và các số này đều dương, phải có ít nhất một số lớn hơn
1 và ít nhất một số nhỏ hơn 1. Không giảm tính tổng quát, giả sử xk < 1 và
xk+1 > 1. Khi đó
1 − xk > 0, xk+1 − 1 > 0 nên(1 − xk) (xk+1 − 1) > 0
Bởi vậy
xk + xk+1 > xk.xk+1 + 1 (1.6)
Từ đẳng thức: x1.x2. · · · xk−1 (xkxk+1) = x1.x2. · · · .xk−1xk.xk+1 = 1 suy ra k số
dương x1, x2, · · · , xk−1, (xkxk+1) có tích bằng 1, nên theo giả thiết quy nạp, có
x1 + x2 + · · · + xkxk+1 ≥ k
Cộng cả hai vế của bất đẳng thức trên với 1 ta có bất đẳng thức
x1 + x2 + · · · + xkxk+1 + 1 ≥ k + 1 (1.7)
Từ bất đẳng thức (1.6) và (1.7) ta có
x1 + x2 + · · · + xk + xk+1 ≥ x1 + x2 + · · · + xkxk+1 + 1 ≥ k + 1
Khẳng định đã được chứng minh.
Với n số dương tùy ý x1 = x2 = · · · = xn và x1.x2. · · · xn = 1 suy ra xi = 1(1 ≤ i ≤
n), nên x1 + x2 + · · · + xn = n.
Ví dụ 1.2.7. (Tìm chữ số tận cùng bằng quy nạp)
Với mọi số nguyên dương k ≥ 2 hãy tìm chữ số tận cùng của số Ak = 22k
+ 1.
8
Chương 1. Phương pháp quy nạp
Lời giải: Bài toán được giải quyết bằng quy nạp.
1) Cơ sở quy nạp. Với k = 2 số A2 = 222
+ 1 = 24 + 1 = 17
2) Quy nạp. Giả sử với k = n ≥ 2 số An đã có tận cùng là 7.
Xét số An+1 = 22n+1
+ 1.
Do An tận cùng số 7, nên tồn tại số nguyên dương m, để An = 10m + 7. Từ đó
An − 1 = 22n
= 10m + 6.
An+1 = 22n+1
+ 1 = 22n.2
+ 1 = 22n 2
+ 1
= (10m + 6)2
+ 1 = 100m2
+ 120m + 36 + 1
= 10 10m2
+ 12m + 37 = 10 10m2
+ 12m + 3 + 7
nên An+1 tận cùng bằng chữ số 7.
Vậy với mọi số k ≥ 2, số Ak tận cùng bằng chữ số 7.
Ví dụ 1.2.8. Tính tổng sau
S(n) = 12
+ 32
+ · · · + (2n + 1)2
,
trong đó n là một số tự nhiên.
Lời giải: Ta sẽ đi dự đoán công thức tổng S(n). Ta thấy S(n) là tổng của các
lũy thừa bậc hai của các số 1, 3, · · · , (2n + 1), nên ta dự đoán S(n) phải là một
đa thức của n có bậc không nhỏ hơn ba. Giả sử S(n) = an3 + bn2 + cn + d. Vì
S(0) = 1 nên d = 1. Lần lượt thay n = 1, n = 2, n = 3, ta được hệ



a + b + c = 9
8a + 4b + 2c = 34
27a + 9b + 3c = 83
Giải hệ này ta thu được a = 4
3, b = 4, c = 11
3 . Khi đó,
S(n) =
4
3
n3
+ 4n2
+
11
3
n + 1 =
(n + 1) (2n + 1) (2n + 3)
3
(1.8)
Ta sẽ chứng minh công thức (1.8) bằng quy nạp theo n.
1) Cơ sở quy nạp. Với n = 0, 1, 2, 3, 4, 5, ta dễ dàng kiểm tra được (1.8) đúng.
2) Quy nạp. Giả sử (1.8) đúng với n = k, ta chứng minh (1.8) đúng với
n = k + 1.
Thật vậy
S(k + 1) = 12
+ 32
+ · · · + (2k + 1)2
+ (2k + 3)2
= S(k) + (2k + 3)2
.
9
Chương 1. Phương pháp quy nạp
Theo giả thiết quy nạp thì S(k) =
(k+1)(2k+1)(2k+3)
3 .
Suy ra
S(k + 1) =
(k + 1) (2k + 1) (2k + 3)
3
+ (2k + 3)2
=
(k + 2) (2k + 3) (2k + 5)
3
Vậy công thức (1.8) đúng với n = k + 1, do đó theo nguyên lý quy nạp (1.8) đúng
với mọi n.
Ví dụ 1.2.9. Chứng minh rằng A(n) = 7n + 3n − 1 chia hết cho 9 với mọi số tự
nhiên n.
Lời giải: Đặt A(n) = 7n + 3n − 1. Ta sẽ chứng minh bài toán bằng quy nạp.
1) Cơ sở quy nạp. Với n = 0, ta có A(0) = 0 chia hết cho 9.
2) Quy nạp. Giả sử A(k) chia hết cho 9 với mọi k ∈ N. Ta sẽ chứng minh
A(k + 1) cũng chia hết cho 9.
Thật vậy, ta có
A(k + 1) = 7k+1
+ 3 (k + 1) − 1
= 7A(k) − 9. (2k − 1) .
(1.9)
Theo giả thiết quy nạp thì A(k) chia hết cho 9. Do đó từ (1.9) ta suy ra A(k + 1)
cũng chia hết cho 9.
Vậy A(n) chia hết cho 9 với mọi số tự nhiên n.
Ví dụ 1.2.10. Gọi S(n) là tổng tất cả các ước số lẻ lớn nhất của các số tự nhiên
từ 1 đến 2n. Chứng minh rằng 3S(n) = 4n + 2.
Lời giải: Các số tự nhiên từ 1 đến 2n bao gồm các số lẻ từ 1 đến 2n và gấp đôi
của các số tự nhiên từ 1 đến 2n−1. Từ đó ta có
S(n) = S(n − 1) + 1 + 3 + · · · + (2n
− 1)
hay
S(n) = S(n − 1) + 4n−1
(1.10)
Ta sẽ chứng minh bài toán đã cho bằng quy nạp theo n.
1) Cơ sở quy nạp. Với n = 1, ta có 3S(1) = 3(1 + 1) = 6 = 41 + 2, khẳng định
đúng với n = 1
10
Chương 1. Phương pháp quy nạp
2) Quy nạp. Giả sử khẳng định đúng với n = k, tức là
3S(k) = 4k
+ 2 (1.11)
Ta cần chỉ ra khẳng định đúng với n = k + 1 hay 3S(k + 1) = 4k+1 + 2.
Thật vậy, theo (1.10) ta có
S(k + 1) = S(k) + 4k
⇔ 3S(k + 1) = 3S(k) + 3.4k
= 4k
+ 2 + 3.4k
= 4k+1
+ 2.
(1.12)
theo giả thiết quy nạp. Vậy khẳng định đã cho đúng với mọi n ∈ N∗.
Ví dụ 1.2.11. Tìm bậc cao nhất k của 2007 sao cho 2007k là ước của số:
200820072006
+ 200620072008
.
Lời giải: Ta chứng minh hai bổ đề sau:
Bổ đề 1.2.1. Với số tự nhiên lẻ a ≥ 3 và với mọi n nguyên dương không chia
hết cho a, ta có:
(1 + a)an
= 1 + Snan+1
,
trong đó Sn là số nguyên dương và không chia hết cho a.
Bổ đề 1.2.2. Với số tự nhiên lẻ b ≥ 3 và với mọi số n nguyên dương, ta có:
(b − 1)bn
= −1 + tnbn+1
,
trong đó tn là số nguyên không chia hết cho b.
+, Chứng minh bổ đề 1: Ta sẽ chứng minh bổ đề 1 bằng quy nạp.
1. Cơ sơ quy nạp. Với n = 1, ta có
(1 + a)a
= 1 + C1
aa + C2
aa2
+ . . . + Ca
a aa
=
= 1 + a2
1 + C2
a + C3
aa + . . . + aa−2
= 1 + S1a2
.
Vì a lẻ nên C2
a = a!
2!(a−2)! =
a(a−1)
2
...a.
Do vậy, S1 = 1 + C2
a + C3
aa + . . . + aa−2 không chia hết cho a.
11
Chương 1. Phương pháp quy nạp
2. Quy nạp. Giả sử khẳng định trên đúng với n = k, tức là
(1 + a)ak
= 1 + Skak+1
,
trong đó Sk là số nguyên dương và không chia hết cho a.
Ta có
(1 + a)ak+1
= (1 + a)ak
.a
= 1 + Skak+1 a
= 1 + C1
aSkak+1
+ C2
a Skak+1 2
+ . . . + Ca
a Skak+1 a
= 1 + ak+2
Sk + C2
aS2
k + . . . + Sa
kaak+a−k−2
= 1 + Sk+1ak+2
.
Vì Sk không chia hết cho a suy ra Sk+1 không chia hết cho a.
Vậy bổ đề được chứng minh.
+, Chứng minh bổ đề 2. Bổ đề 2 được chứng minh bằng phương pháp quy
nạp bằng cách lý luận tương tự như chứng minh bổ đề 1.
Áp dụng hai bổ đề trên với a = b = 2007, ta được
200820072006
+ 200620072008
= S200620072007
+ t200820072009
= S2006 + t200820072
20072007
.
Vì S2006 không chia hết cho 2007 nên số k lớn nhất thỏa mãn điều kiện bài toán
là 2007.
Ví dụ 1.2.12. Dãy số (un) xác định như sau:
u0 = u1 = 1
un+1 = un−1un + 1; n = 1, 2, . . .
Chứng minh rằng: u2008 − 3
...4.
Lời giải: Từ cách xác định dãy số, ta có u2 = 2; u3 = 3; u4 = 7; u5 = 22; u6 =
155; . . . Nhận thấy:
u2 ≡ 2 (mod 4); u5 ≡ 2 (mod 4).
u3 ≡ 3 (mod 4); u4 ≡ 3 (mod 4).
Ta dự đoán:
un ≡ 2 (mod 4) nếu n = 3k + 2, (k ∈ Z)
un ≡ 3 (mod 4) nếu n = 3k + 1 hoặc n = 3k, (k ∈ Z).
(1.13)
12
Chương 1. Phương pháp quy nạp
Ta sẽ chứng minh quan hệ (1.13) bằng quy nạp.
1. Cơ sở quy nạp. Với n = 2, 3, 4, 5, ta có quan hệ (1.13) đúng.
2. Quy nạp. Giả sử (1.13) đúng với n = t. Ta cần chứng minh (1.13) đúng với
n = t + 1.
+) Nếu t = 3k, khi đó
t − 1 = 3k − 1 = 3(k − 1) + 2
t + 1 = 3k + 1.
Do đó, theo giả thiết quy nạp ta có
ut ≡ 3 (mod 4); ut−1 ≡ 2 (mod 4)
Suy ra
ut+1 ≡ 3.2 + 1 ≡ 3 (mod 4),
hay (1.13) đúng.
+) Nếu t = 3k + 1, suy ra
t − 1 = 3k
t + 1 = 3k + 2 .
Do đó, theo giả thiết quy nạp ta có
ut ≡ 3 (mod 4); ut−1 ≡ 3 (mod 4)
Suy ra
ut+1 ≡ 3.3 + 1 ≡ 2 (mod 4),
hay (1.13) đúng.
+) Nếu t = 3k + 2, suy ra
t − 1 = 3k + 1
t + 1 = 3(k + 1) .
Do đó, theo giả thiết quy nạp ta có
ut ≡ 2 (mod 4); ut−1 ≡ 3 (mod 4)
Suy ra
ut+1 ≡ 3.2 + 1 ≡ 3 (mod 4),
hay (1.13) đúng. Vậy (1.13) được chứng minh. Do 2008 = 3.669 + 1, nên u2008 ≡ 3
(mod 4), hay u2008 − 3
...4.
13
Chương 1. Phương pháp quy nạp
Ví dụ 1.2.13. Chứng minh rằng số được thành lập bởi 3n chữ số giống nhau thì
chia hết cho 3n, trong đó n là một số nguyên dương cho trước.
Lời giải: Gọi A(n) = aa . . . a, trong đó A(n) gồm 3n chữ số a và 1 ≤ a ≤ 9. Ta
chứng minh khẳng định bằng quy nạp theo n.
1. Cơ sở quy nạp. Với n = 1, ta có A(1) = aaa chia hết cho 31 = 3. Khẳng
định đúng với n = 1.
2. Quy nạp. Giả sử A(n) chia hết cho 3n, ta cần chứng minh A(n + 1) chia
hết cho 3n+1.
Thật vậy, ta có
A(n + 1) = A(n)A(n)A(n) = A.102.3n
+ A.103n
+ A
= A 102.3n
+ 103n
+ 1 .
Vì 10 ≡ 1 (mod 3) nên 102.3n
+ 103n
+ 1 ≡ 0 (mod 3).
Hơn nữa, theo giả thiết quy nạp, A(n) chia hết 3n.
Do đó, ta có A(n + 1) chia hết 3n.3 = 3n+1, hay khẳng định đúng với n + 1.
Theo nguyên lý quy nạp, khẳng định đã cho đúng với mọi n ∈ Z+.
Ví dụ 1.2.14. Mỗi đầu của đường kính thuộc đường tròn tâm 0 ghi số 1. Sau
đó tại trung điểm của mỗi cung nhận được ghi số 2 (tổng của hai số được ghi ở
hai đầu của mỗi cung) (Bước 2). Coi bốn điểm ghi số là các điểm chia đường
tròn. Khi đó đường tròn được chia thành bốn cung bằng nhau. Giữa mỗi cung
này lại ghi số 3 (tổng của hai số được ghi ở hai đầu của cung tương ứng) (Bước
3). Cứ tiếp tục như vậy. Hỏi sau n bước tổng các số được ghi trên đường tròn là
bao nhiêu?
0
1 1
2
2
3
3
3
3
14
Chương 1. Phương pháp quy nạp
Lời giải: Sau n bước tổng các số trên đường tròn là Sn = 2.3n.
Ta sẽ chứng minh công thức trên bằng quy nạp theo n.
1. Cơ sở quy nạp. Sau Bước 1, trên đường tròn có bốn (22) số 1, 2, 1, 2. Khi
đó
S1 = 1 + 2 + 1 + 2 = 6 = 2.31
2. Quy nạp. Giả sử khẳng định đúng với n = k(k ≥ 1), nghĩa là sau k bước
trên đường tròn đã có 2k+1 số
s1, s2, . . . , s2k+1, . . . , s2k+1 , (1.14)
với tổng là Sk = 2.3k.
1 1
2
2
3
3
3
3
Sang bước (k + 1), ta coi 2k+1 điểm đã ghi là các điểm chia, nên đường tròn
được thành 2k+1 cung bằng nhau. Do trung điểm của mỗi cung này lại ghi tổng
của hai số đã ghi ở đầu của mỗi cung, nên mỗi số thuộc dãy (1.14) được xuất
hiện đúng hai lần trong các tổng mới (các số được ghi tại bước k + 1). Do đó,
tổng các số được ghi trên đường tròn sau k + 1 bước là:
Sk+1 = tổng các số đã ghi sau bước k + tổng các số được ghi tại bước k+1
= Sk + 2Sk = 3Sk = 3.2.3k
= 2.3k+1
Khẳng định được chứng minh.
Ví dụ 1.2.15. Chứng minh rằng trên mặt phẳng n đường thẳng khác nhau cùng
đi qua một điểm, chia mặt phẳng thành 2n phần khác nhau.
Lời giải: Bài toán được giải quyết bằng quy nạp.
1) Cơ sở quy nạp. Với n = 1, ta có một đường thẳng. Nó chia mặt phẳng
thành hai phần, nên khẳng định đúng.
15
Chương 1. Phương pháp quy nạp
2) Quy nạp. Giả sử với n = k khẳng định đã đúng, nghĩa là k đường thẳng
tùy ý cùng đi qua một điểm M đã chia mặt phẳng thành 2k phần khác nhau.
Xét n = k + 1 đường thẳng khác nhau tùy ý cùng đi qua một điểm. Kí
hiệu các đường này, một cách tương ứng bằng δ1, δ2, · · · , δk, δk+1. Theo giả thiết
quy nạp k đường thẳng δ1, δ2, · · · , δk đã chia mặt phẳng thành 2k phần khác nhau.
M
δs
δt
δk+1
Vì các đường thẳng đều khác nhau và cùng đi qua điểm M, nên tồn tại các
chỉ số s, t (1 ≤ s, t ≤ k) để δk+1 là đường thẳng duy nhất nằm trong góc được
lập nên bởi δs và δt. Khi đó δk+1 chia hai phần mặt phẳng được giới hạn bởi
δs và δt thành bốn phần. Bởi vậy k + 1 đường thẳng δ1, δ2, · · · , δk, δk+1 chia mặt
phẳng thành
2k − 2 + 4 = 2k + 2 = 2 (k + 1)
phần khác nhau. Khẳng định được chứng minh.
16
Chương 2
Phương pháp chứng minh phản
chứng
Chứng minh là một nét đặc trưng của toán học, tạo ra sự khác biệt giữa
toán học với các môn khoa học khác. Nắm bắt phương pháp và kĩ thuật chứng
minh cũng là yêu cầu bắt buộc đối với học sinh nói chung. Các phương pháp và
kĩ thuật chứng minh rất phong phú: Từ chứng minh trực tiếp đến gián tiếp, từ
chứng minh bằng quy nạp đến chứng minh bằng phản chứng, từ ví dụ đến phản
ví dụ, từ xây dựng đến không xây dựng.
Trong bài luận văn này xin được đề cập đến phép chứng minh phản chứng,
một trong những phương pháp chứng minh kinh điển và quan trọng nhất của
toán học. Chứng minh phản chứng có thể nói là một trong những vũ khí quan
trọng nhất của toán học. Nó cho phép chúng ta chứng minh sự có thể và không
thể của một tính chất nào đó. Nó cho phép chúng ta biến thuận thành đảo, biến
đảo thành thuận. Nó cho phép chúng ta lý luận trên những đối tượng mà không
rõ là có tồn tại hay không.
2.1 Cơ sở lý thuyết
Cơ sở lý thuyết của phương pháp phản chứng là các định luật trong logic:
Gọi p, q, r là các mệnh đề toán học nào đó, khi đó
Định lý 2.1.1. (Các định luật cơ bản)
1. (Phi mâu thuẫn): p ∧ p = 0
2. (Bài trung):p ∨ p = 1
Định lý 2.1.2. (Các định luật phản chứng)
1. (Phản chứng) (p ⇒ q) ⇔ (q ⇒ p).
17
Chương 2. Phương pháp chứng minh phản chứng
2. (Phản chứng suy rộng):
(p ∧ q ⇒ r) ⇔ (p ∧ r ⇒ q) (2.1)
3. (p ⇒ q ∧ q) ⇒ p.
4. (q ⇒ 0) ⇒ q.
Các định luật trên có thể được chứng minh chẳng hạn bằng phương pháp
lập bảng chân lý hoặc phương pháp biến đổi tương đương. Chẳng hạn, ta có thể
chứng minh (2.1) như sau:
(p ∧ q ⇒ r) ⇔ p ∧ q hoặc r (do a ⇒ b ⇔ a hoặc b)
⇔p hoặc q hoặc r (do a ∧ b ⇔ a hoặc b)
⇔(p hoặc r) hoặc q (giao hoán, kết hợp và r ⇔ r)
⇔p ∧ r hoặc q
⇔(p ∧ r ⇒ q) đpcm.
2.2 Nội dung của phương pháp phản chứng
Để chứng minh khẳng định p ⇒ q bằng phương pháp phản chứng ta giả sử q
sai, tức là q là mệnh đề đúng. Nếu từ đó thu được một điều vô lý (vl) thì điều
đó chứng tỏ giả sử của ta là sai, tức là q đúng. Điều vô lý (vl) có thể thuộc một
trong các dạng sau:
+) Điều trái với giả thiết p (vl ≡ p)
+) Điều trái với một trong các kiến thức đã biết (vl ≡ 0)
+) Điều trái với giả sử phản chứng (vl ≡ q).
Khi xây dựng mệnh đề phản chứng q ta cần nhớ
1. p ⇔ p.
2. p ∧ q ⇔ p hoặc q.
3. p ∨ q ⇔ p ∧ q.
4. ∀x, p(x) ⇔ ∃x, p(x).
5. ∃x, p(x) ⇔ ∀x, p(x).
6. ∀x, p(x) ⇔ ∃x, p(x).
7. ∃x, p(x) ⇔ ∀x, p(x).
8. Nếu p(x) := f(x)Rg(x), thì p(x) là : f(x)Rg(x) trong đó, R và R được xác
định như sau:
18
Chương 2. Phương pháp chứng minh phản chứng
R
R
=
==
= >
>
≥
≥
<
<
≤
≤
Ngoài ra còn có:
Định lý 2.2.1. (Định luật về âm bản). Cho A là một công thức logic mà trong
đó chỉ chứa các phép toán: Phủ định, ∧, hoặc đối với các mệnh đề: p1, p2, · · · , pn.
Âm bản của A, kí hiệu là Ad là một công thức thu được từ A bằng cách thay
pj ∼ pj; pj ∼ pj, ∨ ∼ ∧; ∧ ∼ ∨. Khi đó:
A ⇔ Ad
Phương pháp phản chứng còn dựa trên nguyên lý Dirichlet do nhà toán học
Đức nổi tiếng Peter Dirichlet (1805-1859) đề xuất, mà dạng đơn giản nhất của
nguyên lý này được phát biểu như sau: "Không thể nhốt 7 chú thỏ vào 3 cái
lồng sao cho mỗi lồng không có quá hai chú thỏ". Nói cách khác "Nếu nhốt 7
chú thỏ vào 3 cái lồng, thì phải có ít nhất một lồng có không ít hơn 3 chú thỏ".
2.3 Trình bày lời giải của phương pháp phản chứng
Bài toán: Chứng minh p ⇒ q
Lời giải: Giả sử ngược lại, q sai, tức là q. Mà q ⇒ · · · ⇒ vl. Vậy giả sử của ta là
sai, tức là q đúng.
Ngoài ra, ta sẽ giải một số bài toán bằng phương pháp phản chứng dựa trên
nguyên lý Dirichlet.
2.4 Một số ví dụ minh họa
Ví dụ 2.4.1. Cho f(x) = ax2 + bx + c. Giả sử
|a| + |b| + |c| > 17 (2.2)
Chứng minh rằng
∃x ∈ [0; 1], |f(x)| > 1 (2.3)
Lời giải: Ta sẽ chứng minh bằng phương pháp phản chứng. Giả sử (2.3) sai, tức
là
∀x ∈ [0; 1], |f(x)| ≤ 1 (2.4)
19
Chương 2. Phương pháp chứng minh phản chứng
Chọn x = 0; 1
2; 1, từ (2.4) ta được |c| ≤ 1 và:
|a + b + c| ≤ 1
a
4 + b
2 + c ≤ 1
Suy ra
|a| + |b| + |c| ≤ 17
Đó là điều vô lý (trái với (2.2)). Vậy giả sử của ta là sai, tức là (2.3) đúng.
Ví dụ 2.4.2. Chứng minh tập các số nguyên tố P là tập vô hạn.
Lời giải: (phản chứng). Giả sử ngược lại, tập P là hữu hạn. Giả sử P =
{p1, p2, · · · , pn}. Khi đó, tồn tại số nguyên tố lớn nhất. Gọi đó là pn. Tức là:
pn ∈ P và ∀p ∈ P, p ≤ pn.
Xét số x = p1.p2. . . . .pn +1. Ta có, x ∈ N; x không chia hết cho các số p1; p2; . . . ; pn
(vì nếu x
...pj nào đó thì 1
...pj: vô lý). Vậy x ∈ P. Mà hiển nhiên, x > pn. Đó là điều
vô lý (trái với cách chọn pn).
Vậy điều giả sử của ta là sai, tức là P là tập vô hạn.
Ví dụ 2.4.3. Có thể chia các số tự nhiên từ 1 đến 21 thành các nhóm đôi một rời
nhau, sao cho trong mỗi nhóm số lớn nhất bằng tổng các số còn lại hay không?
Lời giải:
Giả sử chia được. Khi đó tổng các số ở mỗi nhóm là một số chẵn (bằng hai
lần số lớn nhất). Vậy tổng của 21 số đã cho là một số chẵn (vì các nhóm đôi
một rời nhau và tổng của các số chẵn là số chẵn).
Nhưng tổng của 21 số đó là 21.11 = 231 là số lẻ. Điều vô lý này chứng tỏ giả
sử của ta là sai, tức là không chia được thành các nhóm thỏa mãn yêu cầu đề
bài.
Ví dụ 2.4.4. Có thể tìm được hay không 5 số nguyên, sao cho các tổng của hai
số một trong 5 số đó lập thành 10 số nguyên liên tiếp?
Lời giải:
Giả sử tìm được 5 số như vậy. Gọi s là tổng của 5 số đó và n là giá trị nhỏ
nhất của tổng các cặp hai số. Khi đó 10 số nguyên liên tiếp nói trong đề bài là
n, n + 1, . . . , n + 9
20
Chương 2. Phương pháp chứng minh phản chứng
Ta tính tổng τ của 10 số đó theo hai cách khác nhau:
Một mặt, τ = n + (n + 1) + (n + 2) + . . . + (n + 9) = 5 (2n + 9)
Mặt khác τ = 4s (do trong τ mỗi số đã cho có mặt đúng 4 lần). Từ đó suy ra
4s = 5 (2n + 9) là điều vô lý.
Vậy giả sử ban đầu là sai, tức là không thể chọn được 5 số thỏa mãn yêu cầu
bài ra.
Ví dụ 2.4.5. Cho ba điểm A, B, C phân biệt trên mặt phẳng. Chứng minh rằng
nếu tồn tại điểm G thuộc mặt phẳng đó thỏa mãn:
−→
GA +
−−→
GB +
−→
GC =
−→
0
thì điểm G đó là duy nhất.
Lời giải: Ta chứng minh bằng phương pháp phản chứng.
Giả sử còn có điểm O = G thỏa mãn yêu cầu bài toán, tức là
−→
OA +
−−→
OB +
−→
OC =
−→
0
Ta có
−→
GA +
−−→
GB +
−→
GC =
−→
0
⇔
−→
GO +
−→
OA +
−→
GO +
−−→
OB +
−→
GO +
−→
OC =
−→
0
⇔ 3
−→
GO +
−→
OA +
−−→
OB +
−→
OC =
−→
0
Do
−→
OA +
−−→
OB +
−→
OC =
−→
0 , suy ra
3
−→
GO =
−→
0
⇔
−→
GO =
−→
0
⇔ G ≡ O
Vậy G là duy nhất.
Ví dụ 2.4.6. (IMO 1982) Cho phương trình
x3
− 3xy2
+ y3
= n, n ∈ N∗
(2.5)
1) Chứng minh rằng nếu (2.5) có nghiệm nguyên thì nó không có nghiệm
nguyên duy nhất.
2) Tìm nghiệm nguyên của (2.5) khi n = 2005.
21
Chương 2. Phương pháp chứng minh phản chứng
Lời giải:
1) Dễ dàng biến đổi
x3
− 3xy2
+ y3
= (y − x)3
− 3 (y − x) x2
+ (−x)3
= (−y)3
− 3 (−y) (x − y)2
+ (x − y)3
Từ đó ta có: Nếu (x; y) là một nghiệm của (2.5) thì (y − x; −x) , (−y; x − y) cũng
là các nghiệm của (2.5). Ba nghiệm này đôi một khác nhau vì nếu có hai nghiệm
nào đó bằng nhau thì ta có x = y = 0 là điều vô lý do n > 0.
Vậy nếu (2.5) có nghiệm, thì nó có ít nhất ba nghiệm phân biệt, tức là (2.5)
không thể có nghiệm duy nhất.
2) Với n = 2005 giả sử (2.5) có nghiệm (x; y) ∈ Z2. Ta viết các nghiệm theo
(mod 3), phương trình đã cho trở thành
x3
+ y3
= −1 (mod 3) ⇒ x + y = −1 (mod 3).
Do vậy ta có các trường hợp sau:
(i)x ≡ 0 (mod 3) và y ≡ −1 (mod 3); (2.6)
(ii)x ≡ 1 (mod 3) và y ≡ 1 (mod 3); (2.7)
(iii)x ≡ −1 (mod 3) và y ≡ 0 (mod 3). (2.8)
Trong trường hợp (i) đặt x = 3m, y = 3n − 1, thay vào phương trình đã cho ta
được V T ≡ −1 (mod 3) còn V P = 2005 ≡ −2 (mod 9) ⇒ vô lý.
Trong trường hợp (ii), do (y − x; −x) cũng là nghiệm, mà y − x ≡ 0 (mod 3) và
−x ≡ −1 (mod 3) nên ta cũng thu được một điều vô lý.
Tương tự từ trường hợp (iii) ta cũng thu được một điều vô lý.
Vậy với n = 2005 phương trình đã cho không có nghiệm nguyên.
Ví dụ 2.4.7. (IMO 1983). Tìm tất cả những hàm số f(x) : R+ → R+, là toàn
ánh và thỏa mãn đồng thời hai điều kiện sau:
1) f(xf(y)) = yf(x), ∀x, y ∈ R+.
2) f(x) → 0 khi x → +∞.
Lời giải: Do f(x) là toàn ánh và 1 ∈ R+ nên ∃y0 ∈ R+ : f(y0) = 1. Trong 1) cho
x = 1, y = y0 ta được
f(1) = f(1.1) = f(1.f(y0)) = y0f(1) → y0 = 1(do f(1) > 0) ⇒ y0 = 1 ⇒ f(1) = 1.
22
Chương 2. Phương pháp chứng minh phản chứng
Vậy f(x) có một điểm bất động là x = 1. Ta sẽ chứng minh đó là điểm bất động
duy nhất.
Thật vậy, giả sử f(x) có hai điểm bất động x, y phân biệt, khi đó ta có:
(i) f(xy) = f(xf(y)) = yf(x) = yx = xy → xy cũng là điểm bất động của
f(x).
(ii) 1 = f(1) = f 1
xf(x) = xf(1
x) → 1
x cũng là điểm bất động của f(x).
Giả sử còn có điểm bất động x = 1, khi đó theo (ii), f(x) còn có điểm bất động
1
x . Trong hai số đó phải có một số lớn hơn 1. Giả sử x > 1. Theo (i), f(x) sẽ có
vô số điểm bất động xn = xn, n ∈ N∗. Ta có dãy đối số (xn) → +∞ (do x > 1), do
đó dãy hàm (f(xn)) = (xn) → +∞. Điều đó trái với 2) : f(x) → 0 khi x → +∞.
Vậy giả sử của ta là sai, tức là f(x) có duy nhất một điểm bất động là x = 1.
Nhưng trong (i) cho y = x ta được f(xf(x)) = xf(x), ∀x ∈ R+ ⇒ xf(x) cũng là
điểm bất động của f(x). Vậy ta phải có
∀x ∈ R+
: xf(x) = 1 ⇔ f(x) =
1
x
.
Dễ tìm thấy hàm số này thỏa mãn các điều kiện bài ra.
Vậy f(x) = 1
x là hàm số cần tìm.
Ví dụ 2.4.8. (IMO 1983) Cho các số a, b, c ∈ N∗, đôi một nguyên tố cùng nhau.
Chứng minh rằng phương trình
xbc + yca + zab = 2abc − ab − bc − ca (2.9)
không có nghiệm tự nhiên.
Lời giải: Giả sử (2.9) có nghiệm (x; y; z) ∈ N3. Từ (2.9) ta có (z + 1) ab
...c. Mà
(a; c) = (b; c) = 1 ⇒ z + 1
...c, mà z + 1 > 0 nên từ đó có z + 1 ≥ c. Tương tự, có
x + 1 ≥ a, y + 1 ≥ b. Từ các đánh giá này ta được
V T(2.9) ≥ (a − 1) bc + (b − 1) ca + (c − 1) ab
= 3abc − ab − bc − ca
> 2abc − ab − bc − ca = V P(2.9)
Điều này vô lý chứng tỏ giả sử của ta là sai, tức là phương trình (2.9) không có
nghiệm tự nhiên.
Ví dụ 2.4.9. (Putnam 1998) Chứng minh rằng với mọi số nguyên a, b, c luôn
tìm được số nguyên dương n, sao cho số
f(n) = n3
+ an2
+ bn + c
23
Chương 2. Phương pháp chứng minh phản chứng
không phải là số chính phương.
Lời giải: Ta chứng minh mệnh đề sau:
∀ (a; b; c) ∈ Z3
, ∃n ∈ N∗
: f(n) không phải là số chính phương. (2.10)
Nhận xét rằng mọi số chính phương đều ≡ 0 (mod 4) hoặc ≡ 1 (mod 4). Giả sử
(2.10) là mệnh đề sai, tức là
∃ (a; b; c) ∈ Z3
, ∀n ∈ N∗
: f(n) là số chính phương. (2.11)
Đặc biệt, từ đó ta có



f(1) = a + b + c + 1 là số chính phương(i)
f(2) = 4a + 2b + c + 8 là số chính phương(ii)
f(3) = 9a + 3b + c + 27 là số chính phương(iii)
f(4) = 16a + 4b + c + 64 là số chính phương(iv)
Từ (ii) và (iv) ta có f(4) − f(2) = 2b (mod 4). Mà 2b là số chẵn, còn theo nhận
xét thì f(4) − f(2) chỉ có thể ≡ 0, 1, −1 (mod 4) ⇒ 2b ≡ 0 (mod 4).
Từ (i) và (iii) ta có f(3) − f(1) ≡ (2b + 2) (mod 4).
Tương tự trên ta cũng có 2b + 2 ≡ 0 (mod 4) ⇒ 2 ≡ 0 (mod 4) là điều vô lí.
Vậy giả sử của ta là sai, tức là (2.10) là mệnh đề đúng.
Ví dụ 2.4.10. Cho a, b, c là các số thực dương. Chứng minh rằng
a
√
a2 + 8bc
+
b
√
b2 + 8ca
+
c
c2 + 8ab
≥ 1.
Lời giải: Ta sẽ giải bài toán trên bằng phương pháp phản chứng như sau: Đặt
x =
a
√
a2 + 8bc
, y =
b
√
b2 + 8ca
, z =
c
c2 + 8ab
Khi đó, ta có
1
x2
− 1
1
y2
− 1
1
z2
− 1 = 64
và ta cần chứng minh
x + y + z ≥ 1
Giả sử ngược lại, x + y + z < 1. Khi đó
1
x2 − 1 1
y2 − 1 1
z2 − 1 >
(x + y + z)2
x2
− 1
(x + y + z)2
y2
− 1
(x + y + z)2
z2
− 1
=
(2x + y + z) (y + z) (2y + x + z) (x + z) (2z + x + y) (x + y)
x2y2z2
(2.12)
24
Chương 2. Phương pháp chứng minh phản chứng
Áp dụng bất đẳng thức Cauchy cho các dấu ngoặc trong V P(2.12) ta được:
2x + y + z = x + x + y + z ≥ 4 4
√
x.x.y.z
2y + x + z = y + y + x + z ≥ 4 4
√
y.y.z.z
2z + x + y = z + z + x + y ≥ 4 4
√
z.z.x.y
y + z ≥ 2
√
y.z
x + z ≥ 2
√
x.z
x + y ≥ 2
√
x.y
Thay vào biểu thức (2.12) ta được VP(2.12) > 192, mà theo giả thiết VT(2.12) =
64, do đó ta có (64 > 192) (mâu thuẫn).
Vậy điều giả sử của ta là sai. Tức là bài toán được chứng minh.
Ví dụ 2.4.11. Cho a, b, c là các số thực sao cho a+b+c > 0, ab+bc+ca > 0, abc > 0.
Chứng minh rằng a, b, c > 0.
Lời giải: (phản chứng). Giả sử một trong các số a, b, c < 0. Vì ta có abc > 0 nên
phải có hai số âm, một số dương. Không mất tính tổng quát, giả sử a, b < 0, c > 0.
Khi đó
ab + bc + ca = a(a + b + c) − a2
+ bc < 0
Mâu thuẫn với điều kiện ban đầu. Do đó giả sử của ta là sai.
Vậy ta có điều phải chứng minh.
Ví dụ 2.4.12. Chứng minh rằng với mỗi số nguyên dương k tồn tại một số
nguyên dương n > 1, sao cho: Ci
n
...k(∀i = 1, n − 1).
Lời giải: (phản chứng). Giả sử Ci
n
...k. Do đó, lấy k = 4, khi đó tồn tại một số
nguyên dương n > 1 để Ci
n
...4 (∀i = 1, n).
Suy ra
n−1
i=1
= Ci
n ≡ 0 (mod 4).
Mặt khác
n−1
i=1
= Ci
n = 2n
− 2 ≡ −2 (mod 4).
Do đó, ta có mâu thuẫn.
Vậy điều giả sử của ta là sai. Ta được điều phải chứng minh.
25
Chương 2. Phương pháp chứng minh phản chứng
Ví dụ 2.4.13. Xét các số gồm 7 chữ số phân biệt được lập thành từ các số
{1, 2, . . . , 7} .
Liệu trong các số được lập có ba số a, b, c mà a + b = c.
Hỏi có hai số khác nhau a, b mà a
...b.
Lời giải: 1. Ta có:
1 + 2 + . . . + 7 = 28 ≡ 1 (mod 9).
Vậy số bất kỳ trong các số nói trên khi chia cho 9 đều dư 1.
Giả sử tồn tại ba số a, b, c mà a + b = c. Khi đó: a + b ≡ 2 (mod 9), còn c ≡ 1
(mod 9).
Điều này vô lý, nên không tồn tại ba số thỏa mãn yêu cầu bài toán.
2. Giả sử có hai số a, b mà a
...b, tức là ∃n ∈ N∗ : a = nb.
Do b.7 ≥ 1234567.7 > 7654321 ≥ a, nên 7b ≥ nb ⇔ n ≤ 7.
Vì a = nb, nên a ≡ n (mod 9)(1 < n ≤ 7).
Mặt khác, a ≡ 1 (mod 9). Điều này vô lý, nên trong các số được lập không có
hai số nào, mà một trong hai số chia hết cho số kia.
Ví dụ 2.4.14. Cho hai đường thẳng chéo nhau. Chứng minh rằng có duy nhất
một mặt phẳng chứa đường thẳng này và song song với đường thẳng kia.
Lời giải: Giả sử ta có hai đường thẳng chéo nhau a và b.
lấy điểm M bất kỳ thuộc a. Qua M kẻ đường thẳng b′ song song với b. Gọi (α)
là mặt phẳng xác định bởi a và b′.
a
b
M
b′
α
Ta có: b//b′ và b′ ⊂ (α), từ đó suy ra b//(α).
Hơn nữa (α) ⊃ a nên (α) là mặt phẳng cần tìm.
Ta chứng minh (α) là mặt phẳng duy nhất. Thật vậy, giả sử có một mặt phẳng
(β) khác (α), chứa a và song song với b thì khi đó (α), (β) là hai mặt phẳng phân
biệt cùng song song với b nên giao tuyến của chúng là a, phải song song với b.
26
Chương 2. Phương pháp chứng minh phản chứng
Điều này mâu thuẫn với giả thiết a và b chéo nhau.
Tương tự, ta có thể chứng minh có duy nhất một mặt phẳng chứa b và song
song với a.
Vậy ta được điều phải chứng minh.
Ví dụ 2.4.15. Chứng minh rằng nếu mặt phẳng (α) chứa hai đường thẳng cắt
nhau a, b và a, b cùng song song với mặt phẳng (β) thì (α) song song với (β)
Lời giải: Gọi M là giao điểm của a và b. Vì (α) chứa a mà a song song với (β)
nên (α) và (β) là hai mặt phẳng phân biệt. Ta cần chứng minh (α) song song
với (β).
Giả sử (α) và (β) không song song và cắt nhau theo giao tuyến c.
M
a
b
α
β
c
Ta có 


a//(β)
(α) ⊃ a
(α) ∩ (β) = c
⇒ c//a
và 


b//(β)
(α) ⊃ b
(α) ∩ (β) = c
⇒ c//b
.
Như vậy từ M kẻ được hai đường thẳng a, b cùng song song với c. Theo định
lý 1, §2 (SGK hình học 11), điều này mâu thuẫn.
Vậy (α) và (β) phải song song với nhau.
27
Chương 3
Phương pháp suy luận trực tiếp
3.1 Vài nét về phương pháp suy luận trực tiếp
Các bài toán logic hiểu theo nghĩa tương đối rộng gồm các bài toán logic, các
bài toán không mẫu mực (không có cách giải nhất định) thường có nhiều cách
giải khác nhau, trong đó có phương pháp suy luận trực tiếp.
Phương pháp suy luận logic đã có từ xa xưa và để giải các bài toán logic
người ta chỉ có duy nhất phương pháp này (sau này mới có thêm các phương
pháp khác). Các bài toán logic đa dạng về đề tài, phong phú về chủng loại đòi
hỏi chúng ta phải biết suy luận đúng đắn, chặt chẽ trên cơ sở vận dụng kiến
thức cơ bản và kinh nghiệm sống của mình. Vì vậy cần phải luyện tập óc quan
sát, cách lập luận, cách xem xét khả năng có thể xảy ra của một sự kiện và vận
dụng những kiến thức đã học vào các tình huống muôn hình muôn vẻ trong cuộc
sống hàng ngày.
Đôi khi để giải những bài toán này, chỉ cần những kiến thức toán học đơn
giản, nhưng lại đòi hỏi khả năng chọn lọc trường hợp, suy luận chặt chẽ, rõ ràng.
Sự phát triển của toán học, chẳng hạn giải tích tổ hợp, phương pháp quy nạp,
phản chứng, góp phần phong phú thêm phương pháp suy luận logic trực tiếp.
Những bài toán trong chương này là những ví dụ không quá phức tạp giúp
cho người đọc thấy được những kĩ năng cơ bản của tư duy logic. Thông qua lời
giải giúp người đọc biết cách diễn đạt bằng ngôn ngữ logic những bài toán cùng
dạng có thể gặp trong đời sống thực tiễn và bằng cách đó người đọc tìm ra lời
giải dễ dàng nhanh gọn hơn. Mỗi bài toán là một ví dụ điển hình và có thể có
nhiều bài toán khác tương tự được diễn đạt dưới nhiều hình thức khác nhau.
Những bài toán giải bằng phương pháp suy luận trực tiếp rất có ích cho người
học toán, dạy toán và những người quan tâm đến logic toán. Nó là bước tiếp
28
Chương 3. Phương pháp suy luận trực tiếp
cận đầu tiên đến logic toán học. Nó rèn luyện tư duy logic, khả năng phản xạ,
trí thông minh, là hình thức thể thao trí tuệ phục vụ cho đông đảo người đọc
đặc biệt là lứa tuổi học sinh ở nhiều cấp học khác nhau.
Điều cơ bản của phương pháp này là thông qua việc phân tích các điều kiện
của bài toán, cần tìm ra mối quan hệ logic giữa các mệnh đề.
3.2 Các ví dụ về vận dụng phương pháp suy luận
trực tiếp
Ví dụ 3.2.1. Có ba em chơi trò đội mũ. Em chủ trì giơ ba mũ đỏ, hai mũ xanh,
rồi yêu cầu ba em ngồi theo hàng dọc và không nhìn về phía sau, rồi từ phía sau
chụp lên đầu mỗi em một cái mũ còn hai cái cất đi. Chứng minh rằng trong mọi
cách đội đều có một em nhận ra màu mũ của mình.
Lời giải: Giả sử em A ngồi sau em B, em B ngồi sau em C, rõ ràng em A quan
sát được mũ của em B và C còn em B quan sát được mũ của em C.
- Trước hết do chỉ có hai mũ xanh nên nếu B và C đội mũ xanh thì A đoán
được ngay mũ của mình là mũ đỏ.
- Nếu A im lặng, chứng tỏ trong B và C có một em đội mũ xanh, một em
đội mũ đỏ hoặc cả hai em đội mũ đỏ. Khi đó B sẽ phải suy nghĩ và quan sát C.
Nếu C đội mũ xanh thì B đoán ngay mình đội mũ đỏ. Nếu B thấy C đội mũ đỏ
thì B đành im lặng và C sẽ nhận biết ngay mình đội mũ đỏ.
Lời giải của bài toán trên có thể tóm tắt theo sơ đồ sau
A
Đ
im
B
X
C
X
XĐ
Hình 3.1
Như vậy, trong mọi trường hợp, đều có ít nhất một em đội mũ đỏ. Dấu hiệu
để các em dự đoán được mũ của mình là số mũ xanh mà mình hoặc bạn mình
quan sát được. Em đoán đúng màu mũ của mình sẽ luôn là em đội mũ đỏ.
29
Chương 3. Phương pháp suy luận trực tiếp
Ví dụ 3.2.2. (Vị sứ giả thông minh)
Một viên quan nước Lỗ đi sang xứ Tề, bị vua Tề tuyên phạt tử hình và bị hành
quyết hoặc chém đầu hoặc treo cổ. Trước khi hành quyết nhà vua cho sứ giả được
nói một câu, nếu nói đúng thì bị chém đầu, nếu nói sai thì bị treo cổ. Sứ giả
mỉm cười nói một câu, nhờ đó đã thoát chết. Bạn hãy cho biết câu nói của sứ
giả đó như thế nào?
Lời giải: Vị sứ giả đã nói rằng "Tôi sẽ bị treo cổ". Như vậy, nếu nhà vua đem
treo cổ sứ giả thì sứ giả đó nói đúng. Mà theo điều lệnh sử phạt của nhà vua
thì phải đưa sứ giả đi chém đầu (vì ông ta nói đúng). Nếu nhà vua chém đầu
thì ông ta đã nói sai. Theo điều lệnh sử phạt của nhà vua thì phải đem sứ giả
đi treo cổ.
Thành thử nhà vua không thể hành quyết sứ giả bằng chém đầu cũng như
treo cổ, nên sứ giả đã thoát chết.
Ví dụ 3.2.3. Người bản sứ và tên thực dân
Trước vành móng ngựa là ba người đàn ông, họ là người bản sứ hoặc tên thực
dân. Quan tòa biết khi được hỏi người bản sứ bao giờ cũng nói thật, còn tên
thực dân bao giờ cũng nói dối, nhưng quan tòa không biết ai là người bản sứ, ai
là tên thực dân. Quan tòa hỏi người thứ nhất: "Anh là ai?". Nhưng anh ta nói
ngọng nên quan tòa không hiểu câu trả lời. Hãy xác định câu trả lời của người
thứ nhất?
Lời giải: Nếu người được hỏi thứ nhất là tên thực dân thì theo bản chất của
thực dân, anh ta sẽ trả lời "Tôi là người bản sứ". Nếu người đó là dân bản sứ
thì theo bản chất của người dân bản sứ anh ta cũng trả lời "Tôi là người bản
sứ".
Như vậy câu trả lời của người thứ nhất chỉ có thể là: "Tôi là người bản sứ".
Ví dụ 3.2.4. a. Trong một căn phòng có 10 người, biết rằng giữa ba người bất
kỳ có hai người quen nhau. Chứng minh rằng, có thể tìm được bốn người mà hai
người bất kỳ trong số đó đều quen nhau.
b. Khẳng định trên có còn đúng nữa không nếu ở câu a số người trong phòng là
9.
Lời giải: a. Giả sử bốn người bất kỳ có hai người không quen nhau. Khi đó
A không thể có quá ba người không quen. Nếu A có bốn người không quen
thì theo giả thiết giữa bốn người này có hai người không quen và họ cùng với
30
Chương 3. Phương pháp suy luận trực tiếp
A hợp thành bộ ba đôi một không quen nhau. Vậy A có không nhiều hơn ba
người không quen, nghĩa là A có không ít hơn sáu người quen. Giả sử A quen
với B1, B2, · · · , B6. Khi đó giữa B1, B2, · · · , B6 không có bộ ba nào đôi một quen
nhau (Nếu khác thì bộ ba này hợp thành với A thành bộ bốn đôi một quen
nhau- trái với giả thiết). Hơn nữa có bộ ba mà hai người không quen nhau trong
số sáu người B1, B2, · · · , B6. Chẳng hạn, nếu B1 không quen với B2, B3, B4 thì
B2, B3, B4 đôi một quen nhau. Vì thế B1 phải có ít nhất ba người quen trong
số B2, B3, · · · , B6. Khi đó trong số ba người này không tìm được hai người quen
nhau, ngược lại họ tạo với A và B1 thành bộ bốn người đôi một quen nhau. Trái
với giả thiết, suy ra tồn tại bộ bốn mà hai người bất kỳ quen nhau.
b. Ta chứng minh cho khẳng định trên vẫn đúng.
Nếu người nào đó quen với tất cả sáu người thì chứng minh sẽ tương tự phần
a. Nếu mỗi người quen có đúng năm người thì tổng số các cặp quen nhau sẽ là
9.5/2 không là số nguyên- điều này không thể xảy ra.
Cuối cùng nếu tìm được một người nào đó không quen với ít nhất bốn người
thì bốn người này phải đôi một quen nhau (nếu khác ta sẽ tìm được bộ ba đôi
một không quen nhau) có nghĩa là tạo thành bộ bốn cần tìm- đó là điều phải
chứng minh.
Ví dụ 3.2.5. Mỗi bạn đạt giải mấy trong kỳ thi vô địch toàn quốc?
Ba bạn Quân, Hùng, Mạnh vừa đạt giải nhất, nhì, ba trong kì thi học sinh
giỏi toàn quốc. Biết rằng:
1. Không có học sinh trường chuyên nào đạt giải cao hơn Quân.
2. Nếu Quân đạt giải thấp hơn một bạn nào đó thì Quân không phải là học
sinh trường chuyên.
3. Chỉ có duy nhất một bạn không học trường chuyên.
4. Nếu Hùng hoặc Mạnh đạt giải nhì, thì Mạnh đạt giải cao hơn bạn quê ở
Hải Phòng.
Hãy cho biết: Mỗi bạn đã đạt được giải nào? Bạn nào không học trường
chuyên và bạn nào quê ở Hải Phòng?
Lời giải: Ta nhận xét rằng: Nếu Quân đạt giải nhì hoặc ba, thì theo (2) Quân
không học trường chuyên. Ta suy ra, theo (3) Hùng và Mạnh học trường chuyên,
thành thử theo (1) Quân đạt giải nhất. Điều này vô lý. Vậy Quân phải đạt giải
nhất. Trong hai bạn Hùng và Mạnh một người đạt giải nhì và một người đạt
giải ba. Mạnh không thể đạt giải ba (vì theo (4) Mạnh còn đạt giải cao hơn bạn
quê ở Hải Phòng).
31
Chương 3. Phương pháp suy luận trực tiếp
Vậy Mạnh phải đạt giải nhì, Hùng đạt giải ba. Đồng thời ta cũng suy ra
Quân không học trường chuyên và Hùng quê ở Hải Phòng.
Ví dụ 3.2.6. Ai đã nói đùa?
Nhà trường cử thầy Nghiêm dẫn bốn học sinh Lê, Huy, Hoàng, Tiến đi thi
đấu điền kinh. Kết quả ba em đạt giải nhất, nhì, ba và một em không đạt giải.
Khi về trường mọi người hỏi kết quả các em trả lời như sau:
Lê: Mình đạt giải nhì hoặc ba.
Huy: Mình đã đạt giải.
Hoàng: Mình được giải nhất.
Tiến: Mình không được giải.
Nghe xong thầy Nghiêm mỉm cười và nói: "Chỉ có ba bạn nói thật còn một
bạn đã nói đùa "
Hãy cho biết học sinh nào đã nói đùa và ai đạt giải nhất, ai không đạt giải?
Lời giải:
1) Nếu Lê nói đùa thì ba bạn Huy, Hoàng, Tiến nói thật. Như vậy Lê và
Hoàng cùng đạt giải nhất. Điều này vô lý. Vậy Lê phải nói thật.
2) Nếu Huy nói đùa thì Huy không được giải và cả ba bạn còn lại đều nói
thật. Như vậy cả Huy, Tiến đều không đạt giải. Điều này trái với giả thiết của
đầu bài. Vậy Huy phải nói thật.
3) Nếu Tiến nói đùa thì Tiến đạt giải và cả ba bạn còn lại đều đạt giải. Như
vậy cả bốn bạn đều đạt giải. Điều này trái với giả thiết. Như vậy Tiến nói thật.
Vậy Hoàng đã nói đùa. Có nghĩa là Hoàng đã đạt giải nhì hoặc ba cho nên
Huy đạt giải nhất. Còn Tiến không đạt giải.
Kết luận: Huy đạt giải nhất, Tiến không đạt giải và Hoàng nói đùa.
Ví dụ 3.2.7. Điều mâu thuẫn ở đâu?
Trong một tòa nhà chỉ có những cặp vợ chồng và những con nhỏ chưa lập gia
đình. Ban điều tra dân số yêu cầu báo cáo về số người sống trong tòa nhà, đại
diện là một anh thợ thích đùa báo cáo như sau:
Sống trong tòa nhà bố mẹ nhiều hơn con cái. Mỗi con trai đều có một chị hay
em gái. Số con trai nhiều hơn số con gái. Mỗi cặp vợ chồng đều có con.
Người ta không thể chấp nhận được báo cáo đó (dù là đùa vui) vì trong đó có
mâu thuẫn. Hãy chỉ ra điều mâu thuẫn trong báo cáo trên?
Lời giải: Vì mỗi gia đình đều có con, mỗi con trai đều có một chị gái hay em
gái, nên tất cả các gia đình đều có con gái. Suy ra số con gái ít nhất bằng số gia
đình.
32
Chương 3. Phương pháp suy luận trực tiếp
Mặt khác, số con trai nhiều hơn số con gái, nên tổng số con nhiều hơn hai
lần số gia đình, hay nhiều hơn số bố mẹ, điều này cho ta thấy mâu thuẫn trong
báo cáo của anh thợ thích đùa ở câu đầu tiên "bố mẹ nhiều hơn con cái" với
các câu tiếp theo.
Ví dụ 3.2.8. Khi tổ chức múa hát tập thể một giáo viên đã xếp 20 nữ sinh và
một số nam sinh thành vòng tròn sao cho đối diện với một nữ sinh qua tâm
vòng tròn là một nam sinh. Hỏi trên vòng tròn này có hai nam sinh nào đứng
kề nhau hay không?
Lời giải: Giả sử không có hai nam sinh nào đứng kề nhau, vì vậy cũng không
có hai nữ sinh nào đứng kề nhau. Do đó các bạn nam sinh và nữ sinh được xếp
xen kẽ nhau trên vòng tròn, suy ra có tất cả là 20 nam sinh.
Lấy hai bạn nam và nữ đứng ở vị trí đối xứng nhau qua tâm vòng tròn. Bắt
đầu từ nữ sinh này ta đánh số các bạn đứng trên một nửa vòng tròn cho đến
nam sinh đối diện. Rõ ràng bạn nữ này được đánh số 1 và bạn nam đối diện
đánh số 21. Khi đó các bạn nữ lần lượt được đánh số 1, 3, 5, · · · , 19, còn các bạn
nam được đánh số 2, 4, 6, · · · , 20. Do đó có hai bạn nam sinh được đánh số 20 và
21 đứng kề nhau. Điều này mâu thuẫn với giả thiết ban đầu.
Vậy phải luôn luôn có hai nam sinh đứng kề nhau.
Ví dụ 3.2.9. Trong tám viên bi có bề ngoài hoàn toàn giống nhau có một viên bi
nặng hơn. Bằng hai lần cân trên đĩa (không được dùng quả cân). Hãy xác định
viên bi nặng đó?
Lời giải: Đầu tiên chia tám viên bi thành ba đống theo số lượng 3, 3, 2. Trong
lần cân thứ nhất ta bỏ mỗi bên đĩa cân một đống gồm ba viên bi. Có hai khả
năng xảy ra:
a) Nếu cân thăng bằng. Thì viên bi nặng nằm trong đống hai viên bi.
Khi đó, ta tiến hành cân hai viên bi ở đống gồm hai viên. Mỗi bên đĩa bỏ
một viên bi ở đống hai viên bi. Viên bên chìm chính là viên bi nặng hơn.
b) Nếu cân không thăng bằng, thì bên chìm chứa viên bi nặng hơn.
Khi đó ta tiến hành cân lần thứ hai. Bỏ mỗi đĩa một viên bi trong ba viên
bên chìm.
Nếu cân thăng bằng, thì viên bi nặng hơn chính là viên bi còn lại nằm trên
đĩa chìm.
Nếu cân không thăng bằng, thì viên bi nặng hơn chính là viên bi bên chìm.
Vậy chỉ bằng hai lần cân ta đã xác định được viên bi nặng hơn.
33
Chương 3. Phương pháp suy luận trực tiếp
Ví dụ 3.2.10. Học sinh lớp 11 và 12 tổ chức thi đấu cờ với nhau. Số học sinh
lớp 11 tham gia gấp 10 lần số học sinh lớp 12 tham gia thi đấu xong điểm học
sinh lớp 11 gấp 4, 5 lần số điểm học sinh lớp 12.
Hãy tính xem có bao nhiêu học sinh tham gia đấu cờ. (Nội quy thi đấu là mỗi
người thi đấu một lần với tất cả những người còn lại, người thắng ghi 1 điểm
người thua ghi 0 điểm). Biết rằng tất cả các trận đấu không có trận nào hòa.
Lời giải: Gọi số học sinh lớp 12 tham gia đấu cờ là x. Khi đó số học sinh lớp
11 tham gia đấu cờ là 10x và có tất cả là 11x em tham gia đấu cờ.
Số trận đấu có tất cả là 11x(11x−1)
2 trận và cũng có chừng ấy điểm thắng.
Số điểm thắng của học sinh lớp 11 đạt được là
4, 5
5, 5
×
11x (11x − 1)
2
= 4, 5x (11x − 1)
Các em học sinh lớp 11 sẽ thi đấu với nhau 10x(10−1)
2 trận và sẽ có số điểm là
5x (10x − 1)
Hiển nhiên ta phải có
4, 5x(11x − 1) ≥ 5x(10x − 1)
99x2
− 9x ≥ 100x2
− 10x
x ≥ x2
.
Điều này chỉ xảy ra khi x = 1. Vậy có 11 em tham gia đấu cờ.
34
Chương 4
Phương pháp đồ thị
Rất nhiều bài toán có thể giải bằng cách đưa về bài toán trên đồ thị rồi suy
ra đáp án.
4.1 Một số khái niệm và kết quả cơ bản của lí
thuyết đồ thị
Trên mặt phẳng hay trong không gian lấy n điểm. Giữa một số cặp điểm
được nối bằng những đoạn thẳng hay đoạn cong được định hướng hoặc không.
Người ta gọi hình nhận được là dạng biểu diễn hình học của đồ thị hay một đồ
thị. Các điểm đã chọn được gọi là đỉnh của đồ thị. Các đoạn thẳng hay đoạn
cong đã nối được gọi là cạnh của đồ thị.
Nếu cạnh a nối giữa hai điểm A, B thì A, B được gọi là các đỉnh của cạnh a.
Cặp đỉnh x, y được gọi là hai đỉnh kề nhau, nếu chúng khác nhau và là hai
đầu của cùng một cạnh.
Dãy α các đỉnh
x1, x2, · · · , xi, xi+1, · · · , xm−1, xm
được gọi là một đường, nếu với mọi chỉ số i(1 ≤ i ≤ m − 1) đều có xi và xi+1 là
hai đỉnh kề nhau. Các đỉnh x1, xm được gọi là các đỉnh đầu của đường α. Người
ta còn nói rằng đường α nối giữa đỉnh x1 và đỉnh xm.
Chu trình là một đường có hai đầu trùng nhau.
Chu trình mà nó đi qua mỗi đỉnh không quá một lần được gọi là chu trình
sơ cấp.
Chu trình (α) được gọi là chu trình Hamilton, nếu nó đi qua tất cả các đỉnh
của đồ thị và qua mỗi đỉnh đúng một lần.
35
Chương 4. Phương pháp đồ thị
Đồ thị G được gọi là đồ thị liên thông, nếu mỗi cặp đỉnh của nó đều có đường
nối với nhau.
Đồ thị G được gọi là đồ thị đầy đủ nếu mỗi cặp đỉnh của nó được nối với
nhau bằng đúng một cạnh.
Số cạnh xuất phát từ đỉnh x được gọi là bậc của đỉnh x.
Cây là đồ thị liên thông và không có chu trình.
Trong cây T tách ra một đỉnh được gọi là đỉnh gốc, còn các đỉnh có bậc bằng
1 và không phải gốc được gọi là lá hay đỉnh ngọn.
Định lý 4.1.1. Đồ thị mà trong đó tổng bậc của hai đỉnh tùy ý đều không nhỏ
hơn số đỉnh của đồ thị, liên thông.
Định lý 4.1.2. Đồ thị mà trong đó bậc của mỗi đỉnh đều không nhỏ hơn 2, luôn
luôn có chu trình sơ cấp.
Hệ quả 4.1.1. Nếu trong đồ thị có đúng 2 đỉnh bậc 1, các đỉnh khác có bậc
không nhỏ hơn 2, thì trong G có đường nối giữa hai đỉnh bậc 1.
Định lý 4.1.3. Đồ thị, mà trong đó tổng bậc của hai đỉnh tùy ý không nhỏ hơn
số đỉnh của đồ thị, luôn luôn có chu trình Hamilton.
Định lý 4.1.4. Trong một đồ thị tùy ý số đỉnh, mà mỗi đỉnh có bậc lẻ, luôn
luôn là một số chẵn.
Định lý 4.1.5. Cho dãy số nguyên dương a1 = 2, a2 = 5, · · · , an+1 = (n+1)an +1.
Khi đó đồ thị đầy đủ an+1 đỉnh với các cạnh được tô bằng n màu luôn luôn có
tam giác cùng màu (chu trình gồm ba cạnh cùng màu).
Định lý 4.1.6. Cho dãy số nguyên b2 = 3, b3 = 6, bn+1 = (bn −1)n+2. Đồ thị đầy
đủ G với bn+1 − 1 đỉnh (n ≥ 2) và các cạnh được tô bằng n màu, sao cho không
có tam giác cùng màu, thì trong đồ thị G có hình 5 cạnh với các cạnh cùng màu
và các đường chéo được tô các màu khác.
4.2 Phương pháp đồ thị
Để giải bài toán T bằng cách thông qua đồ thị, cần thực hiện lần lượt hai
bước sau
36
Chương 4. Phương pháp đồ thị
4.2.1 Xây dựng đồ thị mô tả các quan hệ
Lấy các điểm trên mặt phẳng hoặc trong không gian tương ứng với các đối
tượng đã cho trong bài toán. Dùng ngay các kí hiệu đối tượng để ghi trên điểm
tương ứng...
Cặp điểm x, y được nối với nhau bằng một cạnh với "đặc điểm t", khi và chỉ
khi các đối tượng x, y có quan hệ (t) với nhau. Khi đó bài toán T đã được chuyển
về bài toán D trên đồ thị.
4.2.2 Dựa vào các kết quả của lý thuyết đồ thị hoặc lý luận
trực tiếp suy ra đáp án của bài toán D
Nếu đáp án của bài toán D còn dưới dạng "ngôn ngữ đồ thị", thì căn cứ vào
phép đặt tương ứng khi xây dựng đồ thị mà diễn đạt thành đáp án bằng ngôn
ngữ thông thường (tức là đáp án của bài toán T).
4.3 Một số ví dụ
Ví dụ 4.3.1. Trong một cuộc thi đấu bóng bàn An và Bình quy ước với nhau:
Người thắng cuộc là người đầu tiên thắng ba ván hoặc thắng hai ván liên tiếp.
Hãy xác định số khả năng có thể xảy ra?
Lời giải: Dùng A để kí hiệu An thắng, B để kí hiệu Bình thắng. Dùng cây để
mô tả toàn bộ hiện trạng có khả năng xảy ra.
Xây dựng cây: Xuất phát từ điểm S.
Ván đầu tiên có hai khả năng xảy ra: An thắng hoặc Bình thắng, nên lấy hai
điểm sao cho hai điểm này với S không thẳng hàng. Một trong hai điểm này ghi
A, còn điểm kia ghi B. Nối S với A bằng một đoạn thẳng hoặc một đoạn cong
biểu thị A thắng. Tương tự, để biểu thị B thắng nối S với B bằng một đoạn
thẳng hoặc một đoạn cong.
Ván thứ hai lại có hai khả năng: An thắng hoặc Bình thắng, nên xuất phát
từ A cũng lấy hai điểm mới và ghi các kí hiệu tương ứng A, B và từ A kẻ hai
đoạn thẳng hoặc hai đoạn cong tới hai điểm mới thêm. Đối với điểm B cũng
chọn thêm hai đỉnh mới ghi A và B, rồi từ B kẻ hai đoạn thẳng hay hai đoạn
cong tới hai điểm mới thêm.
Tiếp theo thực hiện kéo dài các đường một cách tương tự, nhưng do quy ước
của An và Bình những đường mà trên đó xuất hiện hoặc hai đỉnh liên tiếp ghi
37
Chương 4. Phương pháp đồ thị
cùng bằng một kí hiệu hoặc có ba đỉnh được ghi bằng cùng một kí hiệu đều
không được kéo dài.
S
A
B
B
B
B
B
B
BB
B
A
A
A
A
A
AA
A
Hình 4.1
Vì An và Bình đấu với nhau năm ván, thì hoặc có người thắng hai ván liên
tiếp hoặc có người thắng ba ván. Do đó những đường xuất phát từ S đều không
có quá năm cạnh. (Hình 4.1)
Cây có 10 đỉnh ngọn nên có 10 khả năng xảy ra.
Ví dụ 4.3.2. Trên đảo có một số cụm dân cư. Mỗi cụm dân cư có hai đường
lớn và ba đường mòn đi ra. Mỗi đường lớn cũng như đường mòn đều dẫn tới
một cụm dân cư khác. Hai cụm dân cư khác nhau bất kì được nối liền bằng hoặc
đường lớn hoặc đường mòn. Hỏi trên đảo này có bao nhiêu đường mòn, bao nhiêu
đường lớn?
Lời giải: Trước hết ta cần khẳng định rằng trên đảo có sáu cụm dân cư.
Thật vậy, mỗi cụm dân cư ta biểu diễn bằng một điểm. Hai cụm dân cư có
đường lớn (đường mòn) nối với nhau, thì hai cụm tương ứng được nối bằng một
đường nét liền (đường nét đứt). Vì xuất phát từ mỗi cụm dân cư có hai đường
38
Chương 4. Phương pháp đồ thị
lớn và ba đường mòn đi ra, nên xuất phát từ mỗi điểm đã chọn, chẳng hạn A,
có hai đường nét liền và ba đường nét đứt. Bởi vậy mỗi điểm đã chọn A được
nối với các điểm khác B, C, D, E, F bằng một đường nét liền hoặc một đường nét
đứt. Như vậy phải có ít nhất sáu cụm dân cư (6 điểm đã chọn). Mặt khác hai
cụm dân cư tùy ý đều phải có đường nối với nhau, nên mỗi điểm đều chỉ có thể
nối tới với năm điểm. Do đó không còn điểm nào ngoài sáu điểm A, B, C, D, E, F.
Bởi vậy, có sáu đường lớn và chín đường nhỏ.
Căn cứ vào yêu cầu của bài toán ta có thể đưa ra một vài khả năng xảy ra
các đường nối giữa các cụm dân cư như hình 4.2.
B
C
D
E
F
A A
B
C C
D
E
F
Hình 4.2
Ví dụ 4.3.3. Tại một giải bóng đá có bốn đội Anh, Đan Mạch, Hà Lan, Thụy
Điển vào bán kết. Có mấy dự đoán xếp hạng như sau
1. Đan Mạch vô địch, Thụy Điển nhì.
2. Đan Mạch nhì, Hà Lan ba.
3. Anh nhì, Hà Lan tư.
Kết quả là mỗi dự đoán đúng được một đội. Hãy cho biết kết quả xếp hạng
của các đội?
Lời giải: Dùng xi để kí hiệu đội x được xếp hạng i(1 ≤ i ≤ 4). Ta vẽ cây, hai
nhánh đầu tiên ứng với dự đoán thứ nhất là D1, T2. Từ mỗi nhánh này ta lại có
hai nhánh tương ứng với dự đoán thứ hai. Tiếp tục rẽ nhánh với dự đoán thứ
ba.
Ta chọn đường đi từ gốc 0 tới các điểm thỏa mãn các điều kiện:
• Một đội không thể được xếp hai hạng khác nhau.
• Hai đội không thể xếp cùng một hạng.
39
Chương 4. Phương pháp đồ thị
Suy ra chỉ có đường đi D1H3A2 thỏa mãn.
O
D1
D2
A2 A2 A2
A2
H4 H4 H4 H4
H3H3
D2
T2
Hình 4.3
Đường tô đậm D1H3A2 thỏa mãn điều kiện mỗi dự đoán đúng được một đội
mà thứ tự ghi trên đường. Vậy kết quả xếp hạng như sau:
Đan Mạch vô địch.
Anh nhì.
Hà Lan ba.
Còn lại là Thụy Điển thứ tư.
Ví dụ 4.3.4. Cho 6 số nguyên dương tùy ý. Chứng minh rằng luôn có thể chọn
ra được 2 bộ 3 số mà trong mỗi bộ, từng đôi một đều là nguyên tố cùng nhau
hoặc đều không nguyên tố cùng nhau.
Lời giải:
Bước 1: Chuyển bài toán sang bài toán về đồ thị màu G(X, E).
- Đỉnh: Cho tương ứng mỗi số với một đỉnh X = {A, B, C, D, E, F}
- Cạnh: Đoạn nối giữa hai đỉnh tương ứng với hai số
• Nguyên tố cùng nhau được tô màu xanh (đường nét đứt).
• Không nguyên tố cùng nhau được tô màu đỏ (đường nét liền).
Khi đó, ta được bài toán đồ thị màu: Trong đồ thị đầy đủ G(X, E) có 6 đỉnh
với hai cạnh màu, thì luôn luôn có thể tìm được hai tam giác với cạnh cùng
màu.
Giả sử đỉnh A là mút của AB, AC, AD cùng màu đỏ (đường nét liền) như
hình vẽ dưới đây
40
Chương 4. Phương pháp đồ thị
A
B
C
D
Hình 4.4
Ta xét tam giác được lập nên từ ba đỉnh đối của A là BCD. Trong tam giác
BCD hai khả năng có thể xảy ra:
1) Có ít nhất một cạnh màu đỏ. Chẳng hạn cạnh BC màu đỏ, khi đó tam
giác ABC có cạnh màu đỏ. (Hình 4.5a)
2) Tam giác BCD không có cạnh nào màu đỏ. Nghĩa là tam giác BCD có
cạnh đều là màu xanh. (Hình 4.5b)
A
BC
F
E
D
A
B
D
C
Hình 4.5a Hình 4.5b
Vậy với mọi trường hợp, trong đồ thị G đều có tam giác cùng màu (theo
Định lý (4.1.5))
Giả sử rằng trong G có tam giác màu đỏ, chẳng hạn tam giác ABC màu đỏ.
Ta chứng minh tiếp G còn có tam giác thứ hai nữa với các cạnh cùng màu.
Nếu trong G, ta tạm thời không xét đến một đỉnh của tam giác ABC, chẳng
hạn đỉnh A cùng tất cả các cạnh thuộc nó. Ta được đồ thị con đầy đủ G1 có 5
đỉnh.
Nếu trong G1 có tam giác cùng màu, thì bài toán đã được giải xong.
41
Chương 4. Phương pháp đồ thị
Ngược lại, trong G1 không có tam giác cùng màu, thì theo Định lý (4.1.6) với
n = 2, ta có thể biểu diễn G1 thành một hình 5 cạnh với các cạnh màu đỏ và
đường chéo màu xanh.
Bây giờ, ta khôi phục lại đỉnh thứ 6 A và các cạnh màu thuộc nó. Xét hai
cạnh AD và AF nếu chúng đều màu xanh thì ta có tam giác mới ADF có màu
xanh. Nếu AD hoặc AF màu đỏ thì ta có tam giác màu đỏ nữa được hình thành
là ABD hoặc ACF. Vậy ngoài tam giác ABC, ta có thêm một tam giác nữa có
các cạnh cùng màu. Khẳng định đã được chứng minh.
C
F
E
D
B
C B
D
E
F
AHình 4.6
Bước 3: Thuyết minh lời giải
Trong G luôn tồn tại hai tam giác với cạnh cùng màu. Nếu cả hai tam giác
đều màu đỏ, thì ta có hai bộ ba số, mà trong mỗi bộ chúng đôi một nguyên tố
cùng nhau. Nếu chỉ có một tam giác màu đỏ, thì ta được một bộ ba số đôi một
nguyên tố cùng nhau, và một bộ ba số đôi một không nguyên tố cùng nhau.
Nếu cả hai tam giác màu xanh, nghĩa là ta được hai bộ ba số, mà trong mỗi bộ,
chúng đôi một không nguyên tố cùng nhau.
Ví dụ 4.3.5. Để mừng con đạt giải trong kì thi toán quốc tế lần thứ 42, một gia
đình dự định mời tiệc. Trong số khách mời:
a) Người chồng muốn có ít nhất ba người từng đôi một quen nhau.
42
Chương 4. Phương pháp đồ thị
b) Người vợ lại muốn có ít nhất bốn người từng đôi một chưa quen nhau.
Hỏi họ phải mời ít nhất bao nhiêu bạn, để mong muốn của chồng của vợ được
thỏa mãn?
Lời giải:
Bước 1: Xây dựng đồ thị G(X, E) mô tả quan hệ của các đối tượng.
• Đỉnh: Mỗi khách cho tương ứng với một đỉnh.
• Cạnh: Đoạn nối giữa hai đỉnh tương ứng với hai khách:
– Quen nhau được tô màu xanh (nét liền).
– Không quen nhau tô màu đỏ (nét đứt).
Khi đó đồ thị G(X, E) mô tả quan hệ quen biết của các khách dự kiến mời.
Theo cách xây dựng trên, để thực hiện yêu cầu của người chồng, thì trong số
khách mời phải có 3 người quen nhau từng đôi một, nên đồ thị G(X, E) phải
có ít nhất một tam giác màu xanh, còn để thỏa mãn yêu cầu của người vợ, thì
trong số khách mời phải có ít nhất 4 người từng đôi một không quen biết nhau,
nên trong G(X, E) phải có ít nhất một tứ giác mà các cạnh và các đường chéo
của nó phải có màu đỏ. Do đó bài toán được dẫn về bài toán trên đồ thị: Tìm
số đỉnh ít nhất để đồ thị đầy đủ G(X, E) với các cạnh màu xanh hoặc đỏ mà
a) Hoặc có ba đỉnh được nối với nhau bởi cạnh màu xanh,
b) Hoặc bốn đỉnh từng đôi một được nối với nhau bởi cạnh màu đỏ.
Bước 2: Giải bài toán đồ thị G
1) Nếu G có 8 đỉnh thì có thể xảy ra trường hợp mà cả (a) lẫn (b) không thỏa
mãn. Như cách tô màu G ở hình 4.7, thì không có tam giác nào xanh, hoặc tứ
giác nào với cạnh và đường chéo cùng màu đỏ.
43
Chương 4. Phương pháp đồ thị
x1
x2
x3
x4
x5
x6
x7
x8
Hình 4.7
2) Xét G(X, E) đầy đủ với 9 đỉnh, các cạnh được tô bằng hai màu xanh, đỏ.
X = {x1, x2, . . . , x8, x9}
Hai khả năng có thể xảy ra:
10) Có 4 cạnh màu xanh xuất phát từ một đỉnh.
Chẳng hạn x1 có 4 cạnh x1x2, x1x3, x1x4, x1x5 màu xanh. Khi đó nếu trong các
cạnh nối đôi một giữa 4 đỉnh x2, x3, x4, x5 có một cạnh xanh, giả sử cạnh x3x4
xanh, thì có tam giác x1x3x4 cạnh xanh hình 4.8 a được thỏa mãn, nếu không
thì tứ giác x2x3x4x5 có cạnh và các đường chéo màu đỏ hình 4.8 b, (b) được thỏa
mãn.
44
Chương 4. Phương pháp đồ thị
x1
x2
x3
x4
x5
x1
x2
x3
x4
x5
Hình 4.8 a Hình 4.8 b
20) Có nhiều nhất là 3 cạnh màu xanh xuất phát từ mỗi đỉnh.
Bởi vậy, phải có ít nhất một đỉnh, x1 chẳng hạn, là đầu mút của không quá 2
cạnh màu xanh (vì nếu mỗi đỉnh là đầu mút của đúng 3 cạnh màu xanh, thì số
cạnh màu xanh là 3×9
2 /∈ N). Khi đó, tại x1 phải là đầu mút của ít nhất 6 cạnh
đỏ. Giả sử 6 cạnh đỏ đó là x1x4, x1x5, x1x6, x1x7, x1x8, x1x9.
Xét đồ thị con đầy đủ G1 có 6 đỉnh x4, x5, . . . , x9 với 2 màu cạnh. Theo định
lý (4.1.5) với n = 2, trong G1 có tam giác cùng màu. Nếu là tam giác màu xanh,
thì (a) được thỏa mãn, nếu là tam giác màu đỏ thì 3 đỉnh này cùng với đỉnh x1
tạo thành đồ thị con đầy đủ có 4 đỉnh với các cạnh đều màu đỏ, (b) được thỏa
mãn.
x1
x2
x3
x4
x5
x6
x7
x8x9
Hình 4.9
Bước 3: Chuyển về ngôn ngữ xuất phát.
45
Chương 4. Phương pháp đồ thị
Đồ thị đầy đủ với 9 đỉnh, các cạnh được tô bằng hai màu xanh, đỏ thì luôn
có hoặc tam giác xanh hoặc tứ giác mà các cạnh và đường chéo đều màu đỏ,
nhưng với 8 đỉnh thì tính chất trên không tồn tại. Vậy số đỉnh của đồ thị G phải
ít nhất bằng 9, nên số khách ít nhất họ phải mời là 9, thì mong muốn của vợ
hoặc chồng được thỏa mãn.
Ví dụ 4.3.6. Lấy n(n ≥ 4) số nguyên dương khác nhau tùy ý, sao cho cứ 4 số
bất kỳ có ít nhất một số có ước chung với 3 số còn lại. Chứng minh rằng trong
các số đã chọn ra có ít nhất n − 3 số, mà mỗi số này có ước chung với tất cả các
số đã chọn ra.
Lời giải:
Bước 1: Xây dựng đồ thị mô tả các quan hệ.
Đỉnh: Lấy n điểm trên mặt phẳng hay trong không gian tương ứng với các
số đã chọn ra và ghi các số này lên các điểm tương ứng.
Cạnh: Cặp điểm x, y được nối bằng một đoạn thẳng hay đoạn cong không đi
qua các điểm tương ứng trung gian khác khi và chỉ khi cặp x, y có ước chung.
Đồ thị G nhận được mô tả toàn bộ quan hệ có ước chung giữa các số đã chọn
ra.
Bước 2: Chứng minh trong đồ thị G có ít nhất n − 3 đỉnh có bậc n − 1
a) Nếu trong G có hai cặp đỉnh không kề nhau, thì chúng phải có đỉnh chung.
Thật vậy, giả sử trong G có hai cặp đỉnh không kề nhau, chẳng hạn A, B và C, D
nhưng chúng không có đỉnh chung. Khi đó trong 4 đỉnh này không có một đỉnh
nào kề với 3 đỉnh còn lại. Ta đi tới mâu thuẫn với giả thiết. Vậy hai cặp A, B và
C, D phải có một đỉnh chung chẳng hạn B ≡ D.
b) Trong đồ thị G có không ít hơn n − 3 đỉnh có bậc n − 1
- Nếu trong G tất cả các đỉnh đều kề nhau từng đôi một, thì cả n đỉnh của
G đều có bậc n − 1.
- Nếu trong G có duy nhất một cặp đỉnh không kề nhau, thì G có n − 2 đỉnh
bậc n − 1.
- Giả sử G có hai cặp đỉnh không kề nhau, khi đó theo phần a) hai cặp đỉnh
này phải có đỉnh chung. Giả sử các đỉnh đó là A, B, C. Nếu sử dụng đường nét
đứt để biểu thị tính không kề nhau của các đỉnh ta có hình 4.10a.
Kết nạp đỉnh E tùy ý vào nhóm A, B, C. Khi đó trong bộ 4: A, B, C, E phải
có ít nhất một đỉnh kề với 3 đỉnh còn lại. Đỉnh này không thể là các đỉnh A, B, C
mà phải là E. Vậy E kề với cả ba đỉnh A, B, C. (Hình 4.10b)
46
Chương 4. Phương pháp đồ thị
A B
C
A B
C
E
Hình 4.10a Hình 4.10b
Loại bỏ một trong ba đỉnh A, B, C khỏi bộ 4, chẳng hạn C và kết nạp đỉnh
tùy ý F. Khi đó trong bộ 4 mới A, B, E, F lại có ít nhất một đỉnh kề với 3 đỉnh
còn lại. Nếu F là đỉnh kề với 3 đỉnh còn lại, thì nó phải kề với E, nên trong mọi
trường hợp E vẫn là đỉnh kề với 3 đỉnh còn lại. (Hình 4.11)
A B
E F
Hình 4.11
Vì F là đỉnh tùy ý của đồ thị G và khác với các đỉnh A, B, C, E nên E kề với
tất cả các đỉnh còn lại của G. Bởi vậy, bậc của E bằng n − 1.
Đỉnh E lại được chọn một cách tùy ý trong các đỉnh của đồ thị G khác với
A, B, C nên trong G chỉ có ba đỉnh A, B, C bậc nhỏ hơn n − 1.
Bước 3: Thuyết minh lời giải
Theo cách xác định cạnh của đồ thị G cặp đỉnh x, y kề nhau khi và chỉ khi
cặp số x, y có ước chung, mà trong G là có ít nhất n − 3 đỉnh kề với tất cả các
đỉnh còn lại. Do đó trong n số đã chọn ra có ít nhất n − 3 số có ước chung với
tất cả các số còn lại.
47
Chương 4. Phương pháp đồ thị
Ví dụ 4.3.7. Một cơ quan cần tuyển ba người để thành lập một nhóm có đủ
năng lực biên dịch các tài liệu từ sáu thứ tiếng: Anh, Pháp, Nga, Đức, Trung
Quốc và Bồ Đào Nha sang tiếng Việt. Có bảy người đến dự tuyển, trong đó mỗi
người đều biết hai và chỉ hai trong sáu thứ tiếng nói trên và bất cứ hai người nào
cũng biết nhiều nhất một thứ tiếng chung trong sáu thứ tiếng đó. Biết rằng thứ
tiếng nào cũng có ít nhất hai người biết. Liệu có thể xảy ra trường hợp không
tuyển chọn được như yêu cầu đã nêu hay không?
Lời giải: Ta sẽ chứng minh khẳng định đúng, tức là có thể tuyển chọn được
như yêu cầu đã nêu.
1. Trước hết chứng minh khẳng định
Cho đơn đồ thị G có 6 đỉnh và 7 cạnh, bậc của mỗi đỉnh lớn hơn hoặc bằng
2. Khi đó trong G luôn tìm được 3 cạnh đôi một không kề nhau.
a) Đồ thị G liên thông.
Thật vậy, nếu G không liên thông thì do G gồm 6 đỉnh với bậc không bé hơn
2, phải chia thành 2 tam giác nên G chỉ có 6 cạnh. Ta đi tới mâu thuẫn với điều
kiện đồ thị gồm 7 cạnh.
b) Đồ thị phải có chu trình
Thật vậy, nếu G không có chu trình và G lại liên thông nên G là một cây.
Khi đó G có đỉnh treo, tức là đỉnh bậc 1, nên mâu thuẫn với điều kiện bậc của
mỗi đỉnh thuộc G đều không bé hơn 2.
c) Vì 7 × 2 = 14 = 6 × 2 + 2, nên G phải có 2 đỉnh bậc 3 hoặc 1 đỉnh bậc 4,
các đỉnh còn lại bậc 2.
Ta xét từng trường hợp:
Trường hợp a: G có một đỉnh bậc 4 (chẳng hạn đỉnh A). Không mất tính
tổng quát, ta giả sử 4 cạnh xuất phát từ A là AB, AD, AN, AT.
Lúc đó đỉnh P không thể kề với A, phải kề với 2 đỉnh khác, chẳng hạn P và
N và 2 đỉnh còn lại (B và D) phải kề nhau. Khi đó ba cạnh không kề nhau từng
đôi một là BD, NT, PA. (Hình 4.12)
48
Chương 4. Phương pháp đồ thị
A
B
D N
P
T
Hình 4.12
Trường hợp b: G có hai đỉnh bậc 3. Giả sử A và P là hai đỉnh bậc 3. Vì G
liên thông, nên có ít nhất một đường đi từ A đến P. Ta tạm bỏ đường đó đi
(vẫn giữ nguyên các đỉnh A và P). Có thể xảy ra hai khả năng:
b1) G không còn liên thông. Như vậy, ta có hai chu trình sơ cấp phân biệt,
mỗi chu trình gồm 3 cạnh. Suy ra ta chọn cạnh AP và hai cạnh không kề với nó
là BT, ND. (Hình 4.13)
T
B
A
P
N
D
Hình 4.13
b2) G vẫn liên thông. Như vậy ta còn một chu trình sơ cấp (vì mỗi đỉnh đều
có bậc 2). Ta có 3 đường đi từ A đến P (một đường đã tạm bỏ, hai đường theo
chu trình), trong đó đường ngắn có độ dài 1 hoặc độ dài 2.
49
Chương 4. Phương pháp đồ thị
N
A
B
T
P
D
A
N B
P
T
D
Hình 4.14a Hình 4.14b
- Khi AP có độ dài 1, trên chu trình sơ cấp có độ dài 6 ta luôn chọn được ba
cạnh không kề nhau AD, PT, BN. (Hình 4.14a)
- Khi AP có độ dài 2, chỉ việc chọn một cạnh thuộc đường đi ngắn nhất từ
A đến P và hai cạnh không kề với nó trên chu trình sơ cấp còn lại, chẳng hạn
AD, TP, NB hoặc DP, AT, NB. (Hình 4.14b)
Vậy trong mọi trường hợp ta đều tìm được ba cạnh đôi một không kề nhau.
Khẳng định được chứng minh.
2. Chuyển bài toán về dạng đồ thị
Dùng 6 điểm tương ứng với 6 ngoại ngữ nói trên. Dùng ngay chữ cái đầu của
tên ngoại ngữ để ghi tên các điểm tương ứng: A, P, N, D, T, B.
Mỗi người biết hai ngoại ngữ nào đó được biểu thị bởi một cạnh nối hai đỉnh
tương ứng với hai ngoại ngữ đó.
Đồ thị nhận được kí hiệu là G.
Có 7 người tham gia dự tuyển và mỗi người biết hai ngoại ngữ, do đó đồ thị
G có 7 cạnh.
Bất kỳ hai người dự tuyển nào cũng chỉ cùng biết tối đa một ngoại ngữ nói
trên, nên trong đồ thị G không có một cặp đỉnh nào được nối bằng hai cạnh.
Vậy G là một đồ thị đơn gồm 6 đỉnh 7 cạnh. Do đó, theo khẳng định trên,
luôn luôn tìm được ba cạnh không kề nhau từng đôi một. Khi đó chỉ việc chọn
ba người được biểu thị bằng 3 cạnh nói trên, thì mỗi người này đều biết hai
ngoại ngữ và không có hai người nào cùng biết một ngoại ngữ. Do đó nhóm ba
người này biết tất cả sáu ngoại ngữ: Anh, Pháp, Nga, Đức, Trung Quốc và Bồ
Đào Nha.
50
Chương 4. Phương pháp đồ thị
Ví dụ 4.3.8. Mười bảy nhà khoa học đến dự hội nghị Quốc tế. Mỗi người trong
số họ chỉ biết một trong ba ngoại ngữ: Anh, Nga, Pháp. Chứng minh rằng có ít
nhất 3 nhà khoa học cùng biết một trong ba ngoại ngữ nói trên.
Lời giải:
1. Xây dựng đồ thị mô tả quan hệ.
-) Đỉnh: Lấy 17 điểm trên mặt phẳng hay trong không gian tương ứng với 17
nhà khoa học. Dùng ngay tên các nhà khoa học để ghi trên các điểm tương ứng.
-) Cạnh tô màu: Cặp điểm x, y được nối bằng đoạn thẳng hay đoạn cong tô
• Màu xanh khi và chỉ khi hai nhà khoa học tương ứng cùng biết tiếng Pháp.
• Màu vàng khi và chỉ khi hai nhà khoa học tương ứng cùng biết tiếng Anh.
• Màu đỏ khi và chỉ khi hai nhà khoa học tương ứng cùng biết tiếng Nga.
Đồ thị G nhận được mô tả toàn bộ hiện trạng về ngoại ngữ của các nhà khoa
học đến dự hội nghị.
2. Đáp án của bài toán bằng ngôn ngữ đồ thị
Đồ thị G gồm 17 đỉnh, đầy đủ và các cạnh được tô bằng ba màu, nên theo
Định lý 4.1.5, với n = 3, có tam giác cùng màu, khi đó ba nhà khoa học tương
ứng với các đỉnh của tam giác cùng biết một trong ba ngoại ngữ: Anh, Pháp,
Nga.
Ví dụ 4.3.9. Một quần đảo có 2n(n ≥ 1) hòn đảo. Mỗi hòn đảo có đường ngầm
nối trực tiếp với ít nhất n hòn đảo khác. Chứng minh rằng từ một hòn đảo bất
kỳ thuộc quần đảo đều có thể đi tới bất kỳ hòn đảo nào khác thuộc quần đảo này
bằng đường ngầm.
Lời giải:
1. Xây dựng đồ thị mô tả các quan hệ
• Đỉnh: Lấy 2n điểm tương ứng với 2n hòn đảo. Dùng ngay tên các hòn đảo
để ghi các điểm tương ứng.
• Cạnh: Cặp điểm x, y được nối bằng một đoạn thẳng hoặc một đoạn cong
không đi qua các điểm chung gian khác khi và chỉ khi hai hòn đảo tương
ứng có đường ngầm nối với nhau.
51
Chương 4. Phương pháp đồ thị
Đồ thị nhận được kí hiệu bằng G. Nó mô tả toàn bộ mối liên hệ bằng đường
ngầm của quần đảo.
2. Đáp án của bài toán bằng "ngôn ngữ đồ thị"
Đối với mỗi đỉnh x của đồ thị G số cạnh xuất phát từ nó bằng đúng số đường
ngầm xuất phát từ hòn đảo tương ứng. Bởi vậy bậc của mỗi đỉnh không nhỏ
hơn n, nên tổng bậc của hai đỉnh tùy ý không nhỏ hơn số đỉnh của đồ thị (2n).
Khi đó, theo Định lý (4.1.1), đồ thị G liên thông. Do đó hai đỉnh tùy ý x, y của
đồ thị G có đường nối với nhau. Mặt khác, mỗi cạnh của đường này biểu thị
một đường ngầm nối trực tiếp giữa hai hòn đảo, nên đường nối giữa hai đỉnh
x, y biểu thị đường ngầm nối giữa hai hòn đảo tương ứng với x và y. Do đó từ
hòn đảo bất kỳ đều có thể đi bằng đường ngầm tới các hòn đảo còn lại của quần
đảo.
52
Chương 5
Phương pháp bảng
5.1 Giới thiệu về phương pháp bảng
Nhiều bài toán phổ thông có thể giải bằng cách lập bảng mô tả mối quan hệ
giữa các đối tượng được cho trong bài toán.
Đối với một số bài toán phổ thông trong đó xuất hiện hai hay nhiều đối
tượng và các cặp phần tử nói lên mối quan hệ giữa các tệp người ta có thể thiết
lập một hay nhiều bảng, để mô tả mối quan hệ giữa các tệp.
Mỗi bảng này có hàng trên cùng ghi các phần tử của một tệp, còn cột tận
cùng bên trái ghi các phần tử thuộc tập kia và các vị trí trong bảng ghi mã số
quan hệ giữa các phần tử thuộc các tệp.
Căn cứ vào các điều kiện đã cho trong bài toán gạch bỏ đi những cặp phần
tử không thích hợp từ đó đi đến lời giải của bài toán.
5.2 Một số ví dụ minh họa
Ví dụ 5.2.1. Trong buổi học nữ công ba bạn Cúc, Đào, Hồng làm ba bông hoa:
cúc, đào, hồng. Bạn làm hoa hồng nói với bạn Cúc "Thế là trong chúng ta không
có ai làm loại hoa trùng với tên mình". Hãy xác định tên hoa mà mỗi bạn đã
làm?
Lời giải:
Bài toán này có hai tệp đối tượng. Tệp thứ nhất gồm các bạn làm hoa, tệp
thứ hai gồm các bông hoa được làm. Ta có thể giải bằng phương pháp bảng như
sau
1. Lập bảng
53
Chương 5. Phương pháp bảng
Bảng cần lập gồm 4 hàng và 4 cột. Hàng đầu, từ cột thứ hai ghi lần lượt tên
các bông hoa được làm viết tắt là các chữ cái đầu, còn trên cột tận cùng bên
trái từ hàng hai ghi lần lượt tên các bạn tham gia làm hoa viết tắt là chữ cái
đầu viết hoa.
2. Điền mã số quan hệ vào các vị trí của bảng
a) Căn cứ vào giả thiết: Mỗi bạn đều không làm hoa trùng với tên mình, mà
điền mã "k" vào các ô nằm trên đường chéo chính.
Nguoi
c d h
hoa
C
D
H
k
k
k
k
Bảng 5.1
b) Căn cứ vào câu "Bạn làm hoa hồng nói với bạn Cúc" suy ra bạn Cúc
không phải làm hoa hồng, mà ghi mã "k" vào ô nằm ở hàng Cúc, cột hồng.
3. Loại bỏ vị trí không thỏa mãn quan hệ để nhận được lời giải
Trong bảng trên cột cuối vị trí 1 và 3 bị gạch bỏ, nên vị trí duy nhất còn lại
là vị trí thứ hai phải thỏa mãn quan hệ giữa người làm hoa và hoa được làm.
Do đó bạn Đào làm hoa hồng.
Vì trên hàng 2 Đào đã có vị trí thỏa mãn quan hệ nên toàn bộ hàng này bị
loại ra khỏi diện xét. Bởi vậy cột Cúc chỉ còn vị trí cuối cùng trong diện xét.
Bởi vậy nó phải thỏa mãn quan hệ giữa người làm hoa và hoa được làm, nên
bạn Hồng làm hoa cúc.
Từ đó suy ra người còn lại bạn Cúc phải làm hoa đào.
Vậy Bạn Cúc làm hoa đào, Bạn Đào làm hoa hồng, Bạn Hồng làm hoa cúc.
Ví dụ 5.2.2. Ba bạn Long, Hoan, Lan đều là học sinh giỏi toán. Cô giáo thưởng
cho ba bạn thứ đồ chơi là: Búp bê, ô tô và con quay điện. Biết rằng bạn Long
không thích chơi búp bê, bạn Hoan không nhận búp bê và cũng không thích ô tô.
Hỏi cô giáo đã thưởng cho ai đồ chơi gì?
Lời giải: Bài toán gồm hai nhóm đối tượng:
54
Chương 5. Phương pháp bảng
• Nhóm thứ nhất gồm các bạn: Long, Hoan, Lan.
• Nhóm thứ hai là các đồ chơi mà cô giáo thưởng cho các bạn: Búp bê, ô tô,
con quay.
Ta lập một bảng có hàng trên là tên các bạn, và cột bên trái là các đồ chơi.
Long Hoan Lan
Búp bê 0 0 ×
Ô tô × 0
Con quay ×
Bảng 5.2
Nhìn vào bảng, ta suy luận và có kết quả sau:
Lan được thưởng con búp bê.
Long được thưởng ô tô.
Hoan được thưởng con quay.
Ví dụ 5.2.3. Có sáu em học sinh: An, Bình, Nam, Long, Trang, Minh tham
gia cuộc thi leo núi. Có hai em về được đích, đáp lại câu hỏi ai về tới đích, có
năm câu trả lời:
1. An và Nam.
2. Long và Trang.
3. Minh và Long.
4. An và Minh.
5. An và Linh.
Thật ra trong năm câu trả lời trên có 4 câu mà mỗi câu chỉ đúng một nửa
(đúng một tên), còn sai một nửa, còn một câu sai cả hai tên. Hỏi ai đã về tới
đích của cuộc thi?
Lời giải: Dựa vào năm câu trả lời, ta lập bảng sau:
55
Luận văn: Sáu phương pháp giải các bài toán phổ thông, HOT
Luận văn: Sáu phương pháp giải các bài toán phổ thông, HOT
Luận văn: Sáu phương pháp giải các bài toán phổ thông, HOT
Luận văn: Sáu phương pháp giải các bài toán phổ thông, HOT
Luận văn: Sáu phương pháp giải các bài toán phổ thông, HOT
Luận văn: Sáu phương pháp giải các bài toán phổ thông, HOT
Luận văn: Sáu phương pháp giải các bài toán phổ thông, HOT
Luận văn: Sáu phương pháp giải các bài toán phổ thông, HOT
Luận văn: Sáu phương pháp giải các bài toán phổ thông, HOT
Luận văn: Sáu phương pháp giải các bài toán phổ thông, HOT
Luận văn: Sáu phương pháp giải các bài toán phổ thông, HOT
Luận văn: Sáu phương pháp giải các bài toán phổ thông, HOT
Luận văn: Sáu phương pháp giải các bài toán phổ thông, HOT
Luận văn: Sáu phương pháp giải các bài toán phổ thông, HOT
Luận văn: Sáu phương pháp giải các bài toán phổ thông, HOT
Luận văn: Sáu phương pháp giải các bài toán phổ thông, HOT
Luận văn: Sáu phương pháp giải các bài toán phổ thông, HOT
Luận văn: Sáu phương pháp giải các bài toán phổ thông, HOT
Luận văn: Sáu phương pháp giải các bài toán phổ thông, HOT
Luận văn: Sáu phương pháp giải các bài toán phổ thông, HOT
Luận văn: Sáu phương pháp giải các bài toán phổ thông, HOT
Luận văn: Sáu phương pháp giải các bài toán phổ thông, HOT
Luận văn: Sáu phương pháp giải các bài toán phổ thông, HOT
Luận văn: Sáu phương pháp giải các bài toán phổ thông, HOT

More Related Content

What's hot

Bài giảng xác suất thống kê
Bài giảng xác suất thống kêBài giảng xác suất thống kê
Bài giảng xác suất thống kêHọc Huỳnh Bá
 
Đề thi trắc nghiệm Xác suất thống kê có lời giải
Đề thi trắc nghiệm Xác suất thống kê có lời giảiĐề thi trắc nghiệm Xác suất thống kê có lời giải
Đề thi trắc nghiệm Xác suất thống kê có lời giải希夢 坂井
 
Hoan vi chinh hop to hop
Hoan vi  chinh hop  to hopHoan vi  chinh hop  to hop
Hoan vi chinh hop to hoptrongphuckhtn
 
Cđ thuật toán tương tự trong bồi dưỡng hsg toán 8
Cđ thuật toán tương tự trong bồi dưỡng hsg toán 8Cđ thuật toán tương tự trong bồi dưỡng hsg toán 8
Cđ thuật toán tương tự trong bồi dưỡng hsg toán 8Cảnh
 
Xac suat. skkn
Xac suat. skknXac suat. skkn
Xac suat. skknbiballi
 
Xstk de thi mau 01 (may12)
Xstk de thi mau 01 (may12)Xstk de thi mau 01 (may12)
Xstk de thi mau 01 (may12)Võ Thùy Linh
 
De cuong-bt-xstk-2015 19-8
De cuong-bt-xstk-2015 19-8De cuong-bt-xstk-2015 19-8
De cuong-bt-xstk-2015 19-8Hoa Pham
 
Luận án tiến sĩ toán học ngưỡng chính tắc của hàm chỉnh hình và hàm đa điều h...
Luận án tiến sĩ toán học ngưỡng chính tắc của hàm chỉnh hình và hàm đa điều h...Luận án tiến sĩ toán học ngưỡng chính tắc của hàm chỉnh hình và hàm đa điều h...
Luận án tiến sĩ toán học ngưỡng chính tắc của hàm chỉnh hình và hàm đa điều h...https://www.facebook.com/garmentspace
 
Chuong2: ƯỚC LƯỢNG THAM SỐ, môn thống kê ứng dụng
Chuong2: ƯỚC LƯỢNG THAM SỐ, môn thống kê ứng dụngChuong2: ƯỚC LƯỢNG THAM SỐ, môn thống kê ứng dụng
Chuong2: ƯỚC LƯỢNG THAM SỐ, môn thống kê ứng dụngThắng Nguyễn
 
Bài tập xác suất thống kê đào hoàng dũng
Bài tập xác suất thống kê   đào hoàng dũngBài tập xác suất thống kê   đào hoàng dũng
Bài tập xác suất thống kê đào hoàng dũngjackjohn45
 
Cac bai toan suy luan tieu hoc
Cac bai toan suy luan tieu hocCac bai toan suy luan tieu hoc
Cac bai toan suy luan tieu hocngoquangbinh
 

What's hot (17)

Luận án: Giải bất đẳng thức biến phân trên tập điểm bất động, HAY
Luận án: Giải bất đẳng thức biến phân trên tập điểm bất động, HAYLuận án: Giải bất đẳng thức biến phân trên tập điểm bất động, HAY
Luận án: Giải bất đẳng thức biến phân trên tập điểm bất động, HAY
 
Bài giảng xác suất thống kê
Bài giảng xác suất thống kêBài giảng xác suất thống kê
Bài giảng xác suất thống kê
 
Luận văn: Giải bài toán bất đẳng thức biến phân giả đơn điệu, 9đ
Luận văn: Giải bài toán bất đẳng thức biến phân giả đơn điệu, 9đLuận văn: Giải bài toán bất đẳng thức biến phân giả đơn điệu, 9đ
Luận văn: Giải bài toán bất đẳng thức biến phân giả đơn điệu, 9đ
 
bai tap co loi giai xac suat thong ke
bai tap co loi giai xac suat thong kebai tap co loi giai xac suat thong ke
bai tap co loi giai xac suat thong ke
 
Đề thi trắc nghiệm Xác suất thống kê có lời giải
Đề thi trắc nghiệm Xác suất thống kê có lời giảiĐề thi trắc nghiệm Xác suất thống kê có lời giải
Đề thi trắc nghiệm Xác suất thống kê có lời giải
 
Hoan vi chinh hop to hop
Hoan vi  chinh hop  to hopHoan vi  chinh hop  to hop
Hoan vi chinh hop to hop
 
Cđ thuật toán tương tự trong bồi dưỡng hsg toán 8
Cđ thuật toán tương tự trong bồi dưỡng hsg toán 8Cđ thuật toán tương tự trong bồi dưỡng hsg toán 8
Cđ thuật toán tương tự trong bồi dưỡng hsg toán 8
 
Xac suat. skkn
Xac suat. skknXac suat. skkn
Xac suat. skkn
 
Xstk de thi mau 01 (may12)
Xstk de thi mau 01 (may12)Xstk de thi mau 01 (may12)
Xstk de thi mau 01 (may12)
 
De cuong-bt-xstk-2015 19-8
De cuong-bt-xstk-2015 19-8De cuong-bt-xstk-2015 19-8
De cuong-bt-xstk-2015 19-8
 
Tìm bao lồi trực giao của đa giác lưới trong mặt phẳng số, HAY
Tìm bao lồi trực giao của đa giác lưới trong mặt phẳng số, HAYTìm bao lồi trực giao của đa giác lưới trong mặt phẳng số, HAY
Tìm bao lồi trực giao của đa giác lưới trong mặt phẳng số, HAY
 
giao trinh xac suat thong ke
giao trinh xac suat thong kegiao trinh xac suat thong ke
giao trinh xac suat thong ke
 
Luận án tiến sĩ toán học ngưỡng chính tắc của hàm chỉnh hình và hàm đa điều h...
Luận án tiến sĩ toán học ngưỡng chính tắc của hàm chỉnh hình và hàm đa điều h...Luận án tiến sĩ toán học ngưỡng chính tắc của hàm chỉnh hình và hàm đa điều h...
Luận án tiến sĩ toán học ngưỡng chính tắc của hàm chỉnh hình và hàm đa điều h...
 
Chuong2: ƯỚC LƯỢNG THAM SỐ, môn thống kê ứng dụng
Chuong2: ƯỚC LƯỢNG THAM SỐ, môn thống kê ứng dụngChuong2: ƯỚC LƯỢNG THAM SỐ, môn thống kê ứng dụng
Chuong2: ƯỚC LƯỢNG THAM SỐ, môn thống kê ứng dụng
 
Bài tập xác suất thống kê đào hoàng dũng
Bài tập xác suất thống kê   đào hoàng dũngBài tập xác suất thống kê   đào hoàng dũng
Bài tập xác suất thống kê đào hoàng dũng
 
322 bai tap xstk
322 bai tap xstk322 bai tap xstk
322 bai tap xstk
 
Cac bai toan suy luan tieu hoc
Cac bai toan suy luan tieu hocCac bai toan suy luan tieu hoc
Cac bai toan suy luan tieu hoc
 

Similar to Luận văn: Sáu phương pháp giải các bài toán phổ thông, HOT

Luận văn: Một số phương pháp giải bài toán không mẫu mực - Gửi miễn phí qua z...
Luận văn: Một số phương pháp giải bài toán không mẫu mực - Gửi miễn phí qua z...Luận văn: Một số phương pháp giải bài toán không mẫu mực - Gửi miễn phí qua z...
Luận văn: Một số phương pháp giải bài toán không mẫu mực - Gửi miễn phí qua z...Dịch vụ viết bài trọn gói ZALO: 0909232620
 
SH_Lien_ND_Dinh ly thang du Trung Hoa_VP_2016_08_16.pdf
SH_Lien_ND_Dinh ly thang du Trung Hoa_VP_2016_08_16.pdfSH_Lien_ND_Dinh ly thang du Trung Hoa_VP_2016_08_16.pdf
SH_Lien_ND_Dinh ly thang du Trung Hoa_VP_2016_08_16.pdfNguyenTanBinh4
 
DE CUONG BAI TAP_Xac-suat-thong-ke&QHTN_MI3180_2020.1.0.pdf
DE CUONG BAI TAP_Xac-suat-thong-ke&QHTN_MI3180_2020.1.0.pdfDE CUONG BAI TAP_Xac-suat-thong-ke&QHTN_MI3180_2020.1.0.pdf
DE CUONG BAI TAP_Xac-suat-thong-ke&QHTN_MI3180_2020.1.0.pdfTrường Việt Nam
 
bo-de-tham-khao-giua-hoc-ky-2-toan-8-nam-2023-2024-phong-gddt-tp-hai-duong.pdf
bo-de-tham-khao-giua-hoc-ky-2-toan-8-nam-2023-2024-phong-gddt-tp-hai-duong.pdfbo-de-tham-khao-giua-hoc-ky-2-toan-8-nam-2023-2024-phong-gddt-tp-hai-duong.pdf
bo-de-tham-khao-giua-hoc-ky-2-toan-8-nam-2023-2024-phong-gddt-tp-hai-duong.pdfLinhTrnTh14
 
Cđ dãy số viết theo quy luật
Cđ dãy số viết theo quy luậtCđ dãy số viết theo quy luật
Cđ dãy số viết theo quy luậtCảnh
 
Giải bài tập XSTK.pdf
Giải bài tập XSTK.pdfGiải bài tập XSTK.pdf
Giải bài tập XSTK.pdfVHongL3
 
Đề tài: Tìm nghiệm của một bài toán bằng cách xem xét tất cả các phương án có...
Đề tài: Tìm nghiệm của một bài toán bằng cách xem xét tất cả các phương án có...Đề tài: Tìm nghiệm của một bài toán bằng cách xem xét tất cả các phương án có...
Đề tài: Tìm nghiệm của một bài toán bằng cách xem xét tất cả các phương án có...Viết thuê trọn gói ZALO 0934573149
 
Một số dạng toán về đa thức qua các kỳ thi Olympic 6732069.pdf
Một số dạng toán về đa thức qua các kỳ thi Olympic 6732069.pdfMột số dạng toán về đa thức qua các kỳ thi Olympic 6732069.pdf
Một số dạng toán về đa thức qua các kỳ thi Olympic 6732069.pdfTieuNgocLy
 
[Bản đọc thử] Sách Tuyệt Đỉnh Luyện Đề Môn Toán 2015 - Megabook.vn
[Bản đọc thử] Sách Tuyệt Đỉnh Luyện Đề Môn Toán 2015 - Megabook.vn [Bản đọc thử] Sách Tuyệt Đỉnh Luyện Đề Môn Toán 2015 - Megabook.vn
[Bản đọc thử] Sách Tuyệt Đỉnh Luyện Đề Môn Toán 2015 - Megabook.vn Megabook
 
ĐẠI SỐ TỔ HỢP
ĐẠI SỐ TỔ HỢPĐẠI SỐ TỔ HỢP
ĐẠI SỐ TỔ HỢPDANAMATH
 

Similar to Luận văn: Sáu phương pháp giải các bài toán phổ thông, HOT (20)

Luận văn: Một số phương pháp giải bài toán không mẫu mực - Gửi miễn phí qua z...
Luận văn: Một số phương pháp giải bài toán không mẫu mực - Gửi miễn phí qua z...Luận văn: Một số phương pháp giải bài toán không mẫu mực - Gửi miễn phí qua z...
Luận văn: Một số phương pháp giải bài toán không mẫu mực - Gửi miễn phí qua z...
 
Hoán vị lặp tổ hợp
Hoán vị lặp tổ hợpHoán vị lặp tổ hợp
Hoán vị lặp tổ hợp
 
SH_Lien_ND_Dinh ly thang du Trung Hoa_VP_2016_08_16.pdf
SH_Lien_ND_Dinh ly thang du Trung Hoa_VP_2016_08_16.pdfSH_Lien_ND_Dinh ly thang du Trung Hoa_VP_2016_08_16.pdf
SH_Lien_ND_Dinh ly thang du Trung Hoa_VP_2016_08_16.pdf
 
Luận văn: Phương pháp giải bài toán cực trị và ứng dụng, HAY
Luận văn: Phương pháp giải bài toán cực trị và ứng dụng, HAYLuận văn: Phương pháp giải bài toán cực trị và ứng dụng, HAY
Luận văn: Phương pháp giải bài toán cực trị và ứng dụng, HAY
 
Luận văn: Phương pháp giải bài toán cực trị, HAY
Luận văn: Phương pháp giải bài toán cực trị, HAYLuận văn: Phương pháp giải bài toán cực trị, HAY
Luận văn: Phương pháp giải bài toán cực trị, HAY
 
Luận văn thạc sĩ: Quy hoạch toàn phương, HAY, 9đ
Luận văn thạc sĩ: Quy hoạch toàn phương, HAY, 9đLuận văn thạc sĩ: Quy hoạch toàn phương, HAY, 9đ
Luận văn thạc sĩ: Quy hoạch toàn phương, HAY, 9đ
 
DE CUONG BAI TAP_Xac-suat-thong-ke&QHTN_MI3180_2020.1.0.pdf
DE CUONG BAI TAP_Xac-suat-thong-ke&QHTN_MI3180_2020.1.0.pdfDE CUONG BAI TAP_Xac-suat-thong-ke&QHTN_MI3180_2020.1.0.pdf
DE CUONG BAI TAP_Xac-suat-thong-ke&QHTN_MI3180_2020.1.0.pdf
 
Luận văn: Giải hệ phương trình trong chương trình toán THPT, HOT
Luận văn: Giải hệ phương trình trong chương trình toán THPT, HOTLuận văn: Giải hệ phương trình trong chương trình toán THPT, HOT
Luận văn: Giải hệ phương trình trong chương trình toán THPT, HOT
 
Luận văn: Phương pháp giải hệ phương trình trong toán THPT
Luận văn: Phương pháp giải hệ phương trình trong toán THPTLuận văn: Phương pháp giải hệ phương trình trong toán THPT
Luận văn: Phương pháp giải hệ phương trình trong toán THPT
 
bo-de-tham-khao-giua-hoc-ky-2-toan-8-nam-2023-2024-phong-gddt-tp-hai-duong.pdf
bo-de-tham-khao-giua-hoc-ky-2-toan-8-nam-2023-2024-phong-gddt-tp-hai-duong.pdfbo-de-tham-khao-giua-hoc-ky-2-toan-8-nam-2023-2024-phong-gddt-tp-hai-duong.pdf
bo-de-tham-khao-giua-hoc-ky-2-toan-8-nam-2023-2024-phong-gddt-tp-hai-duong.pdf
 
Cđ dãy số viết theo quy luật
Cđ dãy số viết theo quy luậtCđ dãy số viết theo quy luật
Cđ dãy số viết theo quy luật
 
Giải bài tập XSTK.pdf
Giải bài tập XSTK.pdfGiải bài tập XSTK.pdf
Giải bài tập XSTK.pdf
 
Luận văn: Các dạng phương trình lượng giác, HOT - Gửi miễn phí qua zalo=> 090...
Luận văn: Các dạng phương trình lượng giác, HOT - Gửi miễn phí qua zalo=> 090...Luận văn: Các dạng phương trình lượng giác, HOT - Gửi miễn phí qua zalo=> 090...
Luận văn: Các dạng phương trình lượng giác, HOT - Gửi miễn phí qua zalo=> 090...
 
Luận văn: Các dạng phương trình lượng giác, HAY, 9đ
Luận văn: Các dạng phương trình lượng giác, HAY, 9đLuận văn: Các dạng phương trình lượng giác, HAY, 9đ
Luận văn: Các dạng phương trình lượng giác, HAY, 9đ
 
Đề tài: Tìm nghiệm của một bài toán bằng cách xem xét tất cả các phương án có...
Đề tài: Tìm nghiệm của một bài toán bằng cách xem xét tất cả các phương án có...Đề tài: Tìm nghiệm của một bài toán bằng cách xem xét tất cả các phương án có...
Đề tài: Tìm nghiệm của một bài toán bằng cách xem xét tất cả các phương án có...
 
Một số dạng toán về đa thức qua các kỳ thi Olympic 6732069.pdf
Một số dạng toán về đa thức qua các kỳ thi Olympic 6732069.pdfMột số dạng toán về đa thức qua các kỳ thi Olympic 6732069.pdf
Một số dạng toán về đa thức qua các kỳ thi Olympic 6732069.pdf
 
[Bản đọc thử] Sách Tuyệt Đỉnh Luyện Đề Môn Toán 2015 - Megabook.vn
[Bản đọc thử] Sách Tuyệt Đỉnh Luyện Đề Môn Toán 2015 - Megabook.vn [Bản đọc thử] Sách Tuyệt Đỉnh Luyện Đề Môn Toán 2015 - Megabook.vn
[Bản đọc thử] Sách Tuyệt Đỉnh Luyện Đề Môn Toán 2015 - Megabook.vn
 
Luận văn: Hồi quy bội tuyến tính và Hồi quy phi tuyến, HOT, 9đ
Luận văn: Hồi quy bội tuyến tính và Hồi quy phi tuyến, HOT, 9đLuận văn: Hồi quy bội tuyến tính và Hồi quy phi tuyến, HOT, 9đ
Luận văn: Hồi quy bội tuyến tính và Hồi quy phi tuyến, HOT, 9đ
 
ĐẠI SỐ TỔ HỢP
ĐẠI SỐ TỔ HỢPĐẠI SỐ TỔ HỢP
ĐẠI SỐ TỔ HỢP
 
Thpt hoang le kha-MVN 2
Thpt hoang le kha-MVN 2Thpt hoang le kha-MVN 2
Thpt hoang le kha-MVN 2
 

More from Dịch Vụ Viết Bài Trọn Gói ZALO 0917193864

Quản Lý Hoạt Động Dạy Học Các Môn Khoa Học Tự Nhiên Theo Chuẩn Kiến Thức Và K...
Quản Lý Hoạt Động Dạy Học Các Môn Khoa Học Tự Nhiên Theo Chuẩn Kiến Thức Và K...Quản Lý Hoạt Động Dạy Học Các Môn Khoa Học Tự Nhiên Theo Chuẩn Kiến Thức Và K...
Quản Lý Hoạt Động Dạy Học Các Môn Khoa Học Tự Nhiên Theo Chuẩn Kiến Thức Và K...Dịch Vụ Viết Bài Trọn Gói ZALO 0917193864
 
Quản Lý Thu Thuế Giá Trị Gia Tăng Đối Với Doanh Nghiệp Ngoài Quốc Doanh Trên ...
Quản Lý Thu Thuế Giá Trị Gia Tăng Đối Với Doanh Nghiệp Ngoài Quốc Doanh Trên ...Quản Lý Thu Thuế Giá Trị Gia Tăng Đối Với Doanh Nghiệp Ngoài Quốc Doanh Trên ...
Quản Lý Thu Thuế Giá Trị Gia Tăng Đối Với Doanh Nghiệp Ngoài Quốc Doanh Trên ...Dịch Vụ Viết Bài Trọn Gói ZALO 0917193864
 
Thu Hút Nguồn Nhân Lực Trình Độ Cao Vào Các Cơ Quan Hành Chính Nhà Nước Tỉnh ...
Thu Hút Nguồn Nhân Lực Trình Độ Cao Vào Các Cơ Quan Hành Chính Nhà Nước Tỉnh ...Thu Hút Nguồn Nhân Lực Trình Độ Cao Vào Các Cơ Quan Hành Chính Nhà Nước Tỉnh ...
Thu Hút Nguồn Nhân Lực Trình Độ Cao Vào Các Cơ Quan Hành Chính Nhà Nước Tỉnh ...Dịch Vụ Viết Bài Trọn Gói ZALO 0917193864
 
Quản Trị Rủi Ro Tín Dụng Trong Cho Vay Doanh Nghiệp Tại Ngân Hàng Thương Mại ...
Quản Trị Rủi Ro Tín Dụng Trong Cho Vay Doanh Nghiệp Tại Ngân Hàng Thương Mại ...Quản Trị Rủi Ro Tín Dụng Trong Cho Vay Doanh Nghiệp Tại Ngân Hàng Thương Mại ...
Quản Trị Rủi Ro Tín Dụng Trong Cho Vay Doanh Nghiệp Tại Ngân Hàng Thương Mại ...Dịch Vụ Viết Bài Trọn Gói ZALO 0917193864
 
Quản Lý Hoạt Động Dạy Học Các Trường Thpt Trên Địa Bàn Huyện Sơn Hà Tỉnh Quản...
Quản Lý Hoạt Động Dạy Học Các Trường Thpt Trên Địa Bàn Huyện Sơn Hà Tỉnh Quản...Quản Lý Hoạt Động Dạy Học Các Trường Thpt Trên Địa Bàn Huyện Sơn Hà Tỉnh Quản...
Quản Lý Hoạt Động Dạy Học Các Trường Thpt Trên Địa Bàn Huyện Sơn Hà Tỉnh Quản...Dịch Vụ Viết Bài Trọn Gói ZALO 0917193864
 
Quản Trị Rủi Ro Tín Dụng Trong Cho Vay Ngắn Hạn Tại Ngân Hàng Công Thƣơng Chi...
Quản Trị Rủi Ro Tín Dụng Trong Cho Vay Ngắn Hạn Tại Ngân Hàng Công Thƣơng Chi...Quản Trị Rủi Ro Tín Dụng Trong Cho Vay Ngắn Hạn Tại Ngân Hàng Công Thƣơng Chi...
Quản Trị Rủi Ro Tín Dụng Trong Cho Vay Ngắn Hạn Tại Ngân Hàng Công Thƣơng Chi...Dịch Vụ Viết Bài Trọn Gói ZALO 0917193864
 
Quản Lý Nhà Nước Về Nuôi Trồng Thủy Sản Nước Ngọt Trên Địa Bàn Thành Phố Hải ...
Quản Lý Nhà Nước Về Nuôi Trồng Thủy Sản Nước Ngọt Trên Địa Bàn Thành Phố Hải ...Quản Lý Nhà Nước Về Nuôi Trồng Thủy Sản Nước Ngọt Trên Địa Bàn Thành Phố Hải ...
Quản Lý Nhà Nước Về Nuôi Trồng Thủy Sản Nước Ngọt Trên Địa Bàn Thành Phố Hải ...Dịch Vụ Viết Bài Trọn Gói ZALO 0917193864
 
Quản Lý Hoạt Động Giáo Dục Ngoài Giờ Lên Lớp Ở Các Trường Thcs Huyện Chư Păh ...
Quản Lý Hoạt Động Giáo Dục Ngoài Giờ Lên Lớp Ở Các Trường Thcs Huyện Chư Păh ...Quản Lý Hoạt Động Giáo Dục Ngoài Giờ Lên Lớp Ở Các Trường Thcs Huyện Chư Păh ...
Quản Lý Hoạt Động Giáo Dục Ngoài Giờ Lên Lớp Ở Các Trường Thcs Huyện Chư Păh ...Dịch Vụ Viết Bài Trọn Gói ZALO 0917193864
 
Quản Lý Hoạt Động Dạy Học Ngoại Ngữ Tại Các Trung Tâm Ngoại Ngữ - Tin Học Trê...
Quản Lý Hoạt Động Dạy Học Ngoại Ngữ Tại Các Trung Tâm Ngoại Ngữ - Tin Học Trê...Quản Lý Hoạt Động Dạy Học Ngoại Ngữ Tại Các Trung Tâm Ngoại Ngữ - Tin Học Trê...
Quản Lý Hoạt Động Dạy Học Ngoại Ngữ Tại Các Trung Tâm Ngoại Ngữ - Tin Học Trê...Dịch Vụ Viết Bài Trọn Gói ZALO 0917193864
 
Quản Trị Rủi Ro Tín Dụng Trong Cho Vay Doanh Nghiệp Tại Ngân Hàng Thƣơng Mại ...
Quản Trị Rủi Ro Tín Dụng Trong Cho Vay Doanh Nghiệp Tại Ngân Hàng Thƣơng Mại ...Quản Trị Rủi Ro Tín Dụng Trong Cho Vay Doanh Nghiệp Tại Ngân Hàng Thƣơng Mại ...
Quản Trị Rủi Ro Tín Dụng Trong Cho Vay Doanh Nghiệp Tại Ngân Hàng Thƣơng Mại ...Dịch Vụ Viết Bài Trọn Gói ZALO 0917193864
 
Tạo Việc Làm Cho Thanh Niên Trên Địa Bàn Quận Thanh Khê, Thành Phố Đà Nẵng.doc
Tạo Việc Làm Cho Thanh Niên Trên Địa Bàn Quận Thanh Khê, Thành Phố Đà Nẵng.docTạo Việc Làm Cho Thanh Niên Trên Địa Bàn Quận Thanh Khê, Thành Phố Đà Nẵng.doc
Tạo Việc Làm Cho Thanh Niên Trên Địa Bàn Quận Thanh Khê, Thành Phố Đà Nẵng.docDịch Vụ Viết Bài Trọn Gói ZALO 0917193864
 
Quản Trị Rủi Ro Tín Dụng Trong Cho Vay Trung Và Dài Hạn Tại Ngân Hàng Thương ...
Quản Trị Rủi Ro Tín Dụng Trong Cho Vay Trung Và Dài Hạn Tại Ngân Hàng Thương ...Quản Trị Rủi Ro Tín Dụng Trong Cho Vay Trung Và Dài Hạn Tại Ngân Hàng Thương ...
Quản Trị Rủi Ro Tín Dụng Trong Cho Vay Trung Và Dài Hạn Tại Ngân Hàng Thương ...Dịch Vụ Viết Bài Trọn Gói ZALO 0917193864
 

More from Dịch Vụ Viết Bài Trọn Gói ZALO 0917193864 (20)

Yếu Tố Tự Truyện Trong Truyện Ngắn Thạch Lam Và Thanh Tịnh.doc
Yếu Tố Tự Truyện Trong Truyện Ngắn Thạch Lam Và Thanh Tịnh.docYếu Tố Tự Truyện Trong Truyện Ngắn Thạch Lam Và Thanh Tịnh.doc
Yếu Tố Tự Truyện Trong Truyện Ngắn Thạch Lam Và Thanh Tịnh.doc
 
Từ Ngữ Biểu Thị Tâm Lí – Tình Cảm Trong Ca Dao Người Việt.doc
Từ Ngữ Biểu Thị Tâm Lí – Tình Cảm Trong Ca Dao Người Việt.docTừ Ngữ Biểu Thị Tâm Lí – Tình Cảm Trong Ca Dao Người Việt.doc
Từ Ngữ Biểu Thị Tâm Lí – Tình Cảm Trong Ca Dao Người Việt.doc
 
Quản Lý Hoạt Động Dạy Học Các Môn Khoa Học Tự Nhiên Theo Chuẩn Kiến Thức Và K...
Quản Lý Hoạt Động Dạy Học Các Môn Khoa Học Tự Nhiên Theo Chuẩn Kiến Thức Và K...Quản Lý Hoạt Động Dạy Học Các Môn Khoa Học Tự Nhiên Theo Chuẩn Kiến Thức Và K...
Quản Lý Hoạt Động Dạy Học Các Môn Khoa Học Tự Nhiên Theo Chuẩn Kiến Thức Và K...
 
Quản Lý Thu Thuế Giá Trị Gia Tăng Đối Với Doanh Nghiệp Ngoài Quốc Doanh Trên ...
Quản Lý Thu Thuế Giá Trị Gia Tăng Đối Với Doanh Nghiệp Ngoài Quốc Doanh Trên ...Quản Lý Thu Thuế Giá Trị Gia Tăng Đối Với Doanh Nghiệp Ngoài Quốc Doanh Trên ...
Quản Lý Thu Thuế Giá Trị Gia Tăng Đối Với Doanh Nghiệp Ngoài Quốc Doanh Trên ...
 
Thu Hút Nguồn Nhân Lực Trình Độ Cao Vào Các Cơ Quan Hành Chính Nhà Nước Tỉnh ...
Thu Hút Nguồn Nhân Lực Trình Độ Cao Vào Các Cơ Quan Hành Chính Nhà Nước Tỉnh ...Thu Hút Nguồn Nhân Lực Trình Độ Cao Vào Các Cơ Quan Hành Chính Nhà Nước Tỉnh ...
Thu Hút Nguồn Nhân Lực Trình Độ Cao Vào Các Cơ Quan Hành Chính Nhà Nước Tỉnh ...
 
Quản Trị Rủi Ro Tín Dụng Trong Cho Vay Doanh Nghiệp Tại Ngân Hàng Thương Mại ...
Quản Trị Rủi Ro Tín Dụng Trong Cho Vay Doanh Nghiệp Tại Ngân Hàng Thương Mại ...Quản Trị Rủi Ro Tín Dụng Trong Cho Vay Doanh Nghiệp Tại Ngân Hàng Thương Mại ...
Quản Trị Rủi Ro Tín Dụng Trong Cho Vay Doanh Nghiệp Tại Ngân Hàng Thương Mại ...
 
Vaporisation Of Single And Binary Component Droplets In Heated Flowing Gas St...
Vaporisation Of Single And Binary Component Droplets In Heated Flowing Gas St...Vaporisation Of Single And Binary Component Droplets In Heated Flowing Gas St...
Vaporisation Of Single And Binary Component Droplets In Heated Flowing Gas St...
 
Quản Lý Hoạt Động Dạy Học Các Trường Thpt Trên Địa Bàn Huyện Sơn Hà Tỉnh Quản...
Quản Lý Hoạt Động Dạy Học Các Trường Thpt Trên Địa Bàn Huyện Sơn Hà Tỉnh Quản...Quản Lý Hoạt Động Dạy Học Các Trường Thpt Trên Địa Bàn Huyện Sơn Hà Tỉnh Quản...
Quản Lý Hoạt Động Dạy Học Các Trường Thpt Trên Địa Bàn Huyện Sơn Hà Tỉnh Quản...
 
Tác Giả Hàm Ẩn Trong Tiểu Thuyết Nguyễn Việt Hà.doc
Tác Giả Hàm Ẩn Trong Tiểu Thuyết Nguyễn Việt Hà.docTác Giả Hàm Ẩn Trong Tiểu Thuyết Nguyễn Việt Hà.doc
Tác Giả Hàm Ẩn Trong Tiểu Thuyết Nguyễn Việt Hà.doc
 
Quản Trị Rủi Ro Tín Dụng Trong Cho Vay Ngắn Hạn Tại Ngân Hàng Công Thƣơng Chi...
Quản Trị Rủi Ro Tín Dụng Trong Cho Vay Ngắn Hạn Tại Ngân Hàng Công Thƣơng Chi...Quản Trị Rủi Ro Tín Dụng Trong Cho Vay Ngắn Hạn Tại Ngân Hàng Công Thƣơng Chi...
Quản Trị Rủi Ro Tín Dụng Trong Cho Vay Ngắn Hạn Tại Ngân Hàng Công Thƣơng Chi...
 
Quản Lý Nhà Nước Về Nuôi Trồng Thủy Sản Nước Ngọt Trên Địa Bàn Thành Phố Hải ...
Quản Lý Nhà Nước Về Nuôi Trồng Thủy Sản Nước Ngọt Trên Địa Bàn Thành Phố Hải ...Quản Lý Nhà Nước Về Nuôi Trồng Thủy Sản Nước Ngọt Trên Địa Bàn Thành Phố Hải ...
Quản Lý Nhà Nước Về Nuôi Trồng Thủy Sản Nước Ngọt Trên Địa Bàn Thành Phố Hải ...
 
Song Song Hóa Các Thuật Toán Trên Mạng Đồ Thị.doc
Song Song Hóa Các Thuật Toán Trên Mạng Đồ Thị.docSong Song Hóa Các Thuật Toán Trên Mạng Đồ Thị.doc
Song Song Hóa Các Thuật Toán Trên Mạng Đồ Thị.doc
 
Ứng Dụng Số Phức Trong Các Bài Toán Sơ Cấp.doc
Ứng Dụng Số Phức Trong Các Bài Toán Sơ Cấp.docỨng Dụng Số Phức Trong Các Bài Toán Sơ Cấp.doc
Ứng Dụng Số Phức Trong Các Bài Toán Sơ Cấp.doc
 
Vai Trò Của Cái Bi Trong Giáo Dục Thẩm Mỹ.doc
Vai Trò Của Cái Bi Trong Giáo Dục Thẩm Mỹ.docVai Trò Của Cái Bi Trong Giáo Dục Thẩm Mỹ.doc
Vai Trò Của Cái Bi Trong Giáo Dục Thẩm Mỹ.doc
 
Quản Lý Hoạt Động Giáo Dục Ngoài Giờ Lên Lớp Ở Các Trường Thcs Huyện Chư Păh ...
Quản Lý Hoạt Động Giáo Dục Ngoài Giờ Lên Lớp Ở Các Trường Thcs Huyện Chư Păh ...Quản Lý Hoạt Động Giáo Dục Ngoài Giờ Lên Lớp Ở Các Trường Thcs Huyện Chư Păh ...
Quản Lý Hoạt Động Giáo Dục Ngoài Giờ Lên Lớp Ở Các Trường Thcs Huyện Chư Păh ...
 
Thu Hút Vốn Đầu Tư Vào Lĩnh Vực Nông Nghiệp Trên Địa Bàn Tỉnh Gia Lai.doc
Thu Hút Vốn Đầu Tư Vào Lĩnh Vực Nông Nghiệp Trên Địa Bàn Tỉnh Gia Lai.docThu Hút Vốn Đầu Tư Vào Lĩnh Vực Nông Nghiệp Trên Địa Bàn Tỉnh Gia Lai.doc
Thu Hút Vốn Đầu Tư Vào Lĩnh Vực Nông Nghiệp Trên Địa Bàn Tỉnh Gia Lai.doc
 
Quản Lý Hoạt Động Dạy Học Ngoại Ngữ Tại Các Trung Tâm Ngoại Ngữ - Tin Học Trê...
Quản Lý Hoạt Động Dạy Học Ngoại Ngữ Tại Các Trung Tâm Ngoại Ngữ - Tin Học Trê...Quản Lý Hoạt Động Dạy Học Ngoại Ngữ Tại Các Trung Tâm Ngoại Ngữ - Tin Học Trê...
Quản Lý Hoạt Động Dạy Học Ngoại Ngữ Tại Các Trung Tâm Ngoại Ngữ - Tin Học Trê...
 
Quản Trị Rủi Ro Tín Dụng Trong Cho Vay Doanh Nghiệp Tại Ngân Hàng Thƣơng Mại ...
Quản Trị Rủi Ro Tín Dụng Trong Cho Vay Doanh Nghiệp Tại Ngân Hàng Thƣơng Mại ...Quản Trị Rủi Ro Tín Dụng Trong Cho Vay Doanh Nghiệp Tại Ngân Hàng Thƣơng Mại ...
Quản Trị Rủi Ro Tín Dụng Trong Cho Vay Doanh Nghiệp Tại Ngân Hàng Thƣơng Mại ...
 
Tạo Việc Làm Cho Thanh Niên Trên Địa Bàn Quận Thanh Khê, Thành Phố Đà Nẵng.doc
Tạo Việc Làm Cho Thanh Niên Trên Địa Bàn Quận Thanh Khê, Thành Phố Đà Nẵng.docTạo Việc Làm Cho Thanh Niên Trên Địa Bàn Quận Thanh Khê, Thành Phố Đà Nẵng.doc
Tạo Việc Làm Cho Thanh Niên Trên Địa Bàn Quận Thanh Khê, Thành Phố Đà Nẵng.doc
 
Quản Trị Rủi Ro Tín Dụng Trong Cho Vay Trung Và Dài Hạn Tại Ngân Hàng Thương ...
Quản Trị Rủi Ro Tín Dụng Trong Cho Vay Trung Và Dài Hạn Tại Ngân Hàng Thương ...Quản Trị Rủi Ro Tín Dụng Trong Cho Vay Trung Và Dài Hạn Tại Ngân Hàng Thương ...
Quản Trị Rủi Ro Tín Dụng Trong Cho Vay Trung Và Dài Hạn Tại Ngân Hàng Thương ...
 

Recently uploaded

30 ĐỀ PHÁT TRIỂN THEO CẤU TRÚC ĐỀ MINH HỌA BGD NGÀY 22-3-2024 KỲ THI TỐT NGHI...
30 ĐỀ PHÁT TRIỂN THEO CẤU TRÚC ĐỀ MINH HỌA BGD NGÀY 22-3-2024 KỲ THI TỐT NGHI...30 ĐỀ PHÁT TRIỂN THEO CẤU TRÚC ĐỀ MINH HỌA BGD NGÀY 22-3-2024 KỲ THI TỐT NGHI...
30 ĐỀ PHÁT TRIỂN THEO CẤU TRÚC ĐỀ MINH HỌA BGD NGÀY 22-3-2024 KỲ THI TỐT NGHI...Nguyen Thanh Tu Collection
 
Sáng kiến “Sử dụng ứng dụng Quizizz nhằm nâng cao chất lượng ôn thi tốt nghiệ...
Sáng kiến “Sử dụng ứng dụng Quizizz nhằm nâng cao chất lượng ôn thi tốt nghiệ...Sáng kiến “Sử dụng ứng dụng Quizizz nhằm nâng cao chất lượng ôn thi tốt nghiệ...
Sáng kiến “Sử dụng ứng dụng Quizizz nhằm nâng cao chất lượng ôn thi tốt nghiệ...Nguyen Thanh Tu Collection
 
ôn tập lịch sử hhhhhhhhhhhhhhhhhhhhhhhhhh
ôn tập lịch sử hhhhhhhhhhhhhhhhhhhhhhhhhhôn tập lịch sử hhhhhhhhhhhhhhhhhhhhhhhhhh
ôn tập lịch sử hhhhhhhhhhhhhhhhhhhhhhhhhhvanhathvc
 
Trích dẫn trắc nghiệm tư tưởng HCM5.docx
Trích dẫn trắc nghiệm tư tưởng HCM5.docxTrích dẫn trắc nghiệm tư tưởng HCM5.docx
Trích dẫn trắc nghiệm tư tưởng HCM5.docxnhungdt08102004
 
NQA Lợi ích Từ ISO và ESG Tăng Trưởng và Bền Vững ver01.pdf
NQA Lợi ích Từ ISO và ESG Tăng Trưởng và Bền Vững ver01.pdfNQA Lợi ích Từ ISO và ESG Tăng Trưởng và Bền Vững ver01.pdf
NQA Lợi ích Từ ISO và ESG Tăng Trưởng và Bền Vững ver01.pdfNguyễn Đăng Quang
 
QUẢN LÝ HOẠT ĐỘNG GIÁO DỤC KỸ NĂNG SỐNG CHO HỌC SINH CÁC TRƯỜNG TRUNG HỌC CƠ ...
QUẢN LÝ HOẠT ĐỘNG GIÁO DỤC KỸ NĂNG SỐNG CHO HỌC SINH CÁC TRƯỜNG TRUNG HỌC CƠ ...QUẢN LÝ HOẠT ĐỘNG GIÁO DỤC KỸ NĂNG SỐNG CHO HỌC SINH CÁC TRƯỜNG TRUNG HỌC CƠ ...
QUẢN LÝ HOẠT ĐỘNG GIÁO DỤC KỸ NĂNG SỐNG CHO HỌC SINH CÁC TRƯỜNG TRUNG HỌC CƠ ...ThunTrn734461
 
Thong bao 337-DHPY (24.4.2024) thi sat hach Ngoai ngu dap ung Chuan dau ra do...
Thong bao 337-DHPY (24.4.2024) thi sat hach Ngoai ngu dap ung Chuan dau ra do...Thong bao 337-DHPY (24.4.2024) thi sat hach Ngoai ngu dap ung Chuan dau ra do...
Thong bao 337-DHPY (24.4.2024) thi sat hach Ngoai ngu dap ung Chuan dau ra do...hoangtuansinh1
 
30 ĐỀ PHÁT TRIỂN THEO CẤU TRÚC ĐỀ MINH HỌA BGD NGÀY 22-3-2024 KỲ THI TỐT NGHI...
30 ĐỀ PHÁT TRIỂN THEO CẤU TRÚC ĐỀ MINH HỌA BGD NGÀY 22-3-2024 KỲ THI TỐT NGHI...30 ĐỀ PHÁT TRIỂN THEO CẤU TRÚC ĐỀ MINH HỌA BGD NGÀY 22-3-2024 KỲ THI TỐT NGHI...
30 ĐỀ PHÁT TRIỂN THEO CẤU TRÚC ĐỀ MINH HỌA BGD NGÀY 22-3-2024 KỲ THI TỐT NGHI...Nguyen Thanh Tu Collection
 
chuong-7-van-de-gia-dinh-trong-thoi-ky-qua-do-len-cnxh.pdf
chuong-7-van-de-gia-dinh-trong-thoi-ky-qua-do-len-cnxh.pdfchuong-7-van-de-gia-dinh-trong-thoi-ky-qua-do-len-cnxh.pdf
chuong-7-van-de-gia-dinh-trong-thoi-ky-qua-do-len-cnxh.pdfVyTng986513
 
GIÁO TRÌNH KHỐI NGUỒN CÁC LOẠI - ĐIỆN LẠNH BÁCH KHOA HÀ NỘI
GIÁO TRÌNH  KHỐI NGUỒN CÁC LOẠI - ĐIỆN LẠNH BÁCH KHOA HÀ NỘIGIÁO TRÌNH  KHỐI NGUỒN CÁC LOẠI - ĐIỆN LẠNH BÁCH KHOA HÀ NỘI
GIÁO TRÌNH KHỐI NGUỒN CÁC LOẠI - ĐIỆN LẠNH BÁCH KHOA HÀ NỘIĐiện Lạnh Bách Khoa Hà Nội
 
30 ĐỀ PHÁT TRIỂN THEO CẤU TRÚC ĐỀ MINH HỌA BGD NGÀY 22-3-2024 KỲ THI TỐT NGHI...
30 ĐỀ PHÁT TRIỂN THEO CẤU TRÚC ĐỀ MINH HỌA BGD NGÀY 22-3-2024 KỲ THI TỐT NGHI...30 ĐỀ PHÁT TRIỂN THEO CẤU TRÚC ĐỀ MINH HỌA BGD NGÀY 22-3-2024 KỲ THI TỐT NGHI...
30 ĐỀ PHÁT TRIỂN THEO CẤU TRÚC ĐỀ MINH HỌA BGD NGÀY 22-3-2024 KỲ THI TỐT NGHI...Nguyen Thanh Tu Collection
 
Kiểm tra chạy trạm lí thuyết giữa kì giải phẫu sinh lí
Kiểm tra chạy trạm lí thuyết giữa kì giải phẫu sinh líKiểm tra chạy trạm lí thuyết giữa kì giải phẫu sinh lí
Kiểm tra chạy trạm lí thuyết giữa kì giải phẫu sinh líDr K-OGN
 
Chuong trinh dao tao Su pham Khoa hoc tu nhien, ma nganh - 7140247.pdf
Chuong trinh dao tao Su pham Khoa hoc tu nhien, ma nganh - 7140247.pdfChuong trinh dao tao Su pham Khoa hoc tu nhien, ma nganh - 7140247.pdf
Chuong trinh dao tao Su pham Khoa hoc tu nhien, ma nganh - 7140247.pdfhoangtuansinh1
 
30 ĐỀ PHÁT TRIỂN THEO CẤU TRÚC ĐỀ MINH HỌA BGD NGÀY 22-3-2024 KỲ THI TỐT NGHI...
30 ĐỀ PHÁT TRIỂN THEO CẤU TRÚC ĐỀ MINH HỌA BGD NGÀY 22-3-2024 KỲ THI TỐT NGHI...30 ĐỀ PHÁT TRIỂN THEO CẤU TRÚC ĐỀ MINH HỌA BGD NGÀY 22-3-2024 KỲ THI TỐT NGHI...
30 ĐỀ PHÁT TRIỂN THEO CẤU TRÚC ĐỀ MINH HỌA BGD NGÀY 22-3-2024 KỲ THI TỐT NGHI...Nguyen Thanh Tu Collection
 
Sơ đồ tư duy môn sinh học bậc THPT.pdf
Sơ đồ tư duy môn sinh học bậc THPT.pdfSơ đồ tư duy môn sinh học bậc THPT.pdf
Sơ đồ tư duy môn sinh học bậc THPT.pdftohoanggiabao81
 
Sáng kiến Dạy học theo định hướng STEM một số chủ đề phần “vật sống”, Khoa họ...
Sáng kiến Dạy học theo định hướng STEM một số chủ đề phần “vật sống”, Khoa họ...Sáng kiến Dạy học theo định hướng STEM một số chủ đề phần “vật sống”, Khoa họ...
Sáng kiến Dạy học theo định hướng STEM một số chủ đề phần “vật sống”, Khoa họ...Nguyen Thanh Tu Collection
 
sách sinh học đại cương - Textbook.pdf
sách sinh học đại cương   -   Textbook.pdfsách sinh học đại cương   -   Textbook.pdf
sách sinh học đại cương - Textbook.pdfTrnHoa46
 
Đề cương môn giải phẫu......................
Đề cương môn giải phẫu......................Đề cương môn giải phẫu......................
Đề cương môn giải phẫu......................TrnHoa46
 
TỔNG HỢP ĐỀ THI CHÍNH THỨC KỲ THI TUYỂN SINH VÀO LỚP 10 THPT MÔN NGỮ VĂN NĂM ...
TỔNG HỢP ĐỀ THI CHÍNH THỨC KỲ THI TUYỂN SINH VÀO LỚP 10 THPT MÔN NGỮ VĂN NĂM ...TỔNG HỢP ĐỀ THI CHÍNH THỨC KỲ THI TUYỂN SINH VÀO LỚP 10 THPT MÔN NGỮ VĂN NĂM ...
TỔNG HỢP ĐỀ THI CHÍNH THỨC KỲ THI TUYỂN SINH VÀO LỚP 10 THPT MÔN NGỮ VĂN NĂM ...Nguyen Thanh Tu Collection
 
Chàm - Bệnh án (da liễu - bvdlct ctump) .pptx
Chàm - Bệnh án (da liễu - bvdlct ctump) .pptxChàm - Bệnh án (da liễu - bvdlct ctump) .pptx
Chàm - Bệnh án (da liễu - bvdlct ctump) .pptxendkay31
 

Recently uploaded (20)

30 ĐỀ PHÁT TRIỂN THEO CẤU TRÚC ĐỀ MINH HỌA BGD NGÀY 22-3-2024 KỲ THI TỐT NGHI...
30 ĐỀ PHÁT TRIỂN THEO CẤU TRÚC ĐỀ MINH HỌA BGD NGÀY 22-3-2024 KỲ THI TỐT NGHI...30 ĐỀ PHÁT TRIỂN THEO CẤU TRÚC ĐỀ MINH HỌA BGD NGÀY 22-3-2024 KỲ THI TỐT NGHI...
30 ĐỀ PHÁT TRIỂN THEO CẤU TRÚC ĐỀ MINH HỌA BGD NGÀY 22-3-2024 KỲ THI TỐT NGHI...
 
Sáng kiến “Sử dụng ứng dụng Quizizz nhằm nâng cao chất lượng ôn thi tốt nghiệ...
Sáng kiến “Sử dụng ứng dụng Quizizz nhằm nâng cao chất lượng ôn thi tốt nghiệ...Sáng kiến “Sử dụng ứng dụng Quizizz nhằm nâng cao chất lượng ôn thi tốt nghiệ...
Sáng kiến “Sử dụng ứng dụng Quizizz nhằm nâng cao chất lượng ôn thi tốt nghiệ...
 
ôn tập lịch sử hhhhhhhhhhhhhhhhhhhhhhhhhh
ôn tập lịch sử hhhhhhhhhhhhhhhhhhhhhhhhhhôn tập lịch sử hhhhhhhhhhhhhhhhhhhhhhhhhh
ôn tập lịch sử hhhhhhhhhhhhhhhhhhhhhhhhhh
 
Trích dẫn trắc nghiệm tư tưởng HCM5.docx
Trích dẫn trắc nghiệm tư tưởng HCM5.docxTrích dẫn trắc nghiệm tư tưởng HCM5.docx
Trích dẫn trắc nghiệm tư tưởng HCM5.docx
 
NQA Lợi ích Từ ISO và ESG Tăng Trưởng và Bền Vững ver01.pdf
NQA Lợi ích Từ ISO và ESG Tăng Trưởng và Bền Vững ver01.pdfNQA Lợi ích Từ ISO và ESG Tăng Trưởng và Bền Vững ver01.pdf
NQA Lợi ích Từ ISO và ESG Tăng Trưởng và Bền Vững ver01.pdf
 
QUẢN LÝ HOẠT ĐỘNG GIÁO DỤC KỸ NĂNG SỐNG CHO HỌC SINH CÁC TRƯỜNG TRUNG HỌC CƠ ...
QUẢN LÝ HOẠT ĐỘNG GIÁO DỤC KỸ NĂNG SỐNG CHO HỌC SINH CÁC TRƯỜNG TRUNG HỌC CƠ ...QUẢN LÝ HOẠT ĐỘNG GIÁO DỤC KỸ NĂNG SỐNG CHO HỌC SINH CÁC TRƯỜNG TRUNG HỌC CƠ ...
QUẢN LÝ HOẠT ĐỘNG GIÁO DỤC KỸ NĂNG SỐNG CHO HỌC SINH CÁC TRƯỜNG TRUNG HỌC CƠ ...
 
Thong bao 337-DHPY (24.4.2024) thi sat hach Ngoai ngu dap ung Chuan dau ra do...
Thong bao 337-DHPY (24.4.2024) thi sat hach Ngoai ngu dap ung Chuan dau ra do...Thong bao 337-DHPY (24.4.2024) thi sat hach Ngoai ngu dap ung Chuan dau ra do...
Thong bao 337-DHPY (24.4.2024) thi sat hach Ngoai ngu dap ung Chuan dau ra do...
 
30 ĐỀ PHÁT TRIỂN THEO CẤU TRÚC ĐỀ MINH HỌA BGD NGÀY 22-3-2024 KỲ THI TỐT NGHI...
30 ĐỀ PHÁT TRIỂN THEO CẤU TRÚC ĐỀ MINH HỌA BGD NGÀY 22-3-2024 KỲ THI TỐT NGHI...30 ĐỀ PHÁT TRIỂN THEO CẤU TRÚC ĐỀ MINH HỌA BGD NGÀY 22-3-2024 KỲ THI TỐT NGHI...
30 ĐỀ PHÁT TRIỂN THEO CẤU TRÚC ĐỀ MINH HỌA BGD NGÀY 22-3-2024 KỲ THI TỐT NGHI...
 
chuong-7-van-de-gia-dinh-trong-thoi-ky-qua-do-len-cnxh.pdf
chuong-7-van-de-gia-dinh-trong-thoi-ky-qua-do-len-cnxh.pdfchuong-7-van-de-gia-dinh-trong-thoi-ky-qua-do-len-cnxh.pdf
chuong-7-van-de-gia-dinh-trong-thoi-ky-qua-do-len-cnxh.pdf
 
GIÁO TRÌNH KHỐI NGUỒN CÁC LOẠI - ĐIỆN LẠNH BÁCH KHOA HÀ NỘI
GIÁO TRÌNH  KHỐI NGUỒN CÁC LOẠI - ĐIỆN LẠNH BÁCH KHOA HÀ NỘIGIÁO TRÌNH  KHỐI NGUỒN CÁC LOẠI - ĐIỆN LẠNH BÁCH KHOA HÀ NỘI
GIÁO TRÌNH KHỐI NGUỒN CÁC LOẠI - ĐIỆN LẠNH BÁCH KHOA HÀ NỘI
 
30 ĐỀ PHÁT TRIỂN THEO CẤU TRÚC ĐỀ MINH HỌA BGD NGÀY 22-3-2024 KỲ THI TỐT NGHI...
30 ĐỀ PHÁT TRIỂN THEO CẤU TRÚC ĐỀ MINH HỌA BGD NGÀY 22-3-2024 KỲ THI TỐT NGHI...30 ĐỀ PHÁT TRIỂN THEO CẤU TRÚC ĐỀ MINH HỌA BGD NGÀY 22-3-2024 KỲ THI TỐT NGHI...
30 ĐỀ PHÁT TRIỂN THEO CẤU TRÚC ĐỀ MINH HỌA BGD NGÀY 22-3-2024 KỲ THI TỐT NGHI...
 
Kiểm tra chạy trạm lí thuyết giữa kì giải phẫu sinh lí
Kiểm tra chạy trạm lí thuyết giữa kì giải phẫu sinh líKiểm tra chạy trạm lí thuyết giữa kì giải phẫu sinh lí
Kiểm tra chạy trạm lí thuyết giữa kì giải phẫu sinh lí
 
Chuong trinh dao tao Su pham Khoa hoc tu nhien, ma nganh - 7140247.pdf
Chuong trinh dao tao Su pham Khoa hoc tu nhien, ma nganh - 7140247.pdfChuong trinh dao tao Su pham Khoa hoc tu nhien, ma nganh - 7140247.pdf
Chuong trinh dao tao Su pham Khoa hoc tu nhien, ma nganh - 7140247.pdf
 
30 ĐỀ PHÁT TRIỂN THEO CẤU TRÚC ĐỀ MINH HỌA BGD NGÀY 22-3-2024 KỲ THI TỐT NGHI...
30 ĐỀ PHÁT TRIỂN THEO CẤU TRÚC ĐỀ MINH HỌA BGD NGÀY 22-3-2024 KỲ THI TỐT NGHI...30 ĐỀ PHÁT TRIỂN THEO CẤU TRÚC ĐỀ MINH HỌA BGD NGÀY 22-3-2024 KỲ THI TỐT NGHI...
30 ĐỀ PHÁT TRIỂN THEO CẤU TRÚC ĐỀ MINH HỌA BGD NGÀY 22-3-2024 KỲ THI TỐT NGHI...
 
Sơ đồ tư duy môn sinh học bậc THPT.pdf
Sơ đồ tư duy môn sinh học bậc THPT.pdfSơ đồ tư duy môn sinh học bậc THPT.pdf
Sơ đồ tư duy môn sinh học bậc THPT.pdf
 
Sáng kiến Dạy học theo định hướng STEM một số chủ đề phần “vật sống”, Khoa họ...
Sáng kiến Dạy học theo định hướng STEM một số chủ đề phần “vật sống”, Khoa họ...Sáng kiến Dạy học theo định hướng STEM một số chủ đề phần “vật sống”, Khoa họ...
Sáng kiến Dạy học theo định hướng STEM một số chủ đề phần “vật sống”, Khoa họ...
 
sách sinh học đại cương - Textbook.pdf
sách sinh học đại cương   -   Textbook.pdfsách sinh học đại cương   -   Textbook.pdf
sách sinh học đại cương - Textbook.pdf
 
Đề cương môn giải phẫu......................
Đề cương môn giải phẫu......................Đề cương môn giải phẫu......................
Đề cương môn giải phẫu......................
 
TỔNG HỢP ĐỀ THI CHÍNH THỨC KỲ THI TUYỂN SINH VÀO LỚP 10 THPT MÔN NGỮ VĂN NĂM ...
TỔNG HỢP ĐỀ THI CHÍNH THỨC KỲ THI TUYỂN SINH VÀO LỚP 10 THPT MÔN NGỮ VĂN NĂM ...TỔNG HỢP ĐỀ THI CHÍNH THỨC KỲ THI TUYỂN SINH VÀO LỚP 10 THPT MÔN NGỮ VĂN NĂM ...
TỔNG HỢP ĐỀ THI CHÍNH THỨC KỲ THI TUYỂN SINH VÀO LỚP 10 THPT MÔN NGỮ VĂN NĂM ...
 
Chàm - Bệnh án (da liễu - bvdlct ctump) .pptx
Chàm - Bệnh án (da liễu - bvdlct ctump) .pptxChàm - Bệnh án (da liễu - bvdlct ctump) .pptx
Chàm - Bệnh án (da liễu - bvdlct ctump) .pptx
 

Luận văn: Sáu phương pháp giải các bài toán phổ thông, HOT

  • 1. ĐẠI HỌC QUỐC GIA HÀ NỘI TRƯỜNG ĐẠI HỌC KHOA HỌC TỰ NHIÊN ————————– VŨ THỊ HIỀN SÁU PHƯƠNG PHÁP GIẢI CÁC BÀI TOÁN PHỔ THÔNG LUẬN VĂN THẠC SĨ KHOA HỌC Hà Nội - 2015
  • 2. ĐẠI HỌC QUỐC GIA HÀ NỘI TRƯỜNG ĐẠI HỌC KHOA HỌC TỰ NHIÊN ————————– VŨ THỊ HIỀN SÁU PHƯƠNG PHÁP GIẢI CÁC BÀI TOÁN PHỔ THÔNG Chuyên ngành: Phương pháp toán sơ cấp Mã số : 60460113 LUẬN VĂN THẠC SĨ KHOA HỌC Người hướng dẫn khoa học: GS.TS. Đặng Huy Ruận Hà Nội - 2015
  • 3. Mục lục Mở đầu 1 1 Phương pháp quy nạp 2 1.1 Nguyên lý quy nạp . . . . . . . . . . . . . . . . . . . . . . . . . . . 2 1.2 Phương pháp chứng minh bằng quy nạp . . . . . . . . . . . . . . . 2 1.2.1 Cơ sở quy nạp . . . . . . . . . . . . . . . . . . . . . . . . . . 3 1.2.2 Quy nạp . . . . . . . . . . . . . . . . . . . . . . . . . . . . . 3 1.2.3 Vận dụng phương pháp quy nạp để giải một số bài toán . 4 2 Phương pháp chứng minh phản chứng 17 2.1 Cơ sở lý thuyết . . . . . . . . . . . . . . . . . . . . . . . . . . . . . 17 2.2 Nội dung của phương pháp phản chứng . . . . . . . . . . . . . . . 18 2.3 Trình bày lời giải của phương pháp phản chứng . . . . . . . . . . . 19 2.4 Một số ví dụ minh họa . . . . . . . . . . . . . . . . . . . . . . . . . 19 3 Phương pháp suy luận trực tiếp 28 3.1 Vài nét về phương pháp suy luận trực tiếp . . . . . . . . . . . . . . 28 3.2 Các ví dụ về vận dụng phương pháp suy luận trực tiếp . . . . . . 29 4 Phương pháp đồ thị 35 4.1 Một số khái niệm và kết quả cơ bản của lí thuyết đồ thị . . . . . . 35 4.2 Phương pháp đồ thị . . . . . . . . . . . . . . . . . . . . . . . . . . . 36 4.2.1 Xây dựng đồ thị mô tả các quan hệ . . . . . . . . . . . . . 37 4.2.2 Dựa vào các kết quả của lý thuyết đồ thị hoặc lý luận trực tiếp suy ra đáp án của bài toán D . . . . . . . . . . . . . . 37 4.3 Một số ví dụ . . . . . . . . . . . . . . . . . . . . . . . . . . . . . . . 37 5 Phương pháp bảng 53 5.1 Giới thiệu về phương pháp bảng . . . . . . . . . . . . . . . . . . . . 53 i
  • 4. MỤC LỤC 5.2 Một số ví dụ minh họa . . . . . . . . . . . . . . . . . . . . . . . . . 53 6 Phương pháp sơ đồ 67 6.1 Các bước thực hiện phương pháp sơ đồ . . . . . . . . . . . . . . . 67 6.1.1 Thiết lập sơ đồ . . . . . . . . . . . . . . . . . . . . . . . . . 67 6.1.2 Dựa vào cấu trúc của sơ đồ mô tả quan hệ và điều kiện đã cho trong bài toán mà suy ra đáp án . . . . . . . . . . . 67 6.2 Một số ví dụ . . . . . . . . . . . . . . . . . . . . . . . . . . . . . . . 67 Kết luận . . . . . . . . . . . . . . . . . . . . . . . . . . . . . . . . . . . . 77 Tài liệu tham khảo . . . . . . . . . . . . . . . . . . . . . . . . . . . . . 79 ii
  • 5. Mở đầu Toán phổ thông chẳng những nhiều về số lượng, còn phong phú về chủng loại. Mỗi chủng loại đòi hỏi một phương pháp giải thích hợp. Bởi vậy có nhiều phương pháp giải toán phổ thông. Với khối lượng có hạn, luận văn chỉ xin phép trình bày sáu trong những phương pháp thường dùng nhất. Luận văn gồm phần mở đầu và sáu chương: Chương I trình bày về phương pháp quy nạp, Chương II trình bày về phương pháp phản chứng, Chương III trình bày về phương pháp suy luận trực tiếp, Chương IV trình bày về phương pháp đồ thị, Chương V trình bày về phương pháp bảng, Chương V I trình bày về phương pháp sơ đồ. Mỗi phương pháp đều có phần tóm tắt cơ sở lý thuyết và phần vận dụng phương pháp để giải bài tập. Luận văn được hoàn thành dưới sự hướng dẫn tận tình của thầy giáo GS. TS Đặng Huy Ruận. Em xin bày tỏ lòng kính trọng và biết ơn sâu sắc đến Thầy. Em xin trân trọng cảm ơn ban lãnh đạo khoa Toán - Cơ - Tin học, khoa Sau Đại học, Trường Đại học Khoa học tự nhiên, Đại học Quốc Gia Hà Nội, các Thầy, Cô giáo đã trang bị kiến thức, tạo điều kiện cho chúng em trong thời gian học tập tại đây. Tôi cũng xin gửi lời cảm ơn đến các Đồng nghiệp tại trường Phổ Thông Hồng Đức - Hà Nội, những người đã động viên giúp đỡ tôi rất nhiều trong quá trình hoàn thành luận văn này. Luận văn khó tránh khỏi hạn chế và sơ xuất. Rất mong được sự chỉ bảo của Quý thầy cô và Quý bạn đọc để luận văn được hoàn thiện hơn. Tôi xin chân thành cảm ơn! 1
  • 6. Chương 1 Phương pháp quy nạp Phương pháp quy nạp có vai trò vô cùng quan trọng trong toán học, khoa học và cuộc sống. Đối với nhiều bài toán trong chương trình toán phổ thông là những bài toán logic, tức những bài toán không mẫu mực phương pháp quy nạp cho ta nhiều cách giải hữu hiệu. Suy diễn là quá trình từ "tính chất" của tập thể suy ra tính chất của cá thể, nên luôn luôn đúng, còn quá trình ngược lại, tức quá trình quy nạp: đi từ "tính chất" của một số các thể suy ra "tính chất" của tập thể thì không phải lúc nào cũng đúng, mà quá trình này chỉ đúng khi nó thỏa mãn một số điều kiện nào đó, tức thỏa mãn nguyên lý quy nạp. 1.1 Nguyên lý quy nạp Nếu khẳng định S(n) thỏa mãn hai điều kiện sau: a) Đúng với n = k0 (số tự nhiên nhỏ nhất mà S(n) xác định). b) Từ tính đúng đắn của S(n) đến n = t (hoặc đối với mọi giá trị của n (k0 ≤ n ≤ t)) (t ≥ k0), ta cần chứng minh tính đúng đắn của S(n) đối với n = t + 1, thì khiØS(n) đúng với mọi n ≥ k0. 1.2 Phương pháp chứng minh bằng quy nạp Giả sử khẳng định S(n) xác định với mọi n ≥ t0. Để chứng minh S(n) đúng ∀n ≥ t0 bằng quy nạp ta cần thực hiện theo hai bước sau: 2
  • 7. Chương 1. Phương pháp quy nạp 1.2.1 Cơ sở quy nạp Thực hiện bước này tức là ta thử xem sự đúng đắn của S(n) với n = t0 nghĩa là xét S(t0) có đúng hay không? 1.2.2 Quy nạp Giả sử khẳng định S(n) đã đúng đến n = t (hoặc đối với mọi n (t0 ≤ n ≤ t)) (t ≥ t0). Trên cơ sở giả thiết này ta chứng minh tính đúng đắn của S(n) đối với n = t + 1, tức S(t + 1) đúng. Nếu cả ba bước trên thỏa mãn, thì theo nguyên lý quy nạp S(n) đúng với ∀n ≥ t0. Chú ý: Trong quá trình quy nạp nếu không thực hiện đầy đủ cả ba bước: Cơ sở quy nạp, giả thiết quy nạp và chứng minh quy nạp, thì có thể dẫn đến kết quả sai lầm, chẳng hạn: - Do bỏ bước cơ sở quy nạp, ta đưa ra kết luận không đúng: Mọi số tự nhiên đều bằng nhau! Bằng cách quy nạp như sau: Giả sử các số tự nhiên không vượt quá k + 1 đã bằng nhau. Khi đó ta có k = k + 1 Thêm vào mỗi vế của đẳng thức trên một đơn vị ta có k + 1 = k + 1 + 1 = k + 2 Cứ như vậy suy ra mọi số tự nhiên không nhỏ hơn k đều bằng nhau. Kết hợp với giả thiết quy nạp: Mọi số tự nhiên không vượt quá k đều bằng nhau, đi đến kết luận sai lầm: Tất cả các số tự nhiên đều bằng nhau! - Do bỏ qua khâu quy nạp nên nhà toán học Pháp P.Fermat (1601-1665) đã cho rằng các số dạng 22n + 1 đều là số nguyên tố. P.Fermat xét 5 số đầu tiên: Với n = 0 cho 220 + 1 = 21 + 1 = 3 là số nguyên tố. n = 1 cho 221 + 1 = 22 + 1 = 5 là số nguyên tố. n = 2 cho 222 + 1 = 24 + 1 = 17 là số nguyên tố. n = 3 cho 223 + 1 = 28 + 1 = 257 là số nguyên tố. n = 4 cho 224 + 1 = 216 + 1 = 65537 là số nguyên tố. 3
  • 8. Chương 1. Phương pháp quy nạp Nhưng vào thế kỷ 18 Euler đã phát hiện với n = 5 khẳng định trên không đúng, bởi vì: 225 + 1 = 4294967297 = 641 × 6700417 là hợp số. 1.2.3 Vận dụng phương pháp quy nạp để giải một số bài toán Phương pháp quy nạp được sử dụng trong tính toán, trong chứng minh và trong suy luận dưới nhiều dạng khác nhau, nhưng trong phần này chỉ trình bày việc vận dụng phương pháp quy nạp để giải các bài toán logic, tức các bài toán "không mẫu mực". Ví dụ 1.2.1. Chứng minh rằng: Nếu trong túi có một số tiền nguyên (nghìn) không ít hơn 8000đ, thì luôn luôn có thể mua vé sổ số loại 5000đ và 3000đ. Lời giải: Ta sẽ giải quyết bài toán này bằng phương pháp quy nạp. 1) Cơ sở quy nạp. Nếu trong túi có số tiền ít nhất, tức 8000đ, thì ta mua một vé sổ số loại 5000đ và một vé sổ số loại 3000đ. Khi đó 1 × 5000đ + 1 × 3000đ = 8000đ và ta đã tiêu được hết số tiền có trong túi. 2) Quy nạp. Giả sử với k(k ≥ 8000) nghìn đồng ta đã tiêu hết bằng cách mua các vé sổ số loại 5000đ và 3000đ. Nếu có thêm 1000đ nữa ta cũng có thể mua được bằng cách sau đây: a) Nếu trong các vé sổ số đã mua có ít nhất ba vé loại 3000đ, thì ta trả lại ba vé loại 3000đ, đưa thêm 1000đ và lấy về hai vé loại 5000đ. Khi đó 3 × 3000đ + 1000đ = 2 × 5000đ. b) Nếu trong các vé sổ số đã mua có không quá hai vé loại 3000đ, thì phải có ít nhất một vé loại 5000đ. Bởi vì trong túi không ít hơn 8000đ, mà đã tiêu hết. Khi đó đem trả lại một vé loại 5000đ, đưa thêm 1000đ và lấy về hai vé loại 3000đ, ta có 1 × 5000đ + 1000đ = 2 × 3000đ Như vậy trong mọi trường hợp từ kết quả tiêu k nghìn đầu tiên đã suy ra được cách tiêu nghìn thứ k + 1, nên bài toán đã được giải quyết xong. 4
  • 9. Chương 1. Phương pháp quy nạp Ví dụ 1.2.2. Em An cầm một tờ giấy và lấy kéo cắt thành 7 mảnh. Sau đó nhặt một trong những mảnh giấy đã cắt và lại cắt thành 7 mảnh. Và em An cứ tiếp tục cắt giấy như vậy. Sau một hồi em An thu tất cả các mẩu giấy đã cắt ra và đếm được 122 mảnh. Liệu em An đếm đúng hay sai? Lời giải: 1) Mỗi lần cắt mảnh giấy thành 7 mảnh, tức là đã tạo ra thêm 6 mảnh giấy, nên công thức tính số mảnh giấy sau n bước thực hiện một mảnh giấy thành 7 mảnh có dạng: S(n) = 6n + 1. 2) Tính đúng đắn của công thức S(n) được khẳng định bằng quy nạp theo n. 10) Cơ sở quy nạp. Với n = 1, em An cắt mảnh giấy có trong tay thành 7 mảnh, nên có S(1) = 6.1 + 1 = 6 + 1 = 7 20) Quy nạp. Giả sử sau k bước em An đã nhận được số mảnh giấy là S(k) = 6k + 1 Sang bước k + 1 em An lấy một trong những mảnh giấy nhận được trong k bước trước và cắt thành 7 mảnh, tức em An đã lấy đi một trong S(k) mảnh và thay vào đó 7 mảnh được cắt ra nên S(k + 1) = S(k) − 1 + 7 = 6k + 1 − 1 + 7 = 6k + 7 = 6k + 6 + 1 = 6(k + 1) + 1 Vậy số mảnh giấy em An nhận được sau n bước cắt giấy là S(n). 3) Do S(n) = 6n + 1 ≡ 1 (mod 6), nhưng 122 = 6.20 + 2 ≡ 2 (mod 6), nên em An đếm không đúng. Ví dụ 1.2.3. (Chứng minh tính chất bằng quy nạp). Cho x + 1 x, x = 0 là một số nguyên. Chứng minh rằng với mọi số nguyên dương n, số T(n, x) = xn + 1 xn cũng là số nguyên. Lời giải: Khẳng định được chứng minh bằng quy nạp. 1) Cơ sở quy nạp. Với n = 1 có T(1, x) = x + 1 x là số nguyên, theo giả thiết. 2) Quy nạp. Giả sử khẳng định đúng với mọi số nguyên k(n ≥ k ≥ 1) nghĩa là T(k, x) = xk + 1 xk 5
  • 10. Chương 1. Phương pháp quy nạp là số nguyên. Với n = k + 1 số T(k + 1, x) = xk+1 + 1 xk+1 = = x + 1 x xk + 1 xk − xk−1 + 1 xk−1 theo giả thiết quy nạp, các số x + 1 x, xk−1 + 1 xk−1 và xk + 1 xk đều nguyên, nên T(k + 1, x) là số nguyên và khẳng định đúng với mọi số nguyên dương n. Ví dụ 1.2.4. Chứng minh rằng mọi số nguyên lớn hơn 1 đều có thể viết dưới dạng tích của các thừa số nguyên tố. Lời giải: Ta chứng minh khẳng định bằng quy nạp theo n. 1. Cơ sở quy nạp. Với n = 2, ta có 2 = 2, Với n = 3, ta có 3 = 3, n = 4, ta có 4 = 2 × 2 Vậy khẳng định đúng với n = 2, 3, 4. 2. Quy nạp. Giả sử với mọi số nguyên n đều phân tích được thành tích của các thừa số nguyên tố. Ta chứng minh n+ 1 cũng phân tích được thành tích của các thừa số nguyên tố. Thật vậy • Nếu n + 1 là số nguyên tố thì nó bằng tích của chính n + 1. • Nếu n + 1 là hợp số thì n + 1 = a.b với 2 ≤ a, b < n. Theo giả thiết quy nạp, thì a, b đều phân tích được thành tích các thừa số nguyên tố. Suy ra, n + 1 cũng phân tích được thành tích các thừa số nguyên tố. Theo nguyên lý quy nạp, mọi số nguyên n > 1 đều phân tích được thành tích các thừa số nguyên tố. Ví dụ 1.2.5. (Chứng minh tính chia hết bằng quy nạp). Chứng minh rằng với mọi số nguyên dương n số 23n +1 chia hết cho 3n+1 23n + 1 ...3n+1 và số 23n +1 không chia hết cho 3n+2 23n + 1 ...3n+2 . Lời giải: Bài toán được giải quyết bằng quy nạp. Kí hiệu 23n + 1 = An. 1) Cơ sở quy nạp. Với n = 1 ta có A1 = 231 + 1 = 23 + 1 = 8 + 1 = 9, nên A1 ...32 và A1 ...33. Với n = 2 ta có A2 = 232 + 1 = 513, nên A2 ...33 và A2 ...34. 6
  • 11. Chương 1. Phương pháp quy nạp 2) Quy nạp. Giả sử khẳng định đã đúng với n = k ≥ 2, nghĩa là Ak ...3k+1 và Ak ...3k+2. Vì Ak ...3k+1, nên ∃M ∈ N (Ak = M.3k+1 và M ...3) (1.1) Xét n = k + 1 Ak+1 = 23k+1 + 1 = 23k .3 + 1 = 23k 3 + 1 = 23k + 1 23k 2 − 23k + 1 = 3k+1 .M. 23k + 1 2 − 3.23k = 3k+1 .M. 3k+1 .M 2 − 3.23k = 3k+1 .M. 32k+2 .M2 − 3.23k = 3k+2 .M. 32k+1 .M2 − 23k 10) Khi đó Ak+1 ...3k+2, nên với mọi số nguyên dương n đều có 23n + 1 ...3n+1. 20) Ak+1 ...3k+3. a) Vì M ...3 (theo (1.1)), nên 3k+2 .M ...3k+3 (1.2) b) Do k ≥ 2, nên ∃t ∈ N (k = t + 2) và 32k+1.M2 = 3k+3+t.M2. Bởi vậy 32k+1 .M2...3k+3 (1.3) Giả sử 23k ...3k+3. Khi đó 23k ...9, nhưng 23k = 23 k = 8k = ±1 (mod 9), nên 23k ...3k+3 (1.4) Từ các quan hệ (1.3) và (1.4) ta suy ra 32k+1 .M2 − 23k ...3k+3 (1.5) Từ (1.2) và (1.5) suy ra: Ak+1 ...3k+3, nên với mọi số nguyên dương n số An không chia hết cho 3n+2. Ví dụ 1.2.6. (Chứng minh bất đẳng thức bằng quy nạp) Cho n(n ≥ 1) số dương x1, x2, · · · , xn thỏa mãn điều kiện x1.x2. · · · xn−1.xn = 1 Chứng minh rằng x1 + x2 + · · · + xn−1 + xn ≥ n và dấu đẳng thức xảy ra khi và chỉ khi x1 = x2 = · · · = xn 7
  • 12. Chương 1. Phương pháp quy nạp Lời giải: Bài toán được giải quyết bằng quy nạp. 1) Cơ sở quy nạp. Với n = 1 ta chỉ có số x1. Vì x1 = 1, nên x1 thỏa mãn bất đẳng thức x1 ≥ 1. 2) Quy nạp. Giả sử khẳng định đã đúng với k số dương tùy ý có tích bằng 1. Xét k + 1 số dương tùy ý x1, x2, · · · , xk, xk+1 với x1.x2. · · ·xk.xk+1 = 1. Có hai khả năng đặt ra: a) Nếu x1 = x2 = · · · = xk = xk+1, thì xi = 1 (1 ≤ i ≤ k + 1). Khi đó x1 + x2 + · · · + xk + xk+1 = k + 1 và khẳng định được chứng minh. b) Nếu k + 1 số được xét x1, x2, · · · , xk, xk+1 không đồng thời bằng nhau, thì do tích của chúng bằng 1 và các số này đều dương, phải có ít nhất một số lớn hơn 1 và ít nhất một số nhỏ hơn 1. Không giảm tính tổng quát, giả sử xk < 1 và xk+1 > 1. Khi đó 1 − xk > 0, xk+1 − 1 > 0 nên(1 − xk) (xk+1 − 1) > 0 Bởi vậy xk + xk+1 > xk.xk+1 + 1 (1.6) Từ đẳng thức: x1.x2. · · · xk−1 (xkxk+1) = x1.x2. · · · .xk−1xk.xk+1 = 1 suy ra k số dương x1, x2, · · · , xk−1, (xkxk+1) có tích bằng 1, nên theo giả thiết quy nạp, có x1 + x2 + · · · + xkxk+1 ≥ k Cộng cả hai vế của bất đẳng thức trên với 1 ta có bất đẳng thức x1 + x2 + · · · + xkxk+1 + 1 ≥ k + 1 (1.7) Từ bất đẳng thức (1.6) và (1.7) ta có x1 + x2 + · · · + xk + xk+1 ≥ x1 + x2 + · · · + xkxk+1 + 1 ≥ k + 1 Khẳng định đã được chứng minh. Với n số dương tùy ý x1 = x2 = · · · = xn và x1.x2. · · · xn = 1 suy ra xi = 1(1 ≤ i ≤ n), nên x1 + x2 + · · · + xn = n. Ví dụ 1.2.7. (Tìm chữ số tận cùng bằng quy nạp) Với mọi số nguyên dương k ≥ 2 hãy tìm chữ số tận cùng của số Ak = 22k + 1. 8
  • 13. Chương 1. Phương pháp quy nạp Lời giải: Bài toán được giải quyết bằng quy nạp. 1) Cơ sở quy nạp. Với k = 2 số A2 = 222 + 1 = 24 + 1 = 17 2) Quy nạp. Giả sử với k = n ≥ 2 số An đã có tận cùng là 7. Xét số An+1 = 22n+1 + 1. Do An tận cùng số 7, nên tồn tại số nguyên dương m, để An = 10m + 7. Từ đó An − 1 = 22n = 10m + 6. An+1 = 22n+1 + 1 = 22n.2 + 1 = 22n 2 + 1 = (10m + 6)2 + 1 = 100m2 + 120m + 36 + 1 = 10 10m2 + 12m + 37 = 10 10m2 + 12m + 3 + 7 nên An+1 tận cùng bằng chữ số 7. Vậy với mọi số k ≥ 2, số Ak tận cùng bằng chữ số 7. Ví dụ 1.2.8. Tính tổng sau S(n) = 12 + 32 + · · · + (2n + 1)2 , trong đó n là một số tự nhiên. Lời giải: Ta sẽ đi dự đoán công thức tổng S(n). Ta thấy S(n) là tổng của các lũy thừa bậc hai của các số 1, 3, · · · , (2n + 1), nên ta dự đoán S(n) phải là một đa thức của n có bậc không nhỏ hơn ba. Giả sử S(n) = an3 + bn2 + cn + d. Vì S(0) = 1 nên d = 1. Lần lượt thay n = 1, n = 2, n = 3, ta được hệ    a + b + c = 9 8a + 4b + 2c = 34 27a + 9b + 3c = 83 Giải hệ này ta thu được a = 4 3, b = 4, c = 11 3 . Khi đó, S(n) = 4 3 n3 + 4n2 + 11 3 n + 1 = (n + 1) (2n + 1) (2n + 3) 3 (1.8) Ta sẽ chứng minh công thức (1.8) bằng quy nạp theo n. 1) Cơ sở quy nạp. Với n = 0, 1, 2, 3, 4, 5, ta dễ dàng kiểm tra được (1.8) đúng. 2) Quy nạp. Giả sử (1.8) đúng với n = k, ta chứng minh (1.8) đúng với n = k + 1. Thật vậy S(k + 1) = 12 + 32 + · · · + (2k + 1)2 + (2k + 3)2 = S(k) + (2k + 3)2 . 9
  • 14. Chương 1. Phương pháp quy nạp Theo giả thiết quy nạp thì S(k) = (k+1)(2k+1)(2k+3) 3 . Suy ra S(k + 1) = (k + 1) (2k + 1) (2k + 3) 3 + (2k + 3)2 = (k + 2) (2k + 3) (2k + 5) 3 Vậy công thức (1.8) đúng với n = k + 1, do đó theo nguyên lý quy nạp (1.8) đúng với mọi n. Ví dụ 1.2.9. Chứng minh rằng A(n) = 7n + 3n − 1 chia hết cho 9 với mọi số tự nhiên n. Lời giải: Đặt A(n) = 7n + 3n − 1. Ta sẽ chứng minh bài toán bằng quy nạp. 1) Cơ sở quy nạp. Với n = 0, ta có A(0) = 0 chia hết cho 9. 2) Quy nạp. Giả sử A(k) chia hết cho 9 với mọi k ∈ N. Ta sẽ chứng minh A(k + 1) cũng chia hết cho 9. Thật vậy, ta có A(k + 1) = 7k+1 + 3 (k + 1) − 1 = 7A(k) − 9. (2k − 1) . (1.9) Theo giả thiết quy nạp thì A(k) chia hết cho 9. Do đó từ (1.9) ta suy ra A(k + 1) cũng chia hết cho 9. Vậy A(n) chia hết cho 9 với mọi số tự nhiên n. Ví dụ 1.2.10. Gọi S(n) là tổng tất cả các ước số lẻ lớn nhất của các số tự nhiên từ 1 đến 2n. Chứng minh rằng 3S(n) = 4n + 2. Lời giải: Các số tự nhiên từ 1 đến 2n bao gồm các số lẻ từ 1 đến 2n và gấp đôi của các số tự nhiên từ 1 đến 2n−1. Từ đó ta có S(n) = S(n − 1) + 1 + 3 + · · · + (2n − 1) hay S(n) = S(n − 1) + 4n−1 (1.10) Ta sẽ chứng minh bài toán đã cho bằng quy nạp theo n. 1) Cơ sở quy nạp. Với n = 1, ta có 3S(1) = 3(1 + 1) = 6 = 41 + 2, khẳng định đúng với n = 1 10
  • 15. Chương 1. Phương pháp quy nạp 2) Quy nạp. Giả sử khẳng định đúng với n = k, tức là 3S(k) = 4k + 2 (1.11) Ta cần chỉ ra khẳng định đúng với n = k + 1 hay 3S(k + 1) = 4k+1 + 2. Thật vậy, theo (1.10) ta có S(k + 1) = S(k) + 4k ⇔ 3S(k + 1) = 3S(k) + 3.4k = 4k + 2 + 3.4k = 4k+1 + 2. (1.12) theo giả thiết quy nạp. Vậy khẳng định đã cho đúng với mọi n ∈ N∗. Ví dụ 1.2.11. Tìm bậc cao nhất k của 2007 sao cho 2007k là ước của số: 200820072006 + 200620072008 . Lời giải: Ta chứng minh hai bổ đề sau: Bổ đề 1.2.1. Với số tự nhiên lẻ a ≥ 3 và với mọi n nguyên dương không chia hết cho a, ta có: (1 + a)an = 1 + Snan+1 , trong đó Sn là số nguyên dương và không chia hết cho a. Bổ đề 1.2.2. Với số tự nhiên lẻ b ≥ 3 và với mọi số n nguyên dương, ta có: (b − 1)bn = −1 + tnbn+1 , trong đó tn là số nguyên không chia hết cho b. +, Chứng minh bổ đề 1: Ta sẽ chứng minh bổ đề 1 bằng quy nạp. 1. Cơ sơ quy nạp. Với n = 1, ta có (1 + a)a = 1 + C1 aa + C2 aa2 + . . . + Ca a aa = = 1 + a2 1 + C2 a + C3 aa + . . . + aa−2 = 1 + S1a2 . Vì a lẻ nên C2 a = a! 2!(a−2)! = a(a−1) 2 ...a. Do vậy, S1 = 1 + C2 a + C3 aa + . . . + aa−2 không chia hết cho a. 11
  • 16. Chương 1. Phương pháp quy nạp 2. Quy nạp. Giả sử khẳng định trên đúng với n = k, tức là (1 + a)ak = 1 + Skak+1 , trong đó Sk là số nguyên dương và không chia hết cho a. Ta có (1 + a)ak+1 = (1 + a)ak .a = 1 + Skak+1 a = 1 + C1 aSkak+1 + C2 a Skak+1 2 + . . . + Ca a Skak+1 a = 1 + ak+2 Sk + C2 aS2 k + . . . + Sa kaak+a−k−2 = 1 + Sk+1ak+2 . Vì Sk không chia hết cho a suy ra Sk+1 không chia hết cho a. Vậy bổ đề được chứng minh. +, Chứng minh bổ đề 2. Bổ đề 2 được chứng minh bằng phương pháp quy nạp bằng cách lý luận tương tự như chứng minh bổ đề 1. Áp dụng hai bổ đề trên với a = b = 2007, ta được 200820072006 + 200620072008 = S200620072007 + t200820072009 = S2006 + t200820072 20072007 . Vì S2006 không chia hết cho 2007 nên số k lớn nhất thỏa mãn điều kiện bài toán là 2007. Ví dụ 1.2.12. Dãy số (un) xác định như sau: u0 = u1 = 1 un+1 = un−1un + 1; n = 1, 2, . . . Chứng minh rằng: u2008 − 3 ...4. Lời giải: Từ cách xác định dãy số, ta có u2 = 2; u3 = 3; u4 = 7; u5 = 22; u6 = 155; . . . Nhận thấy: u2 ≡ 2 (mod 4); u5 ≡ 2 (mod 4). u3 ≡ 3 (mod 4); u4 ≡ 3 (mod 4). Ta dự đoán: un ≡ 2 (mod 4) nếu n = 3k + 2, (k ∈ Z) un ≡ 3 (mod 4) nếu n = 3k + 1 hoặc n = 3k, (k ∈ Z). (1.13) 12
  • 17. Chương 1. Phương pháp quy nạp Ta sẽ chứng minh quan hệ (1.13) bằng quy nạp. 1. Cơ sở quy nạp. Với n = 2, 3, 4, 5, ta có quan hệ (1.13) đúng. 2. Quy nạp. Giả sử (1.13) đúng với n = t. Ta cần chứng minh (1.13) đúng với n = t + 1. +) Nếu t = 3k, khi đó t − 1 = 3k − 1 = 3(k − 1) + 2 t + 1 = 3k + 1. Do đó, theo giả thiết quy nạp ta có ut ≡ 3 (mod 4); ut−1 ≡ 2 (mod 4) Suy ra ut+1 ≡ 3.2 + 1 ≡ 3 (mod 4), hay (1.13) đúng. +) Nếu t = 3k + 1, suy ra t − 1 = 3k t + 1 = 3k + 2 . Do đó, theo giả thiết quy nạp ta có ut ≡ 3 (mod 4); ut−1 ≡ 3 (mod 4) Suy ra ut+1 ≡ 3.3 + 1 ≡ 2 (mod 4), hay (1.13) đúng. +) Nếu t = 3k + 2, suy ra t − 1 = 3k + 1 t + 1 = 3(k + 1) . Do đó, theo giả thiết quy nạp ta có ut ≡ 2 (mod 4); ut−1 ≡ 3 (mod 4) Suy ra ut+1 ≡ 3.2 + 1 ≡ 3 (mod 4), hay (1.13) đúng. Vậy (1.13) được chứng minh. Do 2008 = 3.669 + 1, nên u2008 ≡ 3 (mod 4), hay u2008 − 3 ...4. 13
  • 18. Chương 1. Phương pháp quy nạp Ví dụ 1.2.13. Chứng minh rằng số được thành lập bởi 3n chữ số giống nhau thì chia hết cho 3n, trong đó n là một số nguyên dương cho trước. Lời giải: Gọi A(n) = aa . . . a, trong đó A(n) gồm 3n chữ số a và 1 ≤ a ≤ 9. Ta chứng minh khẳng định bằng quy nạp theo n. 1. Cơ sở quy nạp. Với n = 1, ta có A(1) = aaa chia hết cho 31 = 3. Khẳng định đúng với n = 1. 2. Quy nạp. Giả sử A(n) chia hết cho 3n, ta cần chứng minh A(n + 1) chia hết cho 3n+1. Thật vậy, ta có A(n + 1) = A(n)A(n)A(n) = A.102.3n + A.103n + A = A 102.3n + 103n + 1 . Vì 10 ≡ 1 (mod 3) nên 102.3n + 103n + 1 ≡ 0 (mod 3). Hơn nữa, theo giả thiết quy nạp, A(n) chia hết 3n. Do đó, ta có A(n + 1) chia hết 3n.3 = 3n+1, hay khẳng định đúng với n + 1. Theo nguyên lý quy nạp, khẳng định đã cho đúng với mọi n ∈ Z+. Ví dụ 1.2.14. Mỗi đầu của đường kính thuộc đường tròn tâm 0 ghi số 1. Sau đó tại trung điểm của mỗi cung nhận được ghi số 2 (tổng của hai số được ghi ở hai đầu của mỗi cung) (Bước 2). Coi bốn điểm ghi số là các điểm chia đường tròn. Khi đó đường tròn được chia thành bốn cung bằng nhau. Giữa mỗi cung này lại ghi số 3 (tổng của hai số được ghi ở hai đầu của cung tương ứng) (Bước 3). Cứ tiếp tục như vậy. Hỏi sau n bước tổng các số được ghi trên đường tròn là bao nhiêu? 0 1 1 2 2 3 3 3 3 14
  • 19. Chương 1. Phương pháp quy nạp Lời giải: Sau n bước tổng các số trên đường tròn là Sn = 2.3n. Ta sẽ chứng minh công thức trên bằng quy nạp theo n. 1. Cơ sở quy nạp. Sau Bước 1, trên đường tròn có bốn (22) số 1, 2, 1, 2. Khi đó S1 = 1 + 2 + 1 + 2 = 6 = 2.31 2. Quy nạp. Giả sử khẳng định đúng với n = k(k ≥ 1), nghĩa là sau k bước trên đường tròn đã có 2k+1 số s1, s2, . . . , s2k+1, . . . , s2k+1 , (1.14) với tổng là Sk = 2.3k. 1 1 2 2 3 3 3 3 Sang bước (k + 1), ta coi 2k+1 điểm đã ghi là các điểm chia, nên đường tròn được thành 2k+1 cung bằng nhau. Do trung điểm của mỗi cung này lại ghi tổng của hai số đã ghi ở đầu của mỗi cung, nên mỗi số thuộc dãy (1.14) được xuất hiện đúng hai lần trong các tổng mới (các số được ghi tại bước k + 1). Do đó, tổng các số được ghi trên đường tròn sau k + 1 bước là: Sk+1 = tổng các số đã ghi sau bước k + tổng các số được ghi tại bước k+1 = Sk + 2Sk = 3Sk = 3.2.3k = 2.3k+1 Khẳng định được chứng minh. Ví dụ 1.2.15. Chứng minh rằng trên mặt phẳng n đường thẳng khác nhau cùng đi qua một điểm, chia mặt phẳng thành 2n phần khác nhau. Lời giải: Bài toán được giải quyết bằng quy nạp. 1) Cơ sở quy nạp. Với n = 1, ta có một đường thẳng. Nó chia mặt phẳng thành hai phần, nên khẳng định đúng. 15
  • 20. Chương 1. Phương pháp quy nạp 2) Quy nạp. Giả sử với n = k khẳng định đã đúng, nghĩa là k đường thẳng tùy ý cùng đi qua một điểm M đã chia mặt phẳng thành 2k phần khác nhau. Xét n = k + 1 đường thẳng khác nhau tùy ý cùng đi qua một điểm. Kí hiệu các đường này, một cách tương ứng bằng δ1, δ2, · · · , δk, δk+1. Theo giả thiết quy nạp k đường thẳng δ1, δ2, · · · , δk đã chia mặt phẳng thành 2k phần khác nhau. M δs δt δk+1 Vì các đường thẳng đều khác nhau và cùng đi qua điểm M, nên tồn tại các chỉ số s, t (1 ≤ s, t ≤ k) để δk+1 là đường thẳng duy nhất nằm trong góc được lập nên bởi δs và δt. Khi đó δk+1 chia hai phần mặt phẳng được giới hạn bởi δs và δt thành bốn phần. Bởi vậy k + 1 đường thẳng δ1, δ2, · · · , δk, δk+1 chia mặt phẳng thành 2k − 2 + 4 = 2k + 2 = 2 (k + 1) phần khác nhau. Khẳng định được chứng minh. 16
  • 21. Chương 2 Phương pháp chứng minh phản chứng Chứng minh là một nét đặc trưng của toán học, tạo ra sự khác biệt giữa toán học với các môn khoa học khác. Nắm bắt phương pháp và kĩ thuật chứng minh cũng là yêu cầu bắt buộc đối với học sinh nói chung. Các phương pháp và kĩ thuật chứng minh rất phong phú: Từ chứng minh trực tiếp đến gián tiếp, từ chứng minh bằng quy nạp đến chứng minh bằng phản chứng, từ ví dụ đến phản ví dụ, từ xây dựng đến không xây dựng. Trong bài luận văn này xin được đề cập đến phép chứng minh phản chứng, một trong những phương pháp chứng minh kinh điển và quan trọng nhất của toán học. Chứng minh phản chứng có thể nói là một trong những vũ khí quan trọng nhất của toán học. Nó cho phép chúng ta chứng minh sự có thể và không thể của một tính chất nào đó. Nó cho phép chúng ta biến thuận thành đảo, biến đảo thành thuận. Nó cho phép chúng ta lý luận trên những đối tượng mà không rõ là có tồn tại hay không. 2.1 Cơ sở lý thuyết Cơ sở lý thuyết của phương pháp phản chứng là các định luật trong logic: Gọi p, q, r là các mệnh đề toán học nào đó, khi đó Định lý 2.1.1. (Các định luật cơ bản) 1. (Phi mâu thuẫn): p ∧ p = 0 2. (Bài trung):p ∨ p = 1 Định lý 2.1.2. (Các định luật phản chứng) 1. (Phản chứng) (p ⇒ q) ⇔ (q ⇒ p). 17
  • 22. Chương 2. Phương pháp chứng minh phản chứng 2. (Phản chứng suy rộng): (p ∧ q ⇒ r) ⇔ (p ∧ r ⇒ q) (2.1) 3. (p ⇒ q ∧ q) ⇒ p. 4. (q ⇒ 0) ⇒ q. Các định luật trên có thể được chứng minh chẳng hạn bằng phương pháp lập bảng chân lý hoặc phương pháp biến đổi tương đương. Chẳng hạn, ta có thể chứng minh (2.1) như sau: (p ∧ q ⇒ r) ⇔ p ∧ q hoặc r (do a ⇒ b ⇔ a hoặc b) ⇔p hoặc q hoặc r (do a ∧ b ⇔ a hoặc b) ⇔(p hoặc r) hoặc q (giao hoán, kết hợp và r ⇔ r) ⇔p ∧ r hoặc q ⇔(p ∧ r ⇒ q) đpcm. 2.2 Nội dung của phương pháp phản chứng Để chứng minh khẳng định p ⇒ q bằng phương pháp phản chứng ta giả sử q sai, tức là q là mệnh đề đúng. Nếu từ đó thu được một điều vô lý (vl) thì điều đó chứng tỏ giả sử của ta là sai, tức là q đúng. Điều vô lý (vl) có thể thuộc một trong các dạng sau: +) Điều trái với giả thiết p (vl ≡ p) +) Điều trái với một trong các kiến thức đã biết (vl ≡ 0) +) Điều trái với giả sử phản chứng (vl ≡ q). Khi xây dựng mệnh đề phản chứng q ta cần nhớ 1. p ⇔ p. 2. p ∧ q ⇔ p hoặc q. 3. p ∨ q ⇔ p ∧ q. 4. ∀x, p(x) ⇔ ∃x, p(x). 5. ∃x, p(x) ⇔ ∀x, p(x). 6. ∀x, p(x) ⇔ ∃x, p(x). 7. ∃x, p(x) ⇔ ∀x, p(x). 8. Nếu p(x) := f(x)Rg(x), thì p(x) là : f(x)Rg(x) trong đó, R và R được xác định như sau: 18
  • 23. Chương 2. Phương pháp chứng minh phản chứng R R = == = > > ≥ ≥ < < ≤ ≤ Ngoài ra còn có: Định lý 2.2.1. (Định luật về âm bản). Cho A là một công thức logic mà trong đó chỉ chứa các phép toán: Phủ định, ∧, hoặc đối với các mệnh đề: p1, p2, · · · , pn. Âm bản của A, kí hiệu là Ad là một công thức thu được từ A bằng cách thay pj ∼ pj; pj ∼ pj, ∨ ∼ ∧; ∧ ∼ ∨. Khi đó: A ⇔ Ad Phương pháp phản chứng còn dựa trên nguyên lý Dirichlet do nhà toán học Đức nổi tiếng Peter Dirichlet (1805-1859) đề xuất, mà dạng đơn giản nhất của nguyên lý này được phát biểu như sau: "Không thể nhốt 7 chú thỏ vào 3 cái lồng sao cho mỗi lồng không có quá hai chú thỏ". Nói cách khác "Nếu nhốt 7 chú thỏ vào 3 cái lồng, thì phải có ít nhất một lồng có không ít hơn 3 chú thỏ". 2.3 Trình bày lời giải của phương pháp phản chứng Bài toán: Chứng minh p ⇒ q Lời giải: Giả sử ngược lại, q sai, tức là q. Mà q ⇒ · · · ⇒ vl. Vậy giả sử của ta là sai, tức là q đúng. Ngoài ra, ta sẽ giải một số bài toán bằng phương pháp phản chứng dựa trên nguyên lý Dirichlet. 2.4 Một số ví dụ minh họa Ví dụ 2.4.1. Cho f(x) = ax2 + bx + c. Giả sử |a| + |b| + |c| > 17 (2.2) Chứng minh rằng ∃x ∈ [0; 1], |f(x)| > 1 (2.3) Lời giải: Ta sẽ chứng minh bằng phương pháp phản chứng. Giả sử (2.3) sai, tức là ∀x ∈ [0; 1], |f(x)| ≤ 1 (2.4) 19
  • 24. Chương 2. Phương pháp chứng minh phản chứng Chọn x = 0; 1 2; 1, từ (2.4) ta được |c| ≤ 1 và: |a + b + c| ≤ 1 a 4 + b 2 + c ≤ 1 Suy ra |a| + |b| + |c| ≤ 17 Đó là điều vô lý (trái với (2.2)). Vậy giả sử của ta là sai, tức là (2.3) đúng. Ví dụ 2.4.2. Chứng minh tập các số nguyên tố P là tập vô hạn. Lời giải: (phản chứng). Giả sử ngược lại, tập P là hữu hạn. Giả sử P = {p1, p2, · · · , pn}. Khi đó, tồn tại số nguyên tố lớn nhất. Gọi đó là pn. Tức là: pn ∈ P và ∀p ∈ P, p ≤ pn. Xét số x = p1.p2. . . . .pn +1. Ta có, x ∈ N; x không chia hết cho các số p1; p2; . . . ; pn (vì nếu x ...pj nào đó thì 1 ...pj: vô lý). Vậy x ∈ P. Mà hiển nhiên, x > pn. Đó là điều vô lý (trái với cách chọn pn). Vậy điều giả sử của ta là sai, tức là P là tập vô hạn. Ví dụ 2.4.3. Có thể chia các số tự nhiên từ 1 đến 21 thành các nhóm đôi một rời nhau, sao cho trong mỗi nhóm số lớn nhất bằng tổng các số còn lại hay không? Lời giải: Giả sử chia được. Khi đó tổng các số ở mỗi nhóm là một số chẵn (bằng hai lần số lớn nhất). Vậy tổng của 21 số đã cho là một số chẵn (vì các nhóm đôi một rời nhau và tổng của các số chẵn là số chẵn). Nhưng tổng của 21 số đó là 21.11 = 231 là số lẻ. Điều vô lý này chứng tỏ giả sử của ta là sai, tức là không chia được thành các nhóm thỏa mãn yêu cầu đề bài. Ví dụ 2.4.4. Có thể tìm được hay không 5 số nguyên, sao cho các tổng của hai số một trong 5 số đó lập thành 10 số nguyên liên tiếp? Lời giải: Giả sử tìm được 5 số như vậy. Gọi s là tổng của 5 số đó và n là giá trị nhỏ nhất của tổng các cặp hai số. Khi đó 10 số nguyên liên tiếp nói trong đề bài là n, n + 1, . . . , n + 9 20
  • 25. Chương 2. Phương pháp chứng minh phản chứng Ta tính tổng τ của 10 số đó theo hai cách khác nhau: Một mặt, τ = n + (n + 1) + (n + 2) + . . . + (n + 9) = 5 (2n + 9) Mặt khác τ = 4s (do trong τ mỗi số đã cho có mặt đúng 4 lần). Từ đó suy ra 4s = 5 (2n + 9) là điều vô lý. Vậy giả sử ban đầu là sai, tức là không thể chọn được 5 số thỏa mãn yêu cầu bài ra. Ví dụ 2.4.5. Cho ba điểm A, B, C phân biệt trên mặt phẳng. Chứng minh rằng nếu tồn tại điểm G thuộc mặt phẳng đó thỏa mãn: −→ GA + −−→ GB + −→ GC = −→ 0 thì điểm G đó là duy nhất. Lời giải: Ta chứng minh bằng phương pháp phản chứng. Giả sử còn có điểm O = G thỏa mãn yêu cầu bài toán, tức là −→ OA + −−→ OB + −→ OC = −→ 0 Ta có −→ GA + −−→ GB + −→ GC = −→ 0 ⇔ −→ GO + −→ OA + −→ GO + −−→ OB + −→ GO + −→ OC = −→ 0 ⇔ 3 −→ GO + −→ OA + −−→ OB + −→ OC = −→ 0 Do −→ OA + −−→ OB + −→ OC = −→ 0 , suy ra 3 −→ GO = −→ 0 ⇔ −→ GO = −→ 0 ⇔ G ≡ O Vậy G là duy nhất. Ví dụ 2.4.6. (IMO 1982) Cho phương trình x3 − 3xy2 + y3 = n, n ∈ N∗ (2.5) 1) Chứng minh rằng nếu (2.5) có nghiệm nguyên thì nó không có nghiệm nguyên duy nhất. 2) Tìm nghiệm nguyên của (2.5) khi n = 2005. 21
  • 26. Chương 2. Phương pháp chứng minh phản chứng Lời giải: 1) Dễ dàng biến đổi x3 − 3xy2 + y3 = (y − x)3 − 3 (y − x) x2 + (−x)3 = (−y)3 − 3 (−y) (x − y)2 + (x − y)3 Từ đó ta có: Nếu (x; y) là một nghiệm của (2.5) thì (y − x; −x) , (−y; x − y) cũng là các nghiệm của (2.5). Ba nghiệm này đôi một khác nhau vì nếu có hai nghiệm nào đó bằng nhau thì ta có x = y = 0 là điều vô lý do n > 0. Vậy nếu (2.5) có nghiệm, thì nó có ít nhất ba nghiệm phân biệt, tức là (2.5) không thể có nghiệm duy nhất. 2) Với n = 2005 giả sử (2.5) có nghiệm (x; y) ∈ Z2. Ta viết các nghiệm theo (mod 3), phương trình đã cho trở thành x3 + y3 = −1 (mod 3) ⇒ x + y = −1 (mod 3). Do vậy ta có các trường hợp sau: (i)x ≡ 0 (mod 3) và y ≡ −1 (mod 3); (2.6) (ii)x ≡ 1 (mod 3) và y ≡ 1 (mod 3); (2.7) (iii)x ≡ −1 (mod 3) và y ≡ 0 (mod 3). (2.8) Trong trường hợp (i) đặt x = 3m, y = 3n − 1, thay vào phương trình đã cho ta được V T ≡ −1 (mod 3) còn V P = 2005 ≡ −2 (mod 9) ⇒ vô lý. Trong trường hợp (ii), do (y − x; −x) cũng là nghiệm, mà y − x ≡ 0 (mod 3) và −x ≡ −1 (mod 3) nên ta cũng thu được một điều vô lý. Tương tự từ trường hợp (iii) ta cũng thu được một điều vô lý. Vậy với n = 2005 phương trình đã cho không có nghiệm nguyên. Ví dụ 2.4.7. (IMO 1983). Tìm tất cả những hàm số f(x) : R+ → R+, là toàn ánh và thỏa mãn đồng thời hai điều kiện sau: 1) f(xf(y)) = yf(x), ∀x, y ∈ R+. 2) f(x) → 0 khi x → +∞. Lời giải: Do f(x) là toàn ánh và 1 ∈ R+ nên ∃y0 ∈ R+ : f(y0) = 1. Trong 1) cho x = 1, y = y0 ta được f(1) = f(1.1) = f(1.f(y0)) = y0f(1) → y0 = 1(do f(1) > 0) ⇒ y0 = 1 ⇒ f(1) = 1. 22
  • 27. Chương 2. Phương pháp chứng minh phản chứng Vậy f(x) có một điểm bất động là x = 1. Ta sẽ chứng minh đó là điểm bất động duy nhất. Thật vậy, giả sử f(x) có hai điểm bất động x, y phân biệt, khi đó ta có: (i) f(xy) = f(xf(y)) = yf(x) = yx = xy → xy cũng là điểm bất động của f(x). (ii) 1 = f(1) = f 1 xf(x) = xf(1 x) → 1 x cũng là điểm bất động của f(x). Giả sử còn có điểm bất động x = 1, khi đó theo (ii), f(x) còn có điểm bất động 1 x . Trong hai số đó phải có một số lớn hơn 1. Giả sử x > 1. Theo (i), f(x) sẽ có vô số điểm bất động xn = xn, n ∈ N∗. Ta có dãy đối số (xn) → +∞ (do x > 1), do đó dãy hàm (f(xn)) = (xn) → +∞. Điều đó trái với 2) : f(x) → 0 khi x → +∞. Vậy giả sử của ta là sai, tức là f(x) có duy nhất một điểm bất động là x = 1. Nhưng trong (i) cho y = x ta được f(xf(x)) = xf(x), ∀x ∈ R+ ⇒ xf(x) cũng là điểm bất động của f(x). Vậy ta phải có ∀x ∈ R+ : xf(x) = 1 ⇔ f(x) = 1 x . Dễ tìm thấy hàm số này thỏa mãn các điều kiện bài ra. Vậy f(x) = 1 x là hàm số cần tìm. Ví dụ 2.4.8. (IMO 1983) Cho các số a, b, c ∈ N∗, đôi một nguyên tố cùng nhau. Chứng minh rằng phương trình xbc + yca + zab = 2abc − ab − bc − ca (2.9) không có nghiệm tự nhiên. Lời giải: Giả sử (2.9) có nghiệm (x; y; z) ∈ N3. Từ (2.9) ta có (z + 1) ab ...c. Mà (a; c) = (b; c) = 1 ⇒ z + 1 ...c, mà z + 1 > 0 nên từ đó có z + 1 ≥ c. Tương tự, có x + 1 ≥ a, y + 1 ≥ b. Từ các đánh giá này ta được V T(2.9) ≥ (a − 1) bc + (b − 1) ca + (c − 1) ab = 3abc − ab − bc − ca > 2abc − ab − bc − ca = V P(2.9) Điều này vô lý chứng tỏ giả sử của ta là sai, tức là phương trình (2.9) không có nghiệm tự nhiên. Ví dụ 2.4.9. (Putnam 1998) Chứng minh rằng với mọi số nguyên a, b, c luôn tìm được số nguyên dương n, sao cho số f(n) = n3 + an2 + bn + c 23
  • 28. Chương 2. Phương pháp chứng minh phản chứng không phải là số chính phương. Lời giải: Ta chứng minh mệnh đề sau: ∀ (a; b; c) ∈ Z3 , ∃n ∈ N∗ : f(n) không phải là số chính phương. (2.10) Nhận xét rằng mọi số chính phương đều ≡ 0 (mod 4) hoặc ≡ 1 (mod 4). Giả sử (2.10) là mệnh đề sai, tức là ∃ (a; b; c) ∈ Z3 , ∀n ∈ N∗ : f(n) là số chính phương. (2.11) Đặc biệt, từ đó ta có    f(1) = a + b + c + 1 là số chính phương(i) f(2) = 4a + 2b + c + 8 là số chính phương(ii) f(3) = 9a + 3b + c + 27 là số chính phương(iii) f(4) = 16a + 4b + c + 64 là số chính phương(iv) Từ (ii) và (iv) ta có f(4) − f(2) = 2b (mod 4). Mà 2b là số chẵn, còn theo nhận xét thì f(4) − f(2) chỉ có thể ≡ 0, 1, −1 (mod 4) ⇒ 2b ≡ 0 (mod 4). Từ (i) và (iii) ta có f(3) − f(1) ≡ (2b + 2) (mod 4). Tương tự trên ta cũng có 2b + 2 ≡ 0 (mod 4) ⇒ 2 ≡ 0 (mod 4) là điều vô lí. Vậy giả sử của ta là sai, tức là (2.10) là mệnh đề đúng. Ví dụ 2.4.10. Cho a, b, c là các số thực dương. Chứng minh rằng a √ a2 + 8bc + b √ b2 + 8ca + c c2 + 8ab ≥ 1. Lời giải: Ta sẽ giải bài toán trên bằng phương pháp phản chứng như sau: Đặt x = a √ a2 + 8bc , y = b √ b2 + 8ca , z = c c2 + 8ab Khi đó, ta có 1 x2 − 1 1 y2 − 1 1 z2 − 1 = 64 và ta cần chứng minh x + y + z ≥ 1 Giả sử ngược lại, x + y + z < 1. Khi đó 1 x2 − 1 1 y2 − 1 1 z2 − 1 > (x + y + z)2 x2 − 1 (x + y + z)2 y2 − 1 (x + y + z)2 z2 − 1 = (2x + y + z) (y + z) (2y + x + z) (x + z) (2z + x + y) (x + y) x2y2z2 (2.12) 24
  • 29. Chương 2. Phương pháp chứng minh phản chứng Áp dụng bất đẳng thức Cauchy cho các dấu ngoặc trong V P(2.12) ta được: 2x + y + z = x + x + y + z ≥ 4 4 √ x.x.y.z 2y + x + z = y + y + x + z ≥ 4 4 √ y.y.z.z 2z + x + y = z + z + x + y ≥ 4 4 √ z.z.x.y y + z ≥ 2 √ y.z x + z ≥ 2 √ x.z x + y ≥ 2 √ x.y Thay vào biểu thức (2.12) ta được VP(2.12) > 192, mà theo giả thiết VT(2.12) = 64, do đó ta có (64 > 192) (mâu thuẫn). Vậy điều giả sử của ta là sai. Tức là bài toán được chứng minh. Ví dụ 2.4.11. Cho a, b, c là các số thực sao cho a+b+c > 0, ab+bc+ca > 0, abc > 0. Chứng minh rằng a, b, c > 0. Lời giải: (phản chứng). Giả sử một trong các số a, b, c < 0. Vì ta có abc > 0 nên phải có hai số âm, một số dương. Không mất tính tổng quát, giả sử a, b < 0, c > 0. Khi đó ab + bc + ca = a(a + b + c) − a2 + bc < 0 Mâu thuẫn với điều kiện ban đầu. Do đó giả sử của ta là sai. Vậy ta có điều phải chứng minh. Ví dụ 2.4.12. Chứng minh rằng với mỗi số nguyên dương k tồn tại một số nguyên dương n > 1, sao cho: Ci n ...k(∀i = 1, n − 1). Lời giải: (phản chứng). Giả sử Ci n ...k. Do đó, lấy k = 4, khi đó tồn tại một số nguyên dương n > 1 để Ci n ...4 (∀i = 1, n). Suy ra n−1 i=1 = Ci n ≡ 0 (mod 4). Mặt khác n−1 i=1 = Ci n = 2n − 2 ≡ −2 (mod 4). Do đó, ta có mâu thuẫn. Vậy điều giả sử của ta là sai. Ta được điều phải chứng minh. 25
  • 30. Chương 2. Phương pháp chứng minh phản chứng Ví dụ 2.4.13. Xét các số gồm 7 chữ số phân biệt được lập thành từ các số {1, 2, . . . , 7} . Liệu trong các số được lập có ba số a, b, c mà a + b = c. Hỏi có hai số khác nhau a, b mà a ...b. Lời giải: 1. Ta có: 1 + 2 + . . . + 7 = 28 ≡ 1 (mod 9). Vậy số bất kỳ trong các số nói trên khi chia cho 9 đều dư 1. Giả sử tồn tại ba số a, b, c mà a + b = c. Khi đó: a + b ≡ 2 (mod 9), còn c ≡ 1 (mod 9). Điều này vô lý, nên không tồn tại ba số thỏa mãn yêu cầu bài toán. 2. Giả sử có hai số a, b mà a ...b, tức là ∃n ∈ N∗ : a = nb. Do b.7 ≥ 1234567.7 > 7654321 ≥ a, nên 7b ≥ nb ⇔ n ≤ 7. Vì a = nb, nên a ≡ n (mod 9)(1 < n ≤ 7). Mặt khác, a ≡ 1 (mod 9). Điều này vô lý, nên trong các số được lập không có hai số nào, mà một trong hai số chia hết cho số kia. Ví dụ 2.4.14. Cho hai đường thẳng chéo nhau. Chứng minh rằng có duy nhất một mặt phẳng chứa đường thẳng này và song song với đường thẳng kia. Lời giải: Giả sử ta có hai đường thẳng chéo nhau a và b. lấy điểm M bất kỳ thuộc a. Qua M kẻ đường thẳng b′ song song với b. Gọi (α) là mặt phẳng xác định bởi a và b′. a b M b′ α Ta có: b//b′ và b′ ⊂ (α), từ đó suy ra b//(α). Hơn nữa (α) ⊃ a nên (α) là mặt phẳng cần tìm. Ta chứng minh (α) là mặt phẳng duy nhất. Thật vậy, giả sử có một mặt phẳng (β) khác (α), chứa a và song song với b thì khi đó (α), (β) là hai mặt phẳng phân biệt cùng song song với b nên giao tuyến của chúng là a, phải song song với b. 26
  • 31. Chương 2. Phương pháp chứng minh phản chứng Điều này mâu thuẫn với giả thiết a và b chéo nhau. Tương tự, ta có thể chứng minh có duy nhất một mặt phẳng chứa b và song song với a. Vậy ta được điều phải chứng minh. Ví dụ 2.4.15. Chứng minh rằng nếu mặt phẳng (α) chứa hai đường thẳng cắt nhau a, b và a, b cùng song song với mặt phẳng (β) thì (α) song song với (β) Lời giải: Gọi M là giao điểm của a và b. Vì (α) chứa a mà a song song với (β) nên (α) và (β) là hai mặt phẳng phân biệt. Ta cần chứng minh (α) song song với (β). Giả sử (α) và (β) không song song và cắt nhau theo giao tuyến c. M a b α β c Ta có    a//(β) (α) ⊃ a (α) ∩ (β) = c ⇒ c//a và    b//(β) (α) ⊃ b (α) ∩ (β) = c ⇒ c//b . Như vậy từ M kẻ được hai đường thẳng a, b cùng song song với c. Theo định lý 1, §2 (SGK hình học 11), điều này mâu thuẫn. Vậy (α) và (β) phải song song với nhau. 27
  • 32. Chương 3 Phương pháp suy luận trực tiếp 3.1 Vài nét về phương pháp suy luận trực tiếp Các bài toán logic hiểu theo nghĩa tương đối rộng gồm các bài toán logic, các bài toán không mẫu mực (không có cách giải nhất định) thường có nhiều cách giải khác nhau, trong đó có phương pháp suy luận trực tiếp. Phương pháp suy luận logic đã có từ xa xưa và để giải các bài toán logic người ta chỉ có duy nhất phương pháp này (sau này mới có thêm các phương pháp khác). Các bài toán logic đa dạng về đề tài, phong phú về chủng loại đòi hỏi chúng ta phải biết suy luận đúng đắn, chặt chẽ trên cơ sở vận dụng kiến thức cơ bản và kinh nghiệm sống của mình. Vì vậy cần phải luyện tập óc quan sát, cách lập luận, cách xem xét khả năng có thể xảy ra của một sự kiện và vận dụng những kiến thức đã học vào các tình huống muôn hình muôn vẻ trong cuộc sống hàng ngày. Đôi khi để giải những bài toán này, chỉ cần những kiến thức toán học đơn giản, nhưng lại đòi hỏi khả năng chọn lọc trường hợp, suy luận chặt chẽ, rõ ràng. Sự phát triển của toán học, chẳng hạn giải tích tổ hợp, phương pháp quy nạp, phản chứng, góp phần phong phú thêm phương pháp suy luận logic trực tiếp. Những bài toán trong chương này là những ví dụ không quá phức tạp giúp cho người đọc thấy được những kĩ năng cơ bản của tư duy logic. Thông qua lời giải giúp người đọc biết cách diễn đạt bằng ngôn ngữ logic những bài toán cùng dạng có thể gặp trong đời sống thực tiễn và bằng cách đó người đọc tìm ra lời giải dễ dàng nhanh gọn hơn. Mỗi bài toán là một ví dụ điển hình và có thể có nhiều bài toán khác tương tự được diễn đạt dưới nhiều hình thức khác nhau. Những bài toán giải bằng phương pháp suy luận trực tiếp rất có ích cho người học toán, dạy toán và những người quan tâm đến logic toán. Nó là bước tiếp 28
  • 33. Chương 3. Phương pháp suy luận trực tiếp cận đầu tiên đến logic toán học. Nó rèn luyện tư duy logic, khả năng phản xạ, trí thông minh, là hình thức thể thao trí tuệ phục vụ cho đông đảo người đọc đặc biệt là lứa tuổi học sinh ở nhiều cấp học khác nhau. Điều cơ bản của phương pháp này là thông qua việc phân tích các điều kiện của bài toán, cần tìm ra mối quan hệ logic giữa các mệnh đề. 3.2 Các ví dụ về vận dụng phương pháp suy luận trực tiếp Ví dụ 3.2.1. Có ba em chơi trò đội mũ. Em chủ trì giơ ba mũ đỏ, hai mũ xanh, rồi yêu cầu ba em ngồi theo hàng dọc và không nhìn về phía sau, rồi từ phía sau chụp lên đầu mỗi em một cái mũ còn hai cái cất đi. Chứng minh rằng trong mọi cách đội đều có một em nhận ra màu mũ của mình. Lời giải: Giả sử em A ngồi sau em B, em B ngồi sau em C, rõ ràng em A quan sát được mũ của em B và C còn em B quan sát được mũ của em C. - Trước hết do chỉ có hai mũ xanh nên nếu B và C đội mũ xanh thì A đoán được ngay mũ của mình là mũ đỏ. - Nếu A im lặng, chứng tỏ trong B và C có một em đội mũ xanh, một em đội mũ đỏ hoặc cả hai em đội mũ đỏ. Khi đó B sẽ phải suy nghĩ và quan sát C. Nếu C đội mũ xanh thì B đoán ngay mình đội mũ đỏ. Nếu B thấy C đội mũ đỏ thì B đành im lặng và C sẽ nhận biết ngay mình đội mũ đỏ. Lời giải của bài toán trên có thể tóm tắt theo sơ đồ sau A Đ im B X C X XĐ Hình 3.1 Như vậy, trong mọi trường hợp, đều có ít nhất một em đội mũ đỏ. Dấu hiệu để các em dự đoán được mũ của mình là số mũ xanh mà mình hoặc bạn mình quan sát được. Em đoán đúng màu mũ của mình sẽ luôn là em đội mũ đỏ. 29
  • 34. Chương 3. Phương pháp suy luận trực tiếp Ví dụ 3.2.2. (Vị sứ giả thông minh) Một viên quan nước Lỗ đi sang xứ Tề, bị vua Tề tuyên phạt tử hình và bị hành quyết hoặc chém đầu hoặc treo cổ. Trước khi hành quyết nhà vua cho sứ giả được nói một câu, nếu nói đúng thì bị chém đầu, nếu nói sai thì bị treo cổ. Sứ giả mỉm cười nói một câu, nhờ đó đã thoát chết. Bạn hãy cho biết câu nói của sứ giả đó như thế nào? Lời giải: Vị sứ giả đã nói rằng "Tôi sẽ bị treo cổ". Như vậy, nếu nhà vua đem treo cổ sứ giả thì sứ giả đó nói đúng. Mà theo điều lệnh sử phạt của nhà vua thì phải đưa sứ giả đi chém đầu (vì ông ta nói đúng). Nếu nhà vua chém đầu thì ông ta đã nói sai. Theo điều lệnh sử phạt của nhà vua thì phải đem sứ giả đi treo cổ. Thành thử nhà vua không thể hành quyết sứ giả bằng chém đầu cũng như treo cổ, nên sứ giả đã thoát chết. Ví dụ 3.2.3. Người bản sứ và tên thực dân Trước vành móng ngựa là ba người đàn ông, họ là người bản sứ hoặc tên thực dân. Quan tòa biết khi được hỏi người bản sứ bao giờ cũng nói thật, còn tên thực dân bao giờ cũng nói dối, nhưng quan tòa không biết ai là người bản sứ, ai là tên thực dân. Quan tòa hỏi người thứ nhất: "Anh là ai?". Nhưng anh ta nói ngọng nên quan tòa không hiểu câu trả lời. Hãy xác định câu trả lời của người thứ nhất? Lời giải: Nếu người được hỏi thứ nhất là tên thực dân thì theo bản chất của thực dân, anh ta sẽ trả lời "Tôi là người bản sứ". Nếu người đó là dân bản sứ thì theo bản chất của người dân bản sứ anh ta cũng trả lời "Tôi là người bản sứ". Như vậy câu trả lời của người thứ nhất chỉ có thể là: "Tôi là người bản sứ". Ví dụ 3.2.4. a. Trong một căn phòng có 10 người, biết rằng giữa ba người bất kỳ có hai người quen nhau. Chứng minh rằng, có thể tìm được bốn người mà hai người bất kỳ trong số đó đều quen nhau. b. Khẳng định trên có còn đúng nữa không nếu ở câu a số người trong phòng là 9. Lời giải: a. Giả sử bốn người bất kỳ có hai người không quen nhau. Khi đó A không thể có quá ba người không quen. Nếu A có bốn người không quen thì theo giả thiết giữa bốn người này có hai người không quen và họ cùng với 30
  • 35. Chương 3. Phương pháp suy luận trực tiếp A hợp thành bộ ba đôi một không quen nhau. Vậy A có không nhiều hơn ba người không quen, nghĩa là A có không ít hơn sáu người quen. Giả sử A quen với B1, B2, · · · , B6. Khi đó giữa B1, B2, · · · , B6 không có bộ ba nào đôi một quen nhau (Nếu khác thì bộ ba này hợp thành với A thành bộ bốn đôi một quen nhau- trái với giả thiết). Hơn nữa có bộ ba mà hai người không quen nhau trong số sáu người B1, B2, · · · , B6. Chẳng hạn, nếu B1 không quen với B2, B3, B4 thì B2, B3, B4 đôi một quen nhau. Vì thế B1 phải có ít nhất ba người quen trong số B2, B3, · · · , B6. Khi đó trong số ba người này không tìm được hai người quen nhau, ngược lại họ tạo với A và B1 thành bộ bốn người đôi một quen nhau. Trái với giả thiết, suy ra tồn tại bộ bốn mà hai người bất kỳ quen nhau. b. Ta chứng minh cho khẳng định trên vẫn đúng. Nếu người nào đó quen với tất cả sáu người thì chứng minh sẽ tương tự phần a. Nếu mỗi người quen có đúng năm người thì tổng số các cặp quen nhau sẽ là 9.5/2 không là số nguyên- điều này không thể xảy ra. Cuối cùng nếu tìm được một người nào đó không quen với ít nhất bốn người thì bốn người này phải đôi một quen nhau (nếu khác ta sẽ tìm được bộ ba đôi một không quen nhau) có nghĩa là tạo thành bộ bốn cần tìm- đó là điều phải chứng minh. Ví dụ 3.2.5. Mỗi bạn đạt giải mấy trong kỳ thi vô địch toàn quốc? Ba bạn Quân, Hùng, Mạnh vừa đạt giải nhất, nhì, ba trong kì thi học sinh giỏi toàn quốc. Biết rằng: 1. Không có học sinh trường chuyên nào đạt giải cao hơn Quân. 2. Nếu Quân đạt giải thấp hơn một bạn nào đó thì Quân không phải là học sinh trường chuyên. 3. Chỉ có duy nhất một bạn không học trường chuyên. 4. Nếu Hùng hoặc Mạnh đạt giải nhì, thì Mạnh đạt giải cao hơn bạn quê ở Hải Phòng. Hãy cho biết: Mỗi bạn đã đạt được giải nào? Bạn nào không học trường chuyên và bạn nào quê ở Hải Phòng? Lời giải: Ta nhận xét rằng: Nếu Quân đạt giải nhì hoặc ba, thì theo (2) Quân không học trường chuyên. Ta suy ra, theo (3) Hùng và Mạnh học trường chuyên, thành thử theo (1) Quân đạt giải nhất. Điều này vô lý. Vậy Quân phải đạt giải nhất. Trong hai bạn Hùng và Mạnh một người đạt giải nhì và một người đạt giải ba. Mạnh không thể đạt giải ba (vì theo (4) Mạnh còn đạt giải cao hơn bạn quê ở Hải Phòng). 31
  • 36. Chương 3. Phương pháp suy luận trực tiếp Vậy Mạnh phải đạt giải nhì, Hùng đạt giải ba. Đồng thời ta cũng suy ra Quân không học trường chuyên và Hùng quê ở Hải Phòng. Ví dụ 3.2.6. Ai đã nói đùa? Nhà trường cử thầy Nghiêm dẫn bốn học sinh Lê, Huy, Hoàng, Tiến đi thi đấu điền kinh. Kết quả ba em đạt giải nhất, nhì, ba và một em không đạt giải. Khi về trường mọi người hỏi kết quả các em trả lời như sau: Lê: Mình đạt giải nhì hoặc ba. Huy: Mình đã đạt giải. Hoàng: Mình được giải nhất. Tiến: Mình không được giải. Nghe xong thầy Nghiêm mỉm cười và nói: "Chỉ có ba bạn nói thật còn một bạn đã nói đùa " Hãy cho biết học sinh nào đã nói đùa và ai đạt giải nhất, ai không đạt giải? Lời giải: 1) Nếu Lê nói đùa thì ba bạn Huy, Hoàng, Tiến nói thật. Như vậy Lê và Hoàng cùng đạt giải nhất. Điều này vô lý. Vậy Lê phải nói thật. 2) Nếu Huy nói đùa thì Huy không được giải và cả ba bạn còn lại đều nói thật. Như vậy cả Huy, Tiến đều không đạt giải. Điều này trái với giả thiết của đầu bài. Vậy Huy phải nói thật. 3) Nếu Tiến nói đùa thì Tiến đạt giải và cả ba bạn còn lại đều đạt giải. Như vậy cả bốn bạn đều đạt giải. Điều này trái với giả thiết. Như vậy Tiến nói thật. Vậy Hoàng đã nói đùa. Có nghĩa là Hoàng đã đạt giải nhì hoặc ba cho nên Huy đạt giải nhất. Còn Tiến không đạt giải. Kết luận: Huy đạt giải nhất, Tiến không đạt giải và Hoàng nói đùa. Ví dụ 3.2.7. Điều mâu thuẫn ở đâu? Trong một tòa nhà chỉ có những cặp vợ chồng và những con nhỏ chưa lập gia đình. Ban điều tra dân số yêu cầu báo cáo về số người sống trong tòa nhà, đại diện là một anh thợ thích đùa báo cáo như sau: Sống trong tòa nhà bố mẹ nhiều hơn con cái. Mỗi con trai đều có một chị hay em gái. Số con trai nhiều hơn số con gái. Mỗi cặp vợ chồng đều có con. Người ta không thể chấp nhận được báo cáo đó (dù là đùa vui) vì trong đó có mâu thuẫn. Hãy chỉ ra điều mâu thuẫn trong báo cáo trên? Lời giải: Vì mỗi gia đình đều có con, mỗi con trai đều có một chị gái hay em gái, nên tất cả các gia đình đều có con gái. Suy ra số con gái ít nhất bằng số gia đình. 32
  • 37. Chương 3. Phương pháp suy luận trực tiếp Mặt khác, số con trai nhiều hơn số con gái, nên tổng số con nhiều hơn hai lần số gia đình, hay nhiều hơn số bố mẹ, điều này cho ta thấy mâu thuẫn trong báo cáo của anh thợ thích đùa ở câu đầu tiên "bố mẹ nhiều hơn con cái" với các câu tiếp theo. Ví dụ 3.2.8. Khi tổ chức múa hát tập thể một giáo viên đã xếp 20 nữ sinh và một số nam sinh thành vòng tròn sao cho đối diện với một nữ sinh qua tâm vòng tròn là một nam sinh. Hỏi trên vòng tròn này có hai nam sinh nào đứng kề nhau hay không? Lời giải: Giả sử không có hai nam sinh nào đứng kề nhau, vì vậy cũng không có hai nữ sinh nào đứng kề nhau. Do đó các bạn nam sinh và nữ sinh được xếp xen kẽ nhau trên vòng tròn, suy ra có tất cả là 20 nam sinh. Lấy hai bạn nam và nữ đứng ở vị trí đối xứng nhau qua tâm vòng tròn. Bắt đầu từ nữ sinh này ta đánh số các bạn đứng trên một nửa vòng tròn cho đến nam sinh đối diện. Rõ ràng bạn nữ này được đánh số 1 và bạn nam đối diện đánh số 21. Khi đó các bạn nữ lần lượt được đánh số 1, 3, 5, · · · , 19, còn các bạn nam được đánh số 2, 4, 6, · · · , 20. Do đó có hai bạn nam sinh được đánh số 20 và 21 đứng kề nhau. Điều này mâu thuẫn với giả thiết ban đầu. Vậy phải luôn luôn có hai nam sinh đứng kề nhau. Ví dụ 3.2.9. Trong tám viên bi có bề ngoài hoàn toàn giống nhau có một viên bi nặng hơn. Bằng hai lần cân trên đĩa (không được dùng quả cân). Hãy xác định viên bi nặng đó? Lời giải: Đầu tiên chia tám viên bi thành ba đống theo số lượng 3, 3, 2. Trong lần cân thứ nhất ta bỏ mỗi bên đĩa cân một đống gồm ba viên bi. Có hai khả năng xảy ra: a) Nếu cân thăng bằng. Thì viên bi nặng nằm trong đống hai viên bi. Khi đó, ta tiến hành cân hai viên bi ở đống gồm hai viên. Mỗi bên đĩa bỏ một viên bi ở đống hai viên bi. Viên bên chìm chính là viên bi nặng hơn. b) Nếu cân không thăng bằng, thì bên chìm chứa viên bi nặng hơn. Khi đó ta tiến hành cân lần thứ hai. Bỏ mỗi đĩa một viên bi trong ba viên bên chìm. Nếu cân thăng bằng, thì viên bi nặng hơn chính là viên bi còn lại nằm trên đĩa chìm. Nếu cân không thăng bằng, thì viên bi nặng hơn chính là viên bi bên chìm. Vậy chỉ bằng hai lần cân ta đã xác định được viên bi nặng hơn. 33
  • 38. Chương 3. Phương pháp suy luận trực tiếp Ví dụ 3.2.10. Học sinh lớp 11 và 12 tổ chức thi đấu cờ với nhau. Số học sinh lớp 11 tham gia gấp 10 lần số học sinh lớp 12 tham gia thi đấu xong điểm học sinh lớp 11 gấp 4, 5 lần số điểm học sinh lớp 12. Hãy tính xem có bao nhiêu học sinh tham gia đấu cờ. (Nội quy thi đấu là mỗi người thi đấu một lần với tất cả những người còn lại, người thắng ghi 1 điểm người thua ghi 0 điểm). Biết rằng tất cả các trận đấu không có trận nào hòa. Lời giải: Gọi số học sinh lớp 12 tham gia đấu cờ là x. Khi đó số học sinh lớp 11 tham gia đấu cờ là 10x và có tất cả là 11x em tham gia đấu cờ. Số trận đấu có tất cả là 11x(11x−1) 2 trận và cũng có chừng ấy điểm thắng. Số điểm thắng của học sinh lớp 11 đạt được là 4, 5 5, 5 × 11x (11x − 1) 2 = 4, 5x (11x − 1) Các em học sinh lớp 11 sẽ thi đấu với nhau 10x(10−1) 2 trận và sẽ có số điểm là 5x (10x − 1) Hiển nhiên ta phải có 4, 5x(11x − 1) ≥ 5x(10x − 1) 99x2 − 9x ≥ 100x2 − 10x x ≥ x2 . Điều này chỉ xảy ra khi x = 1. Vậy có 11 em tham gia đấu cờ. 34
  • 39. Chương 4 Phương pháp đồ thị Rất nhiều bài toán có thể giải bằng cách đưa về bài toán trên đồ thị rồi suy ra đáp án. 4.1 Một số khái niệm và kết quả cơ bản của lí thuyết đồ thị Trên mặt phẳng hay trong không gian lấy n điểm. Giữa một số cặp điểm được nối bằng những đoạn thẳng hay đoạn cong được định hướng hoặc không. Người ta gọi hình nhận được là dạng biểu diễn hình học của đồ thị hay một đồ thị. Các điểm đã chọn được gọi là đỉnh của đồ thị. Các đoạn thẳng hay đoạn cong đã nối được gọi là cạnh của đồ thị. Nếu cạnh a nối giữa hai điểm A, B thì A, B được gọi là các đỉnh của cạnh a. Cặp đỉnh x, y được gọi là hai đỉnh kề nhau, nếu chúng khác nhau và là hai đầu của cùng một cạnh. Dãy α các đỉnh x1, x2, · · · , xi, xi+1, · · · , xm−1, xm được gọi là một đường, nếu với mọi chỉ số i(1 ≤ i ≤ m − 1) đều có xi và xi+1 là hai đỉnh kề nhau. Các đỉnh x1, xm được gọi là các đỉnh đầu của đường α. Người ta còn nói rằng đường α nối giữa đỉnh x1 và đỉnh xm. Chu trình là một đường có hai đầu trùng nhau. Chu trình mà nó đi qua mỗi đỉnh không quá một lần được gọi là chu trình sơ cấp. Chu trình (α) được gọi là chu trình Hamilton, nếu nó đi qua tất cả các đỉnh của đồ thị và qua mỗi đỉnh đúng một lần. 35
  • 40. Chương 4. Phương pháp đồ thị Đồ thị G được gọi là đồ thị liên thông, nếu mỗi cặp đỉnh của nó đều có đường nối với nhau. Đồ thị G được gọi là đồ thị đầy đủ nếu mỗi cặp đỉnh của nó được nối với nhau bằng đúng một cạnh. Số cạnh xuất phát từ đỉnh x được gọi là bậc của đỉnh x. Cây là đồ thị liên thông và không có chu trình. Trong cây T tách ra một đỉnh được gọi là đỉnh gốc, còn các đỉnh có bậc bằng 1 và không phải gốc được gọi là lá hay đỉnh ngọn. Định lý 4.1.1. Đồ thị mà trong đó tổng bậc của hai đỉnh tùy ý đều không nhỏ hơn số đỉnh của đồ thị, liên thông. Định lý 4.1.2. Đồ thị mà trong đó bậc của mỗi đỉnh đều không nhỏ hơn 2, luôn luôn có chu trình sơ cấp. Hệ quả 4.1.1. Nếu trong đồ thị có đúng 2 đỉnh bậc 1, các đỉnh khác có bậc không nhỏ hơn 2, thì trong G có đường nối giữa hai đỉnh bậc 1. Định lý 4.1.3. Đồ thị, mà trong đó tổng bậc của hai đỉnh tùy ý không nhỏ hơn số đỉnh của đồ thị, luôn luôn có chu trình Hamilton. Định lý 4.1.4. Trong một đồ thị tùy ý số đỉnh, mà mỗi đỉnh có bậc lẻ, luôn luôn là một số chẵn. Định lý 4.1.5. Cho dãy số nguyên dương a1 = 2, a2 = 5, · · · , an+1 = (n+1)an +1. Khi đó đồ thị đầy đủ an+1 đỉnh với các cạnh được tô bằng n màu luôn luôn có tam giác cùng màu (chu trình gồm ba cạnh cùng màu). Định lý 4.1.6. Cho dãy số nguyên b2 = 3, b3 = 6, bn+1 = (bn −1)n+2. Đồ thị đầy đủ G với bn+1 − 1 đỉnh (n ≥ 2) và các cạnh được tô bằng n màu, sao cho không có tam giác cùng màu, thì trong đồ thị G có hình 5 cạnh với các cạnh cùng màu và các đường chéo được tô các màu khác. 4.2 Phương pháp đồ thị Để giải bài toán T bằng cách thông qua đồ thị, cần thực hiện lần lượt hai bước sau 36
  • 41. Chương 4. Phương pháp đồ thị 4.2.1 Xây dựng đồ thị mô tả các quan hệ Lấy các điểm trên mặt phẳng hoặc trong không gian tương ứng với các đối tượng đã cho trong bài toán. Dùng ngay các kí hiệu đối tượng để ghi trên điểm tương ứng... Cặp điểm x, y được nối với nhau bằng một cạnh với "đặc điểm t", khi và chỉ khi các đối tượng x, y có quan hệ (t) với nhau. Khi đó bài toán T đã được chuyển về bài toán D trên đồ thị. 4.2.2 Dựa vào các kết quả của lý thuyết đồ thị hoặc lý luận trực tiếp suy ra đáp án của bài toán D Nếu đáp án của bài toán D còn dưới dạng "ngôn ngữ đồ thị", thì căn cứ vào phép đặt tương ứng khi xây dựng đồ thị mà diễn đạt thành đáp án bằng ngôn ngữ thông thường (tức là đáp án của bài toán T). 4.3 Một số ví dụ Ví dụ 4.3.1. Trong một cuộc thi đấu bóng bàn An và Bình quy ước với nhau: Người thắng cuộc là người đầu tiên thắng ba ván hoặc thắng hai ván liên tiếp. Hãy xác định số khả năng có thể xảy ra? Lời giải: Dùng A để kí hiệu An thắng, B để kí hiệu Bình thắng. Dùng cây để mô tả toàn bộ hiện trạng có khả năng xảy ra. Xây dựng cây: Xuất phát từ điểm S. Ván đầu tiên có hai khả năng xảy ra: An thắng hoặc Bình thắng, nên lấy hai điểm sao cho hai điểm này với S không thẳng hàng. Một trong hai điểm này ghi A, còn điểm kia ghi B. Nối S với A bằng một đoạn thẳng hoặc một đoạn cong biểu thị A thắng. Tương tự, để biểu thị B thắng nối S với B bằng một đoạn thẳng hoặc một đoạn cong. Ván thứ hai lại có hai khả năng: An thắng hoặc Bình thắng, nên xuất phát từ A cũng lấy hai điểm mới và ghi các kí hiệu tương ứng A, B và từ A kẻ hai đoạn thẳng hoặc hai đoạn cong tới hai điểm mới thêm. Đối với điểm B cũng chọn thêm hai đỉnh mới ghi A và B, rồi từ B kẻ hai đoạn thẳng hay hai đoạn cong tới hai điểm mới thêm. Tiếp theo thực hiện kéo dài các đường một cách tương tự, nhưng do quy ước của An và Bình những đường mà trên đó xuất hiện hoặc hai đỉnh liên tiếp ghi 37
  • 42. Chương 4. Phương pháp đồ thị cùng bằng một kí hiệu hoặc có ba đỉnh được ghi bằng cùng một kí hiệu đều không được kéo dài. S A B B B B B B BB B A A A A A AA A Hình 4.1 Vì An và Bình đấu với nhau năm ván, thì hoặc có người thắng hai ván liên tiếp hoặc có người thắng ba ván. Do đó những đường xuất phát từ S đều không có quá năm cạnh. (Hình 4.1) Cây có 10 đỉnh ngọn nên có 10 khả năng xảy ra. Ví dụ 4.3.2. Trên đảo có một số cụm dân cư. Mỗi cụm dân cư có hai đường lớn và ba đường mòn đi ra. Mỗi đường lớn cũng như đường mòn đều dẫn tới một cụm dân cư khác. Hai cụm dân cư khác nhau bất kì được nối liền bằng hoặc đường lớn hoặc đường mòn. Hỏi trên đảo này có bao nhiêu đường mòn, bao nhiêu đường lớn? Lời giải: Trước hết ta cần khẳng định rằng trên đảo có sáu cụm dân cư. Thật vậy, mỗi cụm dân cư ta biểu diễn bằng một điểm. Hai cụm dân cư có đường lớn (đường mòn) nối với nhau, thì hai cụm tương ứng được nối bằng một đường nét liền (đường nét đứt). Vì xuất phát từ mỗi cụm dân cư có hai đường 38
  • 43. Chương 4. Phương pháp đồ thị lớn và ba đường mòn đi ra, nên xuất phát từ mỗi điểm đã chọn, chẳng hạn A, có hai đường nét liền và ba đường nét đứt. Bởi vậy mỗi điểm đã chọn A được nối với các điểm khác B, C, D, E, F bằng một đường nét liền hoặc một đường nét đứt. Như vậy phải có ít nhất sáu cụm dân cư (6 điểm đã chọn). Mặt khác hai cụm dân cư tùy ý đều phải có đường nối với nhau, nên mỗi điểm đều chỉ có thể nối tới với năm điểm. Do đó không còn điểm nào ngoài sáu điểm A, B, C, D, E, F. Bởi vậy, có sáu đường lớn và chín đường nhỏ. Căn cứ vào yêu cầu của bài toán ta có thể đưa ra một vài khả năng xảy ra các đường nối giữa các cụm dân cư như hình 4.2. B C D E F A A B C C D E F Hình 4.2 Ví dụ 4.3.3. Tại một giải bóng đá có bốn đội Anh, Đan Mạch, Hà Lan, Thụy Điển vào bán kết. Có mấy dự đoán xếp hạng như sau 1. Đan Mạch vô địch, Thụy Điển nhì. 2. Đan Mạch nhì, Hà Lan ba. 3. Anh nhì, Hà Lan tư. Kết quả là mỗi dự đoán đúng được một đội. Hãy cho biết kết quả xếp hạng của các đội? Lời giải: Dùng xi để kí hiệu đội x được xếp hạng i(1 ≤ i ≤ 4). Ta vẽ cây, hai nhánh đầu tiên ứng với dự đoán thứ nhất là D1, T2. Từ mỗi nhánh này ta lại có hai nhánh tương ứng với dự đoán thứ hai. Tiếp tục rẽ nhánh với dự đoán thứ ba. Ta chọn đường đi từ gốc 0 tới các điểm thỏa mãn các điều kiện: • Một đội không thể được xếp hai hạng khác nhau. • Hai đội không thể xếp cùng một hạng. 39
  • 44. Chương 4. Phương pháp đồ thị Suy ra chỉ có đường đi D1H3A2 thỏa mãn. O D1 D2 A2 A2 A2 A2 H4 H4 H4 H4 H3H3 D2 T2 Hình 4.3 Đường tô đậm D1H3A2 thỏa mãn điều kiện mỗi dự đoán đúng được một đội mà thứ tự ghi trên đường. Vậy kết quả xếp hạng như sau: Đan Mạch vô địch. Anh nhì. Hà Lan ba. Còn lại là Thụy Điển thứ tư. Ví dụ 4.3.4. Cho 6 số nguyên dương tùy ý. Chứng minh rằng luôn có thể chọn ra được 2 bộ 3 số mà trong mỗi bộ, từng đôi một đều là nguyên tố cùng nhau hoặc đều không nguyên tố cùng nhau. Lời giải: Bước 1: Chuyển bài toán sang bài toán về đồ thị màu G(X, E). - Đỉnh: Cho tương ứng mỗi số với một đỉnh X = {A, B, C, D, E, F} - Cạnh: Đoạn nối giữa hai đỉnh tương ứng với hai số • Nguyên tố cùng nhau được tô màu xanh (đường nét đứt). • Không nguyên tố cùng nhau được tô màu đỏ (đường nét liền). Khi đó, ta được bài toán đồ thị màu: Trong đồ thị đầy đủ G(X, E) có 6 đỉnh với hai cạnh màu, thì luôn luôn có thể tìm được hai tam giác với cạnh cùng màu. Giả sử đỉnh A là mút của AB, AC, AD cùng màu đỏ (đường nét liền) như hình vẽ dưới đây 40
  • 45. Chương 4. Phương pháp đồ thị A B C D Hình 4.4 Ta xét tam giác được lập nên từ ba đỉnh đối của A là BCD. Trong tam giác BCD hai khả năng có thể xảy ra: 1) Có ít nhất một cạnh màu đỏ. Chẳng hạn cạnh BC màu đỏ, khi đó tam giác ABC có cạnh màu đỏ. (Hình 4.5a) 2) Tam giác BCD không có cạnh nào màu đỏ. Nghĩa là tam giác BCD có cạnh đều là màu xanh. (Hình 4.5b) A BC F E D A B D C Hình 4.5a Hình 4.5b Vậy với mọi trường hợp, trong đồ thị G đều có tam giác cùng màu (theo Định lý (4.1.5)) Giả sử rằng trong G có tam giác màu đỏ, chẳng hạn tam giác ABC màu đỏ. Ta chứng minh tiếp G còn có tam giác thứ hai nữa với các cạnh cùng màu. Nếu trong G, ta tạm thời không xét đến một đỉnh của tam giác ABC, chẳng hạn đỉnh A cùng tất cả các cạnh thuộc nó. Ta được đồ thị con đầy đủ G1 có 5 đỉnh. Nếu trong G1 có tam giác cùng màu, thì bài toán đã được giải xong. 41
  • 46. Chương 4. Phương pháp đồ thị Ngược lại, trong G1 không có tam giác cùng màu, thì theo Định lý (4.1.6) với n = 2, ta có thể biểu diễn G1 thành một hình 5 cạnh với các cạnh màu đỏ và đường chéo màu xanh. Bây giờ, ta khôi phục lại đỉnh thứ 6 A và các cạnh màu thuộc nó. Xét hai cạnh AD và AF nếu chúng đều màu xanh thì ta có tam giác mới ADF có màu xanh. Nếu AD hoặc AF màu đỏ thì ta có tam giác màu đỏ nữa được hình thành là ABD hoặc ACF. Vậy ngoài tam giác ABC, ta có thêm một tam giác nữa có các cạnh cùng màu. Khẳng định đã được chứng minh. C F E D B C B D E F AHình 4.6 Bước 3: Thuyết minh lời giải Trong G luôn tồn tại hai tam giác với cạnh cùng màu. Nếu cả hai tam giác đều màu đỏ, thì ta có hai bộ ba số, mà trong mỗi bộ chúng đôi một nguyên tố cùng nhau. Nếu chỉ có một tam giác màu đỏ, thì ta được một bộ ba số đôi một nguyên tố cùng nhau, và một bộ ba số đôi một không nguyên tố cùng nhau. Nếu cả hai tam giác màu xanh, nghĩa là ta được hai bộ ba số, mà trong mỗi bộ, chúng đôi một không nguyên tố cùng nhau. Ví dụ 4.3.5. Để mừng con đạt giải trong kì thi toán quốc tế lần thứ 42, một gia đình dự định mời tiệc. Trong số khách mời: a) Người chồng muốn có ít nhất ba người từng đôi một quen nhau. 42
  • 47. Chương 4. Phương pháp đồ thị b) Người vợ lại muốn có ít nhất bốn người từng đôi một chưa quen nhau. Hỏi họ phải mời ít nhất bao nhiêu bạn, để mong muốn của chồng của vợ được thỏa mãn? Lời giải: Bước 1: Xây dựng đồ thị G(X, E) mô tả quan hệ của các đối tượng. • Đỉnh: Mỗi khách cho tương ứng với một đỉnh. • Cạnh: Đoạn nối giữa hai đỉnh tương ứng với hai khách: – Quen nhau được tô màu xanh (nét liền). – Không quen nhau tô màu đỏ (nét đứt). Khi đó đồ thị G(X, E) mô tả quan hệ quen biết của các khách dự kiến mời. Theo cách xây dựng trên, để thực hiện yêu cầu của người chồng, thì trong số khách mời phải có 3 người quen nhau từng đôi một, nên đồ thị G(X, E) phải có ít nhất một tam giác màu xanh, còn để thỏa mãn yêu cầu của người vợ, thì trong số khách mời phải có ít nhất 4 người từng đôi một không quen biết nhau, nên trong G(X, E) phải có ít nhất một tứ giác mà các cạnh và các đường chéo của nó phải có màu đỏ. Do đó bài toán được dẫn về bài toán trên đồ thị: Tìm số đỉnh ít nhất để đồ thị đầy đủ G(X, E) với các cạnh màu xanh hoặc đỏ mà a) Hoặc có ba đỉnh được nối với nhau bởi cạnh màu xanh, b) Hoặc bốn đỉnh từng đôi một được nối với nhau bởi cạnh màu đỏ. Bước 2: Giải bài toán đồ thị G 1) Nếu G có 8 đỉnh thì có thể xảy ra trường hợp mà cả (a) lẫn (b) không thỏa mãn. Như cách tô màu G ở hình 4.7, thì không có tam giác nào xanh, hoặc tứ giác nào với cạnh và đường chéo cùng màu đỏ. 43
  • 48. Chương 4. Phương pháp đồ thị x1 x2 x3 x4 x5 x6 x7 x8 Hình 4.7 2) Xét G(X, E) đầy đủ với 9 đỉnh, các cạnh được tô bằng hai màu xanh, đỏ. X = {x1, x2, . . . , x8, x9} Hai khả năng có thể xảy ra: 10) Có 4 cạnh màu xanh xuất phát từ một đỉnh. Chẳng hạn x1 có 4 cạnh x1x2, x1x3, x1x4, x1x5 màu xanh. Khi đó nếu trong các cạnh nối đôi một giữa 4 đỉnh x2, x3, x4, x5 có một cạnh xanh, giả sử cạnh x3x4 xanh, thì có tam giác x1x3x4 cạnh xanh hình 4.8 a được thỏa mãn, nếu không thì tứ giác x2x3x4x5 có cạnh và các đường chéo màu đỏ hình 4.8 b, (b) được thỏa mãn. 44
  • 49. Chương 4. Phương pháp đồ thị x1 x2 x3 x4 x5 x1 x2 x3 x4 x5 Hình 4.8 a Hình 4.8 b 20) Có nhiều nhất là 3 cạnh màu xanh xuất phát từ mỗi đỉnh. Bởi vậy, phải có ít nhất một đỉnh, x1 chẳng hạn, là đầu mút của không quá 2 cạnh màu xanh (vì nếu mỗi đỉnh là đầu mút của đúng 3 cạnh màu xanh, thì số cạnh màu xanh là 3×9 2 /∈ N). Khi đó, tại x1 phải là đầu mút của ít nhất 6 cạnh đỏ. Giả sử 6 cạnh đỏ đó là x1x4, x1x5, x1x6, x1x7, x1x8, x1x9. Xét đồ thị con đầy đủ G1 có 6 đỉnh x4, x5, . . . , x9 với 2 màu cạnh. Theo định lý (4.1.5) với n = 2, trong G1 có tam giác cùng màu. Nếu là tam giác màu xanh, thì (a) được thỏa mãn, nếu là tam giác màu đỏ thì 3 đỉnh này cùng với đỉnh x1 tạo thành đồ thị con đầy đủ có 4 đỉnh với các cạnh đều màu đỏ, (b) được thỏa mãn. x1 x2 x3 x4 x5 x6 x7 x8x9 Hình 4.9 Bước 3: Chuyển về ngôn ngữ xuất phát. 45
  • 50. Chương 4. Phương pháp đồ thị Đồ thị đầy đủ với 9 đỉnh, các cạnh được tô bằng hai màu xanh, đỏ thì luôn có hoặc tam giác xanh hoặc tứ giác mà các cạnh và đường chéo đều màu đỏ, nhưng với 8 đỉnh thì tính chất trên không tồn tại. Vậy số đỉnh của đồ thị G phải ít nhất bằng 9, nên số khách ít nhất họ phải mời là 9, thì mong muốn của vợ hoặc chồng được thỏa mãn. Ví dụ 4.3.6. Lấy n(n ≥ 4) số nguyên dương khác nhau tùy ý, sao cho cứ 4 số bất kỳ có ít nhất một số có ước chung với 3 số còn lại. Chứng minh rằng trong các số đã chọn ra có ít nhất n − 3 số, mà mỗi số này có ước chung với tất cả các số đã chọn ra. Lời giải: Bước 1: Xây dựng đồ thị mô tả các quan hệ. Đỉnh: Lấy n điểm trên mặt phẳng hay trong không gian tương ứng với các số đã chọn ra và ghi các số này lên các điểm tương ứng. Cạnh: Cặp điểm x, y được nối bằng một đoạn thẳng hay đoạn cong không đi qua các điểm tương ứng trung gian khác khi và chỉ khi cặp x, y có ước chung. Đồ thị G nhận được mô tả toàn bộ quan hệ có ước chung giữa các số đã chọn ra. Bước 2: Chứng minh trong đồ thị G có ít nhất n − 3 đỉnh có bậc n − 1 a) Nếu trong G có hai cặp đỉnh không kề nhau, thì chúng phải có đỉnh chung. Thật vậy, giả sử trong G có hai cặp đỉnh không kề nhau, chẳng hạn A, B và C, D nhưng chúng không có đỉnh chung. Khi đó trong 4 đỉnh này không có một đỉnh nào kề với 3 đỉnh còn lại. Ta đi tới mâu thuẫn với giả thiết. Vậy hai cặp A, B và C, D phải có một đỉnh chung chẳng hạn B ≡ D. b) Trong đồ thị G có không ít hơn n − 3 đỉnh có bậc n − 1 - Nếu trong G tất cả các đỉnh đều kề nhau từng đôi một, thì cả n đỉnh của G đều có bậc n − 1. - Nếu trong G có duy nhất một cặp đỉnh không kề nhau, thì G có n − 2 đỉnh bậc n − 1. - Giả sử G có hai cặp đỉnh không kề nhau, khi đó theo phần a) hai cặp đỉnh này phải có đỉnh chung. Giả sử các đỉnh đó là A, B, C. Nếu sử dụng đường nét đứt để biểu thị tính không kề nhau của các đỉnh ta có hình 4.10a. Kết nạp đỉnh E tùy ý vào nhóm A, B, C. Khi đó trong bộ 4: A, B, C, E phải có ít nhất một đỉnh kề với 3 đỉnh còn lại. Đỉnh này không thể là các đỉnh A, B, C mà phải là E. Vậy E kề với cả ba đỉnh A, B, C. (Hình 4.10b) 46
  • 51. Chương 4. Phương pháp đồ thị A B C A B C E Hình 4.10a Hình 4.10b Loại bỏ một trong ba đỉnh A, B, C khỏi bộ 4, chẳng hạn C và kết nạp đỉnh tùy ý F. Khi đó trong bộ 4 mới A, B, E, F lại có ít nhất một đỉnh kề với 3 đỉnh còn lại. Nếu F là đỉnh kề với 3 đỉnh còn lại, thì nó phải kề với E, nên trong mọi trường hợp E vẫn là đỉnh kề với 3 đỉnh còn lại. (Hình 4.11) A B E F Hình 4.11 Vì F là đỉnh tùy ý của đồ thị G và khác với các đỉnh A, B, C, E nên E kề với tất cả các đỉnh còn lại của G. Bởi vậy, bậc của E bằng n − 1. Đỉnh E lại được chọn một cách tùy ý trong các đỉnh của đồ thị G khác với A, B, C nên trong G chỉ có ba đỉnh A, B, C bậc nhỏ hơn n − 1. Bước 3: Thuyết minh lời giải Theo cách xác định cạnh của đồ thị G cặp đỉnh x, y kề nhau khi và chỉ khi cặp số x, y có ước chung, mà trong G là có ít nhất n − 3 đỉnh kề với tất cả các đỉnh còn lại. Do đó trong n số đã chọn ra có ít nhất n − 3 số có ước chung với tất cả các số còn lại. 47
  • 52. Chương 4. Phương pháp đồ thị Ví dụ 4.3.7. Một cơ quan cần tuyển ba người để thành lập một nhóm có đủ năng lực biên dịch các tài liệu từ sáu thứ tiếng: Anh, Pháp, Nga, Đức, Trung Quốc và Bồ Đào Nha sang tiếng Việt. Có bảy người đến dự tuyển, trong đó mỗi người đều biết hai và chỉ hai trong sáu thứ tiếng nói trên và bất cứ hai người nào cũng biết nhiều nhất một thứ tiếng chung trong sáu thứ tiếng đó. Biết rằng thứ tiếng nào cũng có ít nhất hai người biết. Liệu có thể xảy ra trường hợp không tuyển chọn được như yêu cầu đã nêu hay không? Lời giải: Ta sẽ chứng minh khẳng định đúng, tức là có thể tuyển chọn được như yêu cầu đã nêu. 1. Trước hết chứng minh khẳng định Cho đơn đồ thị G có 6 đỉnh và 7 cạnh, bậc của mỗi đỉnh lớn hơn hoặc bằng 2. Khi đó trong G luôn tìm được 3 cạnh đôi một không kề nhau. a) Đồ thị G liên thông. Thật vậy, nếu G không liên thông thì do G gồm 6 đỉnh với bậc không bé hơn 2, phải chia thành 2 tam giác nên G chỉ có 6 cạnh. Ta đi tới mâu thuẫn với điều kiện đồ thị gồm 7 cạnh. b) Đồ thị phải có chu trình Thật vậy, nếu G không có chu trình và G lại liên thông nên G là một cây. Khi đó G có đỉnh treo, tức là đỉnh bậc 1, nên mâu thuẫn với điều kiện bậc của mỗi đỉnh thuộc G đều không bé hơn 2. c) Vì 7 × 2 = 14 = 6 × 2 + 2, nên G phải có 2 đỉnh bậc 3 hoặc 1 đỉnh bậc 4, các đỉnh còn lại bậc 2. Ta xét từng trường hợp: Trường hợp a: G có một đỉnh bậc 4 (chẳng hạn đỉnh A). Không mất tính tổng quát, ta giả sử 4 cạnh xuất phát từ A là AB, AD, AN, AT. Lúc đó đỉnh P không thể kề với A, phải kề với 2 đỉnh khác, chẳng hạn P và N và 2 đỉnh còn lại (B và D) phải kề nhau. Khi đó ba cạnh không kề nhau từng đôi một là BD, NT, PA. (Hình 4.12) 48
  • 53. Chương 4. Phương pháp đồ thị A B D N P T Hình 4.12 Trường hợp b: G có hai đỉnh bậc 3. Giả sử A và P là hai đỉnh bậc 3. Vì G liên thông, nên có ít nhất một đường đi từ A đến P. Ta tạm bỏ đường đó đi (vẫn giữ nguyên các đỉnh A và P). Có thể xảy ra hai khả năng: b1) G không còn liên thông. Như vậy, ta có hai chu trình sơ cấp phân biệt, mỗi chu trình gồm 3 cạnh. Suy ra ta chọn cạnh AP và hai cạnh không kề với nó là BT, ND. (Hình 4.13) T B A P N D Hình 4.13 b2) G vẫn liên thông. Như vậy ta còn một chu trình sơ cấp (vì mỗi đỉnh đều có bậc 2). Ta có 3 đường đi từ A đến P (một đường đã tạm bỏ, hai đường theo chu trình), trong đó đường ngắn có độ dài 1 hoặc độ dài 2. 49
  • 54. Chương 4. Phương pháp đồ thị N A B T P D A N B P T D Hình 4.14a Hình 4.14b - Khi AP có độ dài 1, trên chu trình sơ cấp có độ dài 6 ta luôn chọn được ba cạnh không kề nhau AD, PT, BN. (Hình 4.14a) - Khi AP có độ dài 2, chỉ việc chọn một cạnh thuộc đường đi ngắn nhất từ A đến P và hai cạnh không kề với nó trên chu trình sơ cấp còn lại, chẳng hạn AD, TP, NB hoặc DP, AT, NB. (Hình 4.14b) Vậy trong mọi trường hợp ta đều tìm được ba cạnh đôi một không kề nhau. Khẳng định được chứng minh. 2. Chuyển bài toán về dạng đồ thị Dùng 6 điểm tương ứng với 6 ngoại ngữ nói trên. Dùng ngay chữ cái đầu của tên ngoại ngữ để ghi tên các điểm tương ứng: A, P, N, D, T, B. Mỗi người biết hai ngoại ngữ nào đó được biểu thị bởi một cạnh nối hai đỉnh tương ứng với hai ngoại ngữ đó. Đồ thị nhận được kí hiệu là G. Có 7 người tham gia dự tuyển và mỗi người biết hai ngoại ngữ, do đó đồ thị G có 7 cạnh. Bất kỳ hai người dự tuyển nào cũng chỉ cùng biết tối đa một ngoại ngữ nói trên, nên trong đồ thị G không có một cặp đỉnh nào được nối bằng hai cạnh. Vậy G là một đồ thị đơn gồm 6 đỉnh 7 cạnh. Do đó, theo khẳng định trên, luôn luôn tìm được ba cạnh không kề nhau từng đôi một. Khi đó chỉ việc chọn ba người được biểu thị bằng 3 cạnh nói trên, thì mỗi người này đều biết hai ngoại ngữ và không có hai người nào cùng biết một ngoại ngữ. Do đó nhóm ba người này biết tất cả sáu ngoại ngữ: Anh, Pháp, Nga, Đức, Trung Quốc và Bồ Đào Nha. 50
  • 55. Chương 4. Phương pháp đồ thị Ví dụ 4.3.8. Mười bảy nhà khoa học đến dự hội nghị Quốc tế. Mỗi người trong số họ chỉ biết một trong ba ngoại ngữ: Anh, Nga, Pháp. Chứng minh rằng có ít nhất 3 nhà khoa học cùng biết một trong ba ngoại ngữ nói trên. Lời giải: 1. Xây dựng đồ thị mô tả quan hệ. -) Đỉnh: Lấy 17 điểm trên mặt phẳng hay trong không gian tương ứng với 17 nhà khoa học. Dùng ngay tên các nhà khoa học để ghi trên các điểm tương ứng. -) Cạnh tô màu: Cặp điểm x, y được nối bằng đoạn thẳng hay đoạn cong tô • Màu xanh khi và chỉ khi hai nhà khoa học tương ứng cùng biết tiếng Pháp. • Màu vàng khi và chỉ khi hai nhà khoa học tương ứng cùng biết tiếng Anh. • Màu đỏ khi và chỉ khi hai nhà khoa học tương ứng cùng biết tiếng Nga. Đồ thị G nhận được mô tả toàn bộ hiện trạng về ngoại ngữ của các nhà khoa học đến dự hội nghị. 2. Đáp án của bài toán bằng ngôn ngữ đồ thị Đồ thị G gồm 17 đỉnh, đầy đủ và các cạnh được tô bằng ba màu, nên theo Định lý 4.1.5, với n = 3, có tam giác cùng màu, khi đó ba nhà khoa học tương ứng với các đỉnh của tam giác cùng biết một trong ba ngoại ngữ: Anh, Pháp, Nga. Ví dụ 4.3.9. Một quần đảo có 2n(n ≥ 1) hòn đảo. Mỗi hòn đảo có đường ngầm nối trực tiếp với ít nhất n hòn đảo khác. Chứng minh rằng từ một hòn đảo bất kỳ thuộc quần đảo đều có thể đi tới bất kỳ hòn đảo nào khác thuộc quần đảo này bằng đường ngầm. Lời giải: 1. Xây dựng đồ thị mô tả các quan hệ • Đỉnh: Lấy 2n điểm tương ứng với 2n hòn đảo. Dùng ngay tên các hòn đảo để ghi các điểm tương ứng. • Cạnh: Cặp điểm x, y được nối bằng một đoạn thẳng hoặc một đoạn cong không đi qua các điểm chung gian khác khi và chỉ khi hai hòn đảo tương ứng có đường ngầm nối với nhau. 51
  • 56. Chương 4. Phương pháp đồ thị Đồ thị nhận được kí hiệu bằng G. Nó mô tả toàn bộ mối liên hệ bằng đường ngầm của quần đảo. 2. Đáp án của bài toán bằng "ngôn ngữ đồ thị" Đối với mỗi đỉnh x của đồ thị G số cạnh xuất phát từ nó bằng đúng số đường ngầm xuất phát từ hòn đảo tương ứng. Bởi vậy bậc của mỗi đỉnh không nhỏ hơn n, nên tổng bậc của hai đỉnh tùy ý không nhỏ hơn số đỉnh của đồ thị (2n). Khi đó, theo Định lý (4.1.1), đồ thị G liên thông. Do đó hai đỉnh tùy ý x, y của đồ thị G có đường nối với nhau. Mặt khác, mỗi cạnh của đường này biểu thị một đường ngầm nối trực tiếp giữa hai hòn đảo, nên đường nối giữa hai đỉnh x, y biểu thị đường ngầm nối giữa hai hòn đảo tương ứng với x và y. Do đó từ hòn đảo bất kỳ đều có thể đi bằng đường ngầm tới các hòn đảo còn lại của quần đảo. 52
  • 57. Chương 5 Phương pháp bảng 5.1 Giới thiệu về phương pháp bảng Nhiều bài toán phổ thông có thể giải bằng cách lập bảng mô tả mối quan hệ giữa các đối tượng được cho trong bài toán. Đối với một số bài toán phổ thông trong đó xuất hiện hai hay nhiều đối tượng và các cặp phần tử nói lên mối quan hệ giữa các tệp người ta có thể thiết lập một hay nhiều bảng, để mô tả mối quan hệ giữa các tệp. Mỗi bảng này có hàng trên cùng ghi các phần tử của một tệp, còn cột tận cùng bên trái ghi các phần tử thuộc tập kia và các vị trí trong bảng ghi mã số quan hệ giữa các phần tử thuộc các tệp. Căn cứ vào các điều kiện đã cho trong bài toán gạch bỏ đi những cặp phần tử không thích hợp từ đó đi đến lời giải của bài toán. 5.2 Một số ví dụ minh họa Ví dụ 5.2.1. Trong buổi học nữ công ba bạn Cúc, Đào, Hồng làm ba bông hoa: cúc, đào, hồng. Bạn làm hoa hồng nói với bạn Cúc "Thế là trong chúng ta không có ai làm loại hoa trùng với tên mình". Hãy xác định tên hoa mà mỗi bạn đã làm? Lời giải: Bài toán này có hai tệp đối tượng. Tệp thứ nhất gồm các bạn làm hoa, tệp thứ hai gồm các bông hoa được làm. Ta có thể giải bằng phương pháp bảng như sau 1. Lập bảng 53
  • 58. Chương 5. Phương pháp bảng Bảng cần lập gồm 4 hàng và 4 cột. Hàng đầu, từ cột thứ hai ghi lần lượt tên các bông hoa được làm viết tắt là các chữ cái đầu, còn trên cột tận cùng bên trái từ hàng hai ghi lần lượt tên các bạn tham gia làm hoa viết tắt là chữ cái đầu viết hoa. 2. Điền mã số quan hệ vào các vị trí của bảng a) Căn cứ vào giả thiết: Mỗi bạn đều không làm hoa trùng với tên mình, mà điền mã "k" vào các ô nằm trên đường chéo chính. Nguoi c d h hoa C D H k k k k Bảng 5.1 b) Căn cứ vào câu "Bạn làm hoa hồng nói với bạn Cúc" suy ra bạn Cúc không phải làm hoa hồng, mà ghi mã "k" vào ô nằm ở hàng Cúc, cột hồng. 3. Loại bỏ vị trí không thỏa mãn quan hệ để nhận được lời giải Trong bảng trên cột cuối vị trí 1 và 3 bị gạch bỏ, nên vị trí duy nhất còn lại là vị trí thứ hai phải thỏa mãn quan hệ giữa người làm hoa và hoa được làm. Do đó bạn Đào làm hoa hồng. Vì trên hàng 2 Đào đã có vị trí thỏa mãn quan hệ nên toàn bộ hàng này bị loại ra khỏi diện xét. Bởi vậy cột Cúc chỉ còn vị trí cuối cùng trong diện xét. Bởi vậy nó phải thỏa mãn quan hệ giữa người làm hoa và hoa được làm, nên bạn Hồng làm hoa cúc. Từ đó suy ra người còn lại bạn Cúc phải làm hoa đào. Vậy Bạn Cúc làm hoa đào, Bạn Đào làm hoa hồng, Bạn Hồng làm hoa cúc. Ví dụ 5.2.2. Ba bạn Long, Hoan, Lan đều là học sinh giỏi toán. Cô giáo thưởng cho ba bạn thứ đồ chơi là: Búp bê, ô tô và con quay điện. Biết rằng bạn Long không thích chơi búp bê, bạn Hoan không nhận búp bê và cũng không thích ô tô. Hỏi cô giáo đã thưởng cho ai đồ chơi gì? Lời giải: Bài toán gồm hai nhóm đối tượng: 54
  • 59. Chương 5. Phương pháp bảng • Nhóm thứ nhất gồm các bạn: Long, Hoan, Lan. • Nhóm thứ hai là các đồ chơi mà cô giáo thưởng cho các bạn: Búp bê, ô tô, con quay. Ta lập một bảng có hàng trên là tên các bạn, và cột bên trái là các đồ chơi. Long Hoan Lan Búp bê 0 0 × Ô tô × 0 Con quay × Bảng 5.2 Nhìn vào bảng, ta suy luận và có kết quả sau: Lan được thưởng con búp bê. Long được thưởng ô tô. Hoan được thưởng con quay. Ví dụ 5.2.3. Có sáu em học sinh: An, Bình, Nam, Long, Trang, Minh tham gia cuộc thi leo núi. Có hai em về được đích, đáp lại câu hỏi ai về tới đích, có năm câu trả lời: 1. An và Nam. 2. Long và Trang. 3. Minh và Long. 4. An và Minh. 5. An và Linh. Thật ra trong năm câu trả lời trên có 4 câu mà mỗi câu chỉ đúng một nửa (đúng một tên), còn sai một nửa, còn một câu sai cả hai tên. Hỏi ai đã về tới đích của cuộc thi? Lời giải: Dựa vào năm câu trả lời, ta lập bảng sau: 55